184
Major Topic Abbreviation Major Topic Abbreviation Abnormalities of Teeth Abn ofTth Odontogenic Tnmors Odont Tum Disesses ofthe Blood Bld Dis Pigmetrted L€sions of the Orat Covitv Pig Les of Oral Cav CoDnectiye Tissue Lesions Con Tis Les Pseudocyst Pseudocysts Itrfl rmmatory Jaw Lesions Infl Jaw Les Red-Blue Lesiors R-B Lcsions Metabolic & Genetic Jaw Diseases M&G Jaw Dis Salivary Ghnd Tumors SG Tum Miscellaneous Misc. Terms Terms Neoplasms Neo Ulc€ratiYe.Conditions Ulc Cond Nerve & Muscl€ Disorders Nrv&Mus Disord Vesiculo-Bullous Diseases V-B Dis Non-Odontogenic Cysts N-O Cyst Verrucal-Prpillary Lesions V-P Les Non-Odontogenic Tumors N-O Tum White Lesions W Les Odontogenic Cysts Odont Cyst ORAL PATHOLOGY Abn of Tth The congenital absence of many, but not all, teeth is known as: . H,?odontia . Oligodontia . Diphyodontia . Anodontia 1 CopyriSht O 20ll'1012 ' Denul Decks

Oral Pathology

Embed Size (px)

Citation preview

Page 1: Oral Pathology

Major Topic Abbreviation Major Topic Abbreviation

Abnormalities of Teeth Abn ofTth Odontogenic Tnmors Odont Tum

Disesses ofthe Blood Bld Dis Pigmetrted L€sions ofthe Orat Covitv

Pig Les ofOral Cav

CoDnectiye Tissue Lesions Con Tis Les Pseudocyst Pseudocysts

Itrfl rmmatory Jaw Lesions Infl Jaw Les Red-Blue Lesiors R-B Lcsions

Metabolic & GeneticJaw Diseases

M&G Jaw Dis Salivary GhndTumors

SG Tum

Miscellaneous Misc. Terms Terms

Neoplasms Neo Ulc€ratiYe.Conditions Ulc Cond

Nerve & Muscl€Disorders

Nrv&Mus Disord Vesiculo-BullousDiseases

V-B Dis

Non-Odontogenic Cysts N-O Cyst Verrucal-PrpillaryLesions

V-P Les

Non-Odontogenic Tumors N-O Tum White Lesions W Les

Odontogenic Cysts Odont Cyst

ORAL PATHOLOGY Abn of Tth

The congenital absence of many, but not all, teeth is known as:

. H,?odontia

. Oligodontia

. Diphyodontia

. Anodontia

1

CopyriSht O 20ll'1012 ' Denul Decks

Page 2: Oral Pathology

Tlr o forms of anodontia:L Complete true: is a rare condition in which alt of the teeth are missing. It may in-volve both the primary and permanent dentitions. It is usually associated with heredi-tarv ectodermal dysplasia. See picture #l in booklet:. Partial anodontia (commonly referred to as congenitally missing teeth): is rathercor non. Teeth usually affected include the third molars (maxillary more often thannandibular), maxillary lateral incisors ald mandibular second premolars. Note: As ageneral rule, if only one or a few teeth are missing, the absent tooth will be the mostdistal tooth ofany glen type (if molat then it would be the third molar). See picture#2 in booklet

Other terms to b€ familiar with include:. Ofigodontia: refers to the congenital absence of many (usually six or more) but notall, teeth. Hypodontia: refers to the absence ofonly a few teeth. Diphyodontia: is having 2 successive sets of teeth (as in humans) as opposed topol)?hyodontia fraving more than two sets of teeth in a ldetime). Hypsodontia: having teeth with high crowns /cusps). Hypsodont dentition is assoc-iated with a diet ofabrasive foo<is

Page 3: Oral Pathology

PATHOLOGY

Which type of dentinogen€sis imperfecta features multiple pulp exposures,perirpical rtdiolucencies, and a variable radiographic appearance?

. Type I

' Type II

' Type III

2

CoDrighr C 20ll-201: - Denral Decks

The permanent maxillary centrals in the x-ray below are vital.What is the most probabl€ interpretation ofthe condition illustrated?

. Mesiodens

. Concrescence

. Fusion

. Dens in Dente

Cop).righr C 201 l'201 2 - Denlal Decks

Page 4: Oral Pathology

Dentinogenesis imp€rfecta is an autosomal dominant condition in which there is an intrinsicalteration in the dentin. lt affects dentin ofboth the primary and the permanent dentition. It isalso known as hereditary opalescent dentin.

Dentinogenesis imperfecta has been divided into three types:. Type I: dentin abnormality occurs in patients that have ost€ogenesis imperfecta fcra,'-

acterized blt blue sclera or a history of bone frqchu?s). In this form, primary teeth are more

severely affected than permanent teeth.. Tlpe lI: most common, only the dentin abnormality exists with no bone involvement.

'Tlpe III (Brandl'whe \pe) bke Type lI, only the dentin abnormality exists, however,there are clinical and radiographic vadations in this type. Features of type III that are notseen in type I and ll include multiple pulp exposures, periapical radiolucencies, and a

variable radiographic appearance

Important: Clinically, dentinogenesis imperfecta is usually easily detected and identified.The teeth exhibit an unusual translucent or opalescent appearance with color variation fiomyellorv-brorvn to gray. The entire crown appears discolored owing to the abnormal underlyingdentin. See picture #3 in bookl€t

Other characteristics ofthe teeth include:. Although the enamel is structurally and chemically normal, it fractures easily, resulting inrapid tvear (due to poor dentin support). There is excessive constdction at the CEJ, giving the crowns a tulip or bell shape. Short roots. Obliterated pulps (\'pe I and I1 only) *** Note' In Type IlI, the dentin appean thin, and

the pulp chambers and root canal are extremely large, giving the appearance ofthin dentinshells. See picture #4 in booklet

Treatment is full crown coverage for cosmetic purposes. Note: These teeth should not be used

as abutments, because the roots are prone to fractue under stress.

Mesiodens is the most common sup€rnumerary tooth, appearing singly or in pairs as a

small tooth with a cone-shaped crown and a short root between the maxillary central

incisors; it may be erupted, impacted or even inverted.

\Iesiodens appear situated in the maxilla near the midline and almost always posterior to

the normal central incisors. Many of them, therefore, are bypassed by the permanent

incisors $'hich are permitted to erupt into their normal position in the arch.

Remember:. Fusion: is a developmental union of two or more teeth in which the dentin and one

other dental tissue are united /rn ay be the root). See picture #6 in bookl€t. Concrescence: is a condition in which only the cementum of two or more teeth be-

comes united. See picture #7 in booklet. Dens in dente /a/so called dens invaginatus)r means "tooth within a tooth"' it iscaused by a deep invagination ofthe enamel organ during formation, most likely found

associated with a maxillary lateral incisor. See picture #5 in booklet

Page 5: Oral Pathology

. Molars

.lncisors

. Premolars

. Canines

4Cop)riglt @ 201l'2012 - D€nral Decks

\-,An enamel defect resulting from the incomplete

formstion of the enamel matrix is eall€d:

. Enamel pearls

. Enamel hypocalcifi cation

. Enamel hypoplasia

. Regional odontodysplasia

Coplright O 201 I -20 12 - Denral Decfts

Page 6: Oral Pathology

Hypercementosis is often confined to the apical half of the root but, in some instances,

may involve the entire root. In large majority of instances, it affecls vital teeth, is not as-

sociated with any one particular systemic disease and may be regarded as a dental anom-

aly. It rnay be seen when a tooth has lost its antagonist or when there is chronicinflammation of the tooth. The premolars are most frequently involved. Next in fre-quency are the first and second molars.

Hypercementosis produces no significant clinical signs or symptoms indicative of itspresence. It is seen radiographically as a bulbous enlargement that has surrounding it acontinuous and unbroken periodontal membrane space and a normal lamina dura. See

picture below.

Note: There is a form of hlpercementosis which is a common feature in Paget's disease

that involves thejaws. In addition to Paget's disease, hypercementosis is associated withsupraeruption, apical periodontal infection, occlusal trauma, toxic thyroid goiter,

acromegaly, and pituitary gigantism.

Example of g€neral hypercementosis: Maxillary premolars

Enamel hypoplasia is a developmcntal def'ect in which the cnamel ofthe tccth is hard in contcxt but

thin and dcficient in amount. It results from incomplete formation ofthe enamel matrix with a

deficiency in the cementing substance. Enamel hypoplasia a1l'ects both the deciduous and pcnna-

nent teeth. It is usually due to illness or injury during tooth formation or due to a genetic disorder

Note: Thc gcnetic lbrms ofcnamel hypoplasia arc generally considcred to be types ofamelogen-

esis imperfecta.

The clinical appearance ofenamel hypoplasia includes: l) the lack ofcontact between teeth, 2) the

rapid breakdown ofocclusal surfaces, 3) a yellowish-brorvn stain that appears whcre the dentin is

exposed. Note: lfonly one permancnt tooth is affected, it is usually caused by physical damagc or

periapical infections to the primary tooth that this pernanent tooth replaced. This is known as

"Turner's h-vpoplasia."

Remember: Enarncl hypocalcification is a hereditary dental dcfcct in which thc enamel is soft and

undercalcificd in context yct normal in quantity. It is caused by the dcfective maturation ofamcloblasts ftrere r.r a defect in the mineraliz tion ofthe fonned matrir). The teeth are chalky in

consistency, the surfaces wear down rapidly and a yellow to brown stain appears as the underlying

dentin is exposcd. This condition affects both the deciduous and permanent teeth as well.See picture #9 in bookl€t

\ote: Regional odontodysplasia involves the hard tissues that are derived from both epithelial(enarnel) an<l mesenchymal (dentin and cemenhnt) components of thc tooth-fonning apparatus.

Thc tccth in a region or quadrant of the maxilla or mandible are affccted to thc extent that they

exhibit short roots, open apical foramina' and enlarged pulp chambers' The thinness and poor

mineralization quality ofthe enamel and dentin layers havc given rise to thl; term "ghost teeth."The permanent teeth are aflected more than the primary teeth , and the maxillary anteriol teeth are

affccted more than other teeth. The cause is unknown, although nume.ous etiologic factors have

been suggested /1.e., trauma, nutritional defcie cies, inlbcliotts). Because ofthe poor quality oftheaffected teeth, their rcmoval is usually indicated.

Page 7: Oral Pathology

ORAL PATHOLOGY Abn of Tth

Match the following alterations in tooth rnorphology on the l€ftwith the correct diagram depicting the alteration on the right.

. Fusion

. Twinning

. Concresence

. Gemination

6

Copyng}r i) l0lt :0ll Dental De.ts

ORALPATHOLOGY Abn of Tth

All of the following statements concerning amelogenesis inperfectarre true EXCEPT one. Which one is the EXCEPTIOM

.It is an inherited condition which is transmitted as a dominant trait

. Because ofthe enamel malformation, the teeth ofindividuals with amelogenesis imper-fecta are often discolored, sensitive to temperature changes, and painful to brush

. It only affects the permanent teeth

. It causes the enamel of the teeth to be soft and thin

. The teeth appear yellow. because the dentin is visible through the thin enamel

. The teeth are easily damaged and susceptible to decay

7

CopyriShr C l0ll-:0l2 - Derral Decks

Page 8: Oral Pathology

Remember:. Gemination is a division of a single tooth germ by invagination. This results in the incomplete tbr-

mation oftwo tceth. Frcquently occurs in the incisor region. Fusion is an abnormally shaped tooth that may appcar as an extra wide crown, a normal crown rvrth

an extra root. or other combinations fesulting from the union of two adjacent tooth gcrms by dentin

during development.. Concresence is the union ofthe roots of two or more normal teeth caused by con'flucnce of their

cemental surfaces.. Trvinning is the completc division of a single tooth bud. The dividert teeth are secn as com-

pletelySepalatewithnoconnectiontoeachotherexcepteachtendstobemirrorimageoftheother. Ank:-losis is the fusion of the cementum or dentin to the surrounding alveolar bone after loss

oflhe intervcning periodontal membrane lt is associated \Pith hypodontia The most commonly anky-

losed tooth is the pfimary second molar; the pemanent second premolar is the tooth that fails to de-

\ elop and crupt. Clinically, percussion ofthe anlllosed tooth produces a drrll sound

\ote: There is a change in the continuity ofthe occlusal plane which is caused by the continued erup-

rion ofthe non-ankylosed teeth and the growth ofthe alveolar process

clinicallJ,, gemination and fusion can look similar. when counting these tceth, the "larger" tooth should

It cc)unred as a single tooth. ln gemination, the patient has a larger tooth but a normal numbel ofteeth

o\ erall. In fusion, the patienl has a larger tooth, but one lcss tooth than normal "

-{melogenesis imp erlecta (AI) is a relatively rare group ofinherited disorders characterized

bl abnormal enamel formation. The term amelogenesis imperfecta is reserved for hereditary

defects ofenamel that are not associated with defects in other parts ofthe body or other health

problems. lt is a hereditary ectod€rmal defect, unlike dentinogenesis imperfecta which is a

hereditary rnesodemral defect.

The AI enamel defects are highly variable and include abnomralities that are classified as hy-poplastic defect in amount ofenamel),hypomaturatron (defect in fnal grov'tlt and mqhrationol'euottel crystallites), andhypocalcified (defect in initial crystallite Jbnnation followed by de-

te. ti\ e gt'ovlh). The enamel in both the hypomaturation and hypocalcified Al rypes is not min-

eralized to the level of normal enamel and can be described as hypomineralized Al can be

intrerited as an x-linked, autosomal recessive (,4RJ, or autosomal dominant (AD) cor'ditiorr.

The color ofthe enamel ranges from white opaque to yellow to brown, it is reduced in volume

and pitled. Contact points between teeth are often open and occlusal surfaces and incisal edges

frequently are severely abraded. Although the enamel are soft and irregular, there is no in-crease jn caries mte. Note: Open bite is a common clinical finding.

The radiographic findings are fiequently distinctive and pathognomonic. when the enamel

is totally absent, the radiographic appearance makes the diagnosis obvious. When some enamel

is present, thin radiopaque coverings on the proximal surfaces ofthe teeth are noted. When the

anatonic crown forms are normal or nearly normal, the softness ofthe defective enamel may

not be easily distinguished from the dentin. ln all cases, howevel the dentin' pulp and ce-

mentum are unaffected by the disease process itself(arlr*e dentinogenesis inperkdq).See picture #8 and #10 in booklet

Exception: Amelogenesis imperfecta will only show pulp obliteration if there is advanced

abrasion with secondary dentin formation.

Other than cosmetic restoration, no treatment is necessary.

Page 9: Oral Pathology

. Erosion

. Abfraction

. Attrition

. Abrasion

. Dentinogenesis imperfecta

. Erythroblastosis fetalis

. Porphyria

. Fluorosis

. Diabetes mellitus

. Pulpal injury

. Intemal resorption

. Tetracyclines

8Cop).dght O 20ll-2012 - Denhl Decks

9Coplrighr O 201 l'2012 - Dental D€cks

Page 10: Oral Pathology

See picture #13 in booklet

Types ofabrasion:

l. Toothbrush abrasion: most olten results in V-shaped wedges at the cervical marginin the canine and premolar areas. lt is caused by the use ofa hard toothbrush and/ora horizontal brushing stroke and/or a gritty dentifrice.2. Occlusal abrasion: results in flattened cusps on all posterior teeth and wom incisaledges. lt results from the chewing or biting ofhard foods or objects and chewing tob-acco.

. Attrition is the wearing away of enamel and dentin due to the normal function or most

commonly, due to the excessive grinding or gdtting together oftecth by the patient lreJbrced

to a.r bruxism). The most noticeable effects ofattrition are polished facets, flat incisal edges.

discolored surfaces ofthe teeth and exposed dentin. Facets usually develop on the linguoin-cisal of the maxillary central incisors, the facioincisal of the mandibular canines and the

linguoincisal ofthe maxillary canines. See picture #14 in booklet

. Erosion is the loss of tooth structure from non-mechanical means. It can result fromdrinking acidic liquids or eating acidic foods. It is common in bulimic individuals as a re-

sult of regurgitated stomach acids. It affects smooth and occlusal surlaces.

See picture #12 in booklet

. Abfraction lesions are cervical erosive lesions that can not be attributed to any partic-ular cause: causing the enamel to "pop'' off starting at the base ofthe tooth and exposing

the gum Iine ofthe tooth to excessive wear See picture #11 in booklet

Cause Intrinsic Stain

Dentinogenesis imperfecta Translucent or opalesccnt hue; usually gray to bluish-brown

Erv1hroblastosis fetalis Bluish-black, greenish-bluc, tan or brown

Poryhyria Red or brownish

Fluorosis white opacities or light brown to brownish-black

Pulpal injury Starts pink and usually becomes orangc-brown to bluish-black

Intcrnal resorption Pinkish

Tetracvcline Varies from light yel)ow-orangc to dark gray-brown; dependingon the tlDe oftetracycline used and the duration oltherapy

H1 pomineralized enamel White to brown

E\trinsic stains can be caused by bacteria, iron, tobacco, foods, beverages, gingival hem-

orhage. restorative materials and medications.

Treatment:. lIost extrinsic stains can be removed with abrasives; reduce or eliminate cause ofstain. Intrinsic stains may need bleaching, esthetic restorations, or prosthetic rehabilitation

Page 11: Oral Pathology

ORAL PATHOLOGY Abn of Tth

What is th€ most probable diagnosis of the x-ray below?

10

Coptright (l 20l l-:0 l 2' Denral Decks

ORAL PATHOLOGY Abn of Tth

A patient with which typ€ of amelogenesis imperfecta will have teeththat demonstrate enamel that varies from thin and smooth to normal

thickness with grooves, furrows and/or pits?

. Type | (H.vpoplastic)

. Ty-pe II (H|pomqturation)

. Type III (Itvpocalcified)

11

CopyriShr C 20ll'101: - Denral Dccks

Page 12: Oral Pathology

Int€rnal resorption is an unusual form oftooth resorption that begins centally within thetooth. Resorption ofthc dentin ofthe pulpal walls may be seen as part ofan inflarnmatoryresponse to pulpal injury, or it may be seen in cases in which no apparent trigger can be

idcntificd. Most cases of intemal resorption present no early clinical symptoms. The first ev-idence of th€ lesion may bc thc appcarancc ofa pink-hued area on the crown of thc tooth,which reprcsents the hyperplastic, vascular pulp tissuc filling thc rcsorbed areas. Ifthe con-dition is discovered before perforation ofthe crown or root has occured, endodontic ther-apy may be canied out with thc cxpectation ofa pretty high success rate.

Idiopathic external resorptive l€sions arc charactcrized by thc invasion ofthe ccrvical rc-gion ofthe root by fibrovascular tissue which progressively resorbs dentin, enamcl and ce-mentum. Thc sourcc of the resorption is in thc attachmcnt apparatus. This is the key tounderstanding diagnosis and treatment. The dental pulp remains protected by an intact layerof dentin and predentin until late in the process. Since the source of the lesion is not in thepulp, endodontic treatment by itselfis ofno assistance in dealing with the sourcc ofthe prob-lcm.

Arrow points to external rootresorption of the maxillary

left c€ntral incisor

Crown size varies fromsmall to normal. small teelhnay Iack proximal con-lacts, color varies from nor-mal to opaque white-ycllow

Varies fiom thin and smooth tonormal rhickness with grooves,firrrows andlor pits

Enamel has normal toslightiy reduced contrast

Varies flom creamy opaqueto marked yello\r',/brown.surfacc ofteeth sofl andrough. dental sensitivityand open bite are common

Normal thickness with enamelthat ofien chips and ab8deseasily

Enamel has contrastsimilar to or less thanclcntrn. uncrupted crownshave normal morphologv

Opaque nhite to yellow-brow_n. soft rough enamelsurface. dental sensiti.i,ityand open bite are common.hea\,1 calculus formation is

Normal thickness with enamelthat ofien chips and abiadeseasily

Enamel has contrastsimilar to or less thandentln, unerupted crownshave normal morphology

See pictures #15, #16, and # l7 in booktet

Remember:

L Amelogenesis imperfecta is a hereditary ectodermal defect, unlike dentinogenesis imper_fecta which is a hereditary mesodermal defect.f. In all three types ofame)ogenesis imperfecta, the dentin, pulp and cementum are unaf-fected by the disease process itself /anlte dentinogenesis imperfecta).

Page 13: Oral Pathology

. Normal

. Somewhat smaller

. Extremely large

. Completely obliterated

. Red blood cells (Erythocytes)

. Platelets

. Plasma cells

. White blood cells (Leukocytes)

't2

Cop)right O 201l-2012 - Denral Decks

13

Coplri8hr O 201 | '2012 - D€nral Decks

Leukemi! is a group ofbone mrrrow diset$esinvolving an uncontrolled incr€ase ln:

Page 14: Oral Pathology

Dentin dysplasia is another autosomal dominant trait that affects dentin. AII teeth ofbothdentitions are affected. This condition has not been associated with any systemic con-nective tissue disorder. This is a rare condition that has been subdivided into type I orradicular type and a more rare type II or coronal type:

. Type | (radicular cl.,-.splasia): more common type. Both dentitions are normal in color and shape. The teeth are generally mobile, frequently abscess and can be lost prematurely. Teeth show greater resistance to caries than do normal teethRadiographic features:. Extremely short roots. Obliterated pulp chambers and root canals before eruption. Residual fragments of pulp tissue appear typically as horizontal lucencies( cnevrons). Periapical radiolucencies (granulomo,s or c.vst-r') around the defective roots

. Type Il (coronal dy.splasia). Color of primary teeth is opalescent. Color of permanent teeth is normal. Coronal pulps ofpermanent teeth are r"rsually enlarged ("thi,\tle lube") a J arclilled rvith globules of abnormal dentinRadiographic features:. Deciduous teeth are similar in appearance to type I, but permanent teeth exhibitenlarged pulp chambers ("thistle nbe") in appearance. .\bsence of neriaoical radiolucencie"

Leukemia is a lomr ofcancer that begins jn the blood-forming cells ofthe bone marrow (the soft,

innet part o.l'1he botret. Undcr nomal circumstances, the blood-fonning, or hematopoictic, cclls

of thc bone marrow make leukocytes to defend thc body against infectioLls organlsms such as

r inrses and bacteria. But il'sot'ne leukocytes are damaged and remain in an inmlature fbrm' they

becornc poor int'ection fighters that multiply excessivcly and do not dic olfas thcy should.

The leukcmic cells accumulate and lesscn the production of oxygcn-carrying red blood cclls /ery-

rlr r or r les/. blood-clo tting cells (plaletets), and normal lcukocytes. If untrcated, the surplus leukemic

cells or er$helm the bone marrow, enter the bloodstream. and eventually invadc other parts ofthe

btrdr. such as the lymph nodes, spleen, livcr, and central nervous syslern (brcin, "^Pfualcoldr' In this

* ar. the bchar ior of leukcmia is different than that of other canccrs, which usually bcgin in major

organs and ultimatcl), spread to thc bone marrow.

Thcrc are more than a dozen varictics ofleukenia, but thc follorving four types arc thc most com-

nlon:

L -\cute lymphocytic lcnkcrlta (ALL). most common type in children1. .\cute myelogenous le]rlemia (AML.): most malignant typej. Chronic lymphocytic lcukernia fczlr: least malignant type-1. Chronic myclogenous leukcmia (CML)t 2 distinct phases, invariably latal

lmportant:

. Leukemia is classified by thc dominant cell type and by the duration from onset to death

. Leukemia can modify thc inflammalory reaction

. Leukemia's affect on teeth/gingiva: Duc to a decrease in immune responsc, periodontal prob-

lenrs can be aggravated. Gingival enlargement can be a finding rvith leukemia Additionally,

deficits with platelets can bc scen intraorally with increased hemorrhaging,/bruising."

Page 15: Oral Pathology

BId Dis

A 48 year old female patient walks into your office. She states that she isdiagnosed with some disease which she can't remember the name of. Herphysician wants her to follow up with you, her dentist, regularly to watchout for cancer of the tongue and throat. She also has a bald tongue, and

states that her fingernails "look funny." What disease does she have?

. Aplastic anemia

. Plummer-Vinson syndrome

. Pernicious anemia

. Cushing's syndrome

14

Copvrighr ali 2011'201: - DenralDecks

ORAL PATHOLOGY Btd Dis

What disorder is a result of a genetic mutation causing thesubstitution of glutamic acid by a valine and results in

dental radiographs with enlarged marrow spaces?

. Cystic fibrosis

. Muscular dystrophy

. Polio

. Sickle-cell anemia

15

Copvrigl'r aC 201l'2012 ' DenlalDecks

Page 16: Oral Pathology

Plummer-Vinson syndrome characterized by an iron-deficiency anemia, atrophicchanges in the buccal, glossopharyngeal, and esophageal mucous metnbranes, koilonycha(spoon-shaped./inger rail.s), and dysphagia. The dysphagia is due to an esophageal stric-ture or web. SCC of the tongue and throat are complications. lt is most common in mid-

dle aged women, rarely in the male. The etiology unknown. Because ofthe predisposition

to the development ofcarcinoma ofthe oral mucous membranes, it is essential that the di-agnosis be established early so that treatment can be given ASAP This includes admin-

istration of iron, vitamin B complex and a high protein diet.

Aplastic anemia is a form of anemia in which the capacity of the bone marrow to gen-

erate red blood cells is defective. Two types:l. Primary: unknown cause, affects young adults. The signs and symptoms includepallor, weakness, malaise, dyspnea (dilfcul4, breathing), headache and vertigo. Oral

symptoms include spontaneous bleeding,bnstsing (petechlaeT and gingival infections.

It is usually fatal.2. Secondary: caused by exposure to toxic agents, such as radiation, chemicals ordrugs (fbr exanple, chloramphenicol). It can occur at any age. Symptoms are the same

as primary. Prognosis is good once you remove the cause.

*** Aplastic anemia is the most serious and life-threatening blood dyscrasia associated

s ith drus toxiciw.

Sickle-cell anemia is an inherited disease in which the red blood cells, normally disc-shaped, become crescent shaped, As a result, they function abnomally and cause smallblood clots. These clots give rise to recurrent painful episodes called "sickle cell paincrises." Sickle-cell anemia (also called sickle-cell disease) is the result ofthe production

ofabnormal hemoglobin (Hemoglobin S/ due to a genetic defect. It is carried as a traitby 10% of African Americans and 0.2olo have sickle-cell anemia. lt is more common in fe-males and usually clinically manifests itselfbefore the age of30. The typical signs ofane-mia are present. The patient is weak, short ofbreath and easily fatigued. Muscle and jointpatns are common.

Denlal radiographs ar€ oft€n of diagnostic value: marrow spaces are markedly en-

larged because ofthe loss ofmany trabeculae; the trabeculae, which are present, are oftenabnormally prominent. Occasionally, osteosclerotic areas are noted in the midst of large

radiolucent marow spaces. However, the lamina dura and the teeth are unaffected.See picture #18 in booklet

\ote: The gene defect is a known mutation ofa single nucleotlde (th),minelbr an adenine)

ofthe beta-globin gene, which results in glutamic acid to be substituted by valine. Sickle-cell anemia occurs when a person inherits two abnormal ger'es (one front each parent). lfa person inherits one abnormal gene for the disease, they have what is called sickle celltrait. The life span ofred blood cells is reduced from 120 to 20 days.

Page 17: Oral Pathology

Bld DisORAL PATHOLOGY

The category of pigmented (usually purple) lesions in the skin caused by€xtravasation ofblood from the capillaries is known as:

. Petechiae

. Ecchymosis

. Varicose veins

16

Copyrighr al 2011-2012 - Dental Decks

ORAL PATHOLOGY Bld Dis

All of the following statements are true EXCEPZ one.which one is the rxcEPZl0M

. Acute leukemias have a slow onset and progression

. Acute leukemias are characterized by the appearance of immature, abnormal cells inthe bone marrow and peripheral blood and frequently in the liver, spleen, lymphnodes, and other parenchymatous organs

. The clinical picture of acute leukemias are marked by the effects ol anemia, which is

usually severe (/ittigue, molaise), an absence of functioning granulocles (proneness

to inlbction and inflammation), and thrombocytopenia (hemorrlrugic diathesis)

. The spleen and liver usually are moderately enlarged, while enlarged lymph nodes

are seen mainly in acute lymphocytic leukemia. Fever and a very high ESR are

found

. Leukocyte counts vary greatly in the acute leukemias

17

Copyrighr /(, 2011'2011 Dental Decks

Page 18: Oral Pathology

Purpura spots (pinpoint spots) are purplish discolorations in the skin produced by smallbleeding vessels near the surlace ofthe skin. Purpura may also occur in the mucous mem-

branes (such as the lining oJ the mouth) and in the intemal organs. Purpura by itselfis onlya sign of other underlying causes of bleeding. Purpura may occur with either normalplatefet counts hrcnlhrombocftopenic purpuras) or decreased platelet counts (thrombo-q)topenic purpltra.rr. Platelets help maintain the integrity of the capillary lining and are

irnportant in the clotting process. Note: Large purpura spots (> 3 mn) are called ecchy-moses.

Major kinds of purpura:. Thrombocytopenic purpura (Werlhol's tlisease): a bleeding disorder characterized

by a deficiency in the number olplatelets. This resr.rlts in multiple bruises, petechiae,

and hemonhage into the tissues. Thrombotic thrombocytopenic purpura (TTP): a severe and frequently fatal formcharacterized by a low platelet count in the blood due to consumption of platelets bythrombosis in the terminal arterioles and capillaries ofmany organs

Oral manifestations of thrombocytopenic purpura:. Severe and profuse gingival hemorrhage. Petechiae occur commonly on the palate

Important: Tooth extractions are contraindicated due to the tendency for excessive

bleedin s.

*** This is false; acute leukemias have a rapid ons€t and progression.

-{cute leukemia is characterized by malignant proliferation olwhite blood cell precursorsI hle.\ts) in bone marrow and lymph tissue and their accumulation in peripheral blood,bone rnanow, and body tissues. Leukemic cells inhjbit normal bone marrow productionrrf en throcytes. platelets, and immune function.

Orher important features ofacute leukemia:. -A.brupt onset (fe11,noltlsl with sudden high fever, weakness, malaise, severe anemia,and -ueneralized lymphadenopathy; bone and joint pain common in children.. Principal organ involved: bone marrow (along witlt the spleen and liver). Petechiae and ecchymoses in skin and mucous membranes, hemorrhage from vari-ous sites: bacterial infections common.. Laboratory findings: leukocytosis 30,000-1000,000 per cu.mm. with immature formsr nn eloblasts and 1.,-mphoblasts) pred,ominating.. ln 7-i9i ofthe cases olacute lymphocytic leukemia, the lymphocytes are neither B norT-cells and are called "null cells. ". L ntreated patients die within six months; with intensive therapy (chemotherap,-,rutliation, and bone marrow transplanls) remissions lasting up to five years may beobtained; death is usually due to a hemorrhage (broin) or a superimposed bacterialinfection.

Page 19: Oral Pathology

. Thromboc)'topenic purpura

. Agranuloc).tosis

. Sickle-cell anemia

. Peutz-Jeghers syndrome

18

Copynghr @ 2011,2012, Dental Decks

The term used to describe a leukemia where leuk€mic cells rppear in the bloodbut there is no significant lncrease in the number of white blood cells is callod: .,

. Aleukemic leukemia

. Subleukemic leukemia

. Stem cell leukemia

't9

CopFight O 20ll-2012 - Dental Dects

Page 20: Oral Pathology

Agranulocytosis is an abnormal condition of the blood, characterized by a severereduction in the number ofgranulocytes (particL arly neutroprilr./. Note: It may alsobe caused by the antithyroid drugs fi.e., propylthiouracil, methimazole, qnd carbima-zole).

Clinical features:. Sudden onset of high feveq chills, jaundice, weakness and sore throat. Oral infection with rapid periodontal destruction. Oral ulcers and gingival bleeding

The most characteristic feature of this condition is the presence of infection, particu-larly in the oral cavity. The signs and symptoms develop very rapidly, usually within a

fev days. and death may occur soon afterward.

The oral lesions are an important phase ofthe clinical aspects ofagranulocytosis. They ap-

pear as necrotizing ulcerations of the oral mucosa, particularly the gingiva and palate.

These lesions appear as ragged necrotic ulcers covered by a gray membrane. One impor-ranr aspect is that there is littl€ or no apparent inflammatory cell infiltration around the

lesions. Histologically, this is pathognomonic of agranulocytosis.

\ote: Cl clic n€utropenia is an unusual form ofagranulocytosis. These patients typicallyerhibit severe gingivitis. The severe ulcerations usually seen in agranulocytosis usuallydLi not occlrr. See picture #19 in bookl€t

Other leukemia terms to know:

. ".A.leukemic" leukemia is a term used to describe a lorm of leukemia in which there

are leukemic cells present in the bone maffow, but the circulating white blood cells are

n€ither immature nor increased in number.

. Stem cell leukemia is a form of leukemia that is characterized by abnormal cells that

are poorly differentiated but are considered to be precursors of lymphoblasts, myelo-blasts. or monoblasts. Note: these cells are too immature to classify.

. A "leukemoid" reaction is a term used to describe a marked increase in the numberof circulating granulocytes. This condition is seen in a variety of disorders inch.rding

cluonic infections and neoolasms.

Page 21: Oral Pathology

. Denuded gingiva

. Glossitis

. Edematous buccal mucosa

. Severe gingivitis

20

Cop),righr O 201 l-2012 - Dental D€cks

In your ofllce, you see a 6 month old child whose lirst teeth are eruptlng andwhose mother iJ conc€rned about the color. The mandibular incboB do show

a brownlsh-blue hue, You are golng to ask the mother about whichof the following conditions during her pregnancy:

. Sickle-cell anemia

. Erythroblastosis fetalis

. Patent ductus arteriosus

. Low-weight preterm birth

21

CopFiglr O 20ll-2012 - Dental Decks

Page 22: Oral Pathology

Pernicious anemia is a relatively common, chronic, progressive, megaloblastic anemia.It is caused by the lack of secretion of the intrinsic factor in normal gastric juice. Thislactor is necessary lor adequate absorption of vitamin 81.,, which is necessary for thematuration of erythrocytes. As a result, they produce fewer erythrocytes than normal.The onset of pemicious anemia usually is insidious and vague. As the condition pro-gresses, there will be a sore, painful ton g.ue (atrophic glossilr.r/, angular cheilitis, a tinglingnumbness of the extremities. difficulty swallowing (d,,-sphagia), painful swallowing(od,vnophagia). See picture #20 in booklet

A Schilling 24-hour urine test is done to evaluate whether vitamin B 1.' is being absorbed

by the body and is most commonly used to evaluate patients for pemicious anemia.

Remember: Thalassemia major and minor are hernolytic an€mias that result from a ge-

netic defect. Both are characterized bv a low level of ervthrocvtes and abnomal hemo-globin.

Oral manifestat ions of thalassemia:. Oral mucosa may exhibit the characteristic anemic pallor. Flaring of the rnaxillary anterior teeth with malocclusion

The fetus'blood is Rh-positive because the father passed along an Rh-positive trait. rvhich

is a dominant trait. The mother responds to the incompatible blood by producing antibod-

ies against it. Thesc antibodies cross thc placenta into the fetus'circulation. whcrc thcy aftach to and destroy thc fctus' red blood cells, leading to anenria

- this is called

er.r throblastosis fetalis.

\ote: It can also rcsult from blood typc incompatibilities. For cxample, the mothcr may

har e ty'pc O blood and thc fetus has type A or B blood.

The most common form oferythroblastosis fctalis is called is ABO incompatibility, whichcan ran in its severity. The less common form is called Rh incompatibility, which more

ottcn causes a very scvere anemia in the baby. The severity olthis condition can vary widcly.

In some instanccs, the baby has no symptoms of the disease. In othcr cases, it can lcad to

death of thc baby beforc or shortly after birth. lt can be treatcd in utero by intrauterine trans-

t usion.

\\'hcn thc child is bom, signs may include an enlarged liver or spleen, generalizcd cdcma,

laundice. and anemia. After birth, depending on the severity, a transfusion usually nccds to

be performed. The most severc form of this disease, Rh incompatibility, can bc prcvcntcd

if thc mothcr takes a medicinc called Rhogam at certain times during and aftcr prcgnancy.

Oral manifestations of crythroblastosis fetalis:. Teeth appear to havc a grccn, blue or brown hue due to thc deposition ofblood pigment

in the enamel and dentin.. Enamel hypoplasia may occur. If it does, it affects thc incisal edges of the anteriorteeth and the middle portion ofthe deciduous cuspid and the first molar crown.

Page 23: Oral Pathology

They have a rapid onset and progressron

They have a shorter, more devastating clinical course than the acute leukemias

They are characterized by proliferations of lymphoid or hematopoietic cells that aremore mature than those ofthe acute leukemias

They constitute 7 5o/o of all leukemias

22Copyright O 201l-2012 - Dental Deck

. Acute lymphocytic leukemia (ALL)

. Acute myeloid leukemia (AML)

. Chronic lymphocytic letkemia (CLL)

. Acute monoblastic

23Coplright @ 20ll-2012, D€nral Decks

Page 24: Oral Pathology

Important: Chronic leukemias have a slower onset and progression. They also have a

longer, less devastating clinical course than the acute leukemias, and they constitute

about 507o of all leukemias.

Other important clinical features of chronic leukemias:

.Insidious onset witl't weakness and weight loss: disease may be detected during

exanination for some other condition (e.g., anemia, unexplained hemorrhages, or

rccw're nt intrqctab le inlbctio t1 ). Organ involvement similar to acute type: massive splenomegaly is characteristic

of chronic myelogenous leukemia; lymph node enlargement is main pathologic find-

ing in lymphocytic tyPe. Petechiae and ecchymoses' recurrent hemorhages, bactedal infections -

lymph-

ocytic anemia may be complicated by autoimmune hemolytic anemia. Laboratory findings: leukocylosis above 100,000 per cu. mm. with matur€ forms(gronulocltes ancl lymphocytes) predominating; Philad€lphia chromosome and low

levels of leukocyte alkaline phosphatase are corunon in chronic myeloid l€ukemia(c.\'tL). \'ledian survival time for patients with chronic myelogenous leukemia /CMI/ is lour

l ears rvith death <lue to hemorrhage or infection; chronic lymphocytic leukemia (CIl)runs a variable course; older patients may survive years even without treatment'

The peak age for ALL is around four years old, and it is the form of actte leukemia that

is the most responsive to therapy. Cunent therapies for ALL include chemotherapy withlbllo\1.up radiation, and possible bone-marrow transplant after particularly high dose

chemotherapy treatment or in cases ofrecurrence or ilnon-responsive to other treatments.

*** Acute myeloid leukemia (AML) and chronic lymphocytic leukemia (CLL) arc themost common types in adults. See picture #22 in booklet

Allhough the exact cause ofmost leukemias remains unklown, increasing evidence sug-

gests a combination ofcontributing factors. These factors include: familial tendency, con-genital disorders (Dov,n syndrome, or the presence of Philadelphia chromosome -chronic myeloid leukemia), viruses (e.g., HTLVI, herpes-like viral particle,s lnve been

c ulared.fion potients ancl leukemic patients have high ontibod), titer to the Epstein-Barrlirr-st. ionizing radiation and the exposure to the chemical benzene and cytotoxins such

as alk) lating agents.

Important: Oral lesions are most likely to be observed in myelogenous leukemia. These

oral lesions may be the initial manifestation ofthe disease. The oral lesions include gin-givitis, gingival hemorrhage, generalized gingival hyperplasia, petechiae, ecchymoses.

and ulcerations. See picture #21 in booklet

Page 25: Oral Pathology

Bld Dis

You have a new patient in your dental oflice who has just movedfrom Denver. He srys his doctor told him that he has some disease

caused by living at a high altitude. When conducting an intraoral €xam,you lind that his tongue is a deep purple and his gingiva bleed easily.

What disease is a likely cause ofthese findings?

. Polycythemia Vera

. Hemophilia B

. Thallesemia Major

. Porphyria

24Coplrighl e 20ll':012 ' Denral Decks

ORAL PATHOLOGY Bld Dis

The translocation from chromosome 22 to chromosome 9is a finding of which leukemia?

. Acute lymphocyttc Ierkemia (ALL)

. Chronic myeloid leukemia (CMZJ

. Acute myeloid leukerma (AML)

. Chronic lymphocytic leukemia (CLL)

25Copright ae 20ll l0ll DenklDeck!

Page 26: Oral Pathology

Polycythemia is the condition oftoo many red blood cells in the circulation. The bloodcan be too thick to pass easily through the small blood vessels ofthe body. This in tumleads to clot formation and blockage ofthe small vessels which can lead to a stroke

There are two types ofpolycythemia:I . Primary polycythemia (also called polycythemia vera or e4lhemia) occurs when

excess erythrocytes are produced as a result ofturnorous abnormalities. This occurs inthe tissues that produce blood cells. Usually accompanied by leukocytosis. Splenomeg-

aly, as a result ofvascular congestion, is seen in 75% ofpatients.2. Secondary polycythemia is an increase in the total number of erytluocytes due to

another condition. For example. chronic tissue hypoxia of advanced pulmonary dis-ease, high altitude fo.r ler's disease) or the secretion oferythropoietins by certain tumors.

Oral manifestations of polycythemia:. Oral mucous membranes (especialb tlrc gingiva and tongue) appear deep purplish-

red.. The gingiva are very swollen and bleed very easily.. Submucosal petechiae (purplish spots), ecchymoses (same as petechiae, but bigger)

and hematomas are common.

\ote: Erl-thromelalgia is a rare syndrome of paroxysmal vasodilation with buming

oain. incrcascd skin tcmpcrature, and redncss ofthc feet and, less often, thc hands.

\lmost 90% of patients with chronic myeloid (mteloctttic, n|'elogenous, granuloc,-tic)leukemia have the philadelphia chromosome, an abnormality in which the long am ofchromosome 22 is translocated. usually to chromosome 9. Radiation and carcinogenicchemicals may induce this chromosomal abnormaliry

C\IL is characterized by the abnormal overgrowth ol granulocytic precursorsrn^elctblqsts and promvelocvtes) in bone marrow, peripheral blood and body tissues.C\tL is most common in young and middle-aged adults and is slightly more common inmen than in women: it is rare in children.

The npical symptoms of CML include:. spongy bleeding gums. t'atigue. fer er. \\ eight loss. moderate splenomegaly. joint and bone pain. repeated infections

\ote: Acute myeloid leukemia (AML) rs a malignant disease of the bone marrow ins hich hematopoietic precursors are arrested in an early stage of development. AML isdistinguished ftom other related blood disorders by the presence of greater than 30%blasts in the blood and/or bone rnarrow. These blasts /m|eloblasts) contain Luer rods intherr cvtoDlasm.

Page 27: Oral Pathology

Which of the following is { troublesome librobLsticneoplssm that is locally sggrer$ive rnd infiltrrtive?

. Peripheral fibroma

. Traumatic neuroma

. Nodular fasciitis

. Fibromatosis

. Angiomas

. Lymphangiomas

. Schwanaomas

. Fibrosarcomas

26

Copynghr Q 20ll-2012 - D€ntal Deck

27Coplright O 201l-2012 - Denlal Decks

\While in the OR on a general surgery rotation, a 3 month old is brought in withalarge (20cm) flaid-lilled mass on her neck. The diagnosis is r cystic hygroma

(hygroma coli).This lesion is under whlch umbrella oflesions, which also' contdns enlarged tissue on the postcrior rnd lat€ral border of the tongoe?

Page 28: Oral Pathology

Tumor Etiolog) Clinical Charictenstics Trcatmenl and Prognosis

Most common sit€ ov€I mentllforamen m edentulous moufts;nodule or swelling, wbich maybe painful to digilal pressure

[xcision wjth snall proximalpodion ofinvohed neNei

Encapsulated mass lhal prese.tsa5 an a*mptomatic lump. Tbelongue ls the most commonlocation.

Consenati!e e{cisioni

siy it is denved from 1. Solitlry nelrollbromr -

asympromaric ndule, occurs ontongue. buccal mucosa and

2. iltultiple lesions as pad oflhesFdrome neurofi bromatosis

L Soli(ary: surSical excision2. Neurofibromatosis:removal is impractical. wrtchlbr hiSh rate ofmalignant

P.es€ms as {irrn mass; €xhibitsrapid gowlh; pain and

Su.gical excivont rccuncncc

Tbe mandible and condguoussoi lissues are most fr€qu€nllyinvolved intrno.ally. LocaLlyaggressive and infi ltrarive.

Aggressive sursical approach isrecommended. Rccurrence's

\ traumatic ncuroma is a lesion causcd by lrauma to the periphcral ncnc. In thc oral catity, thc injury rnay be nr

rhr rirmr olrrauma fiom u surgical procedure such as a toolh cxlraclion, from a local ancslhctic injcclion or fionl an

.r..ideni. It is usually a vcry small nodulc 1/€.\'r /,{utt 0.5 .r in didnrcter).In thc oral cavity it is most commonly sccn

.! rIc mentat foramen. Il is firm, movable ard wcll cncapsulatcd. ll is painfut when pslpaled. Prcssurc applicd to

rhr naurona clicits a resporsc oftcn dcscribcd as an "electric shock".

Hi!tologic features:. \bundant ncnc lissuc a'rd collagcnous fibrous tissuc in haphazard arangcnrcri. Chronic lillammalory ccll infilirate may be prcscnt. Sch\ ann cclls rri1l bc prcscnt

Ly mphangiomas are benign bamartomas of lymphatic channels that develop early in life$ ith no sex predilection. They may occur on the skin or mucous membrane. ln addition tothe tongue, they occur commonly on the lips and labial mucosa.

Clinical features:. Raised, diffuse. bubbly nodules or vesicles. Range in color from clear to pink. dark red, brown or black

' .As)'mptomatic. Soft. fhrctuant. \ aries in size. Usuall.v painless

Histologic features: four types of lymphangiomas:

. Ll mphangioma simplex (capillory lymphangiono) - composed of small, thin-walledlvmphatics. Calernous lymphangioma - comprised ofdilated lymphatic vessels with surround-

ing adventitia. cl stic lymphangiom? (cyslic hlgroma or hygroma col, - consisting ofhuge, macro-

scopic lymphatic spaces with surounding fibrovascular tissues and smooth muscle.Benign lymphangioendothelioma facquired progressive lltnphangiont) lymphaticchannels appear to be dissecting through dense collagenous bundles

Important: Lymphangiomas do not undergo malignant change. Some lymphangiomas, es-

pecially congenital types, regress spontaneously during childhood. Aspiration is manda-tory before surgical excision ola lyn.rphangioma to prevent complications associated withthe similar-appearing hemangioma

Page 29: Oral Pathology

. Lipoma

. Pyogenic granuloma

. Epulis granulomatosum

. Peripheral fibroma

28Coplright O 20ll-201: - Dental Decks

A pati€nt presents to your clinic with multiple exophytic masses coveringthe buccal mucosa, tongue rnd lips. A biopsy reveals that these are mucosrlneuromas. The most importrnt rerson this patient should be referred to a

physician is bectuse ofthe risk of related:

. Squamous cell carcinoma ofthe tongue

. Pituitary hyperplasia

. Medullary carcinoma ofthe thyroid

. Sipple's syndrome

29

Coplaiglit O 201l-?012, Denral Deckr

Page 30: Oral Pathology

slate during pregnancy

l0 days of

Remember:L Thc peripheral fibroma is a rcactive hyperplaslic mass that occurs in the Singiva and may be dc-rivcd from connective tissue ofthc submucosa or the PDL. It presents as well-demarcatcd lbcal mass

\\ ith eithcr a sessile or pedunculated base. Ii is similar in qolor to the suffounding connective tissue.

It may be ulccratcd. The treatment for a pcriphcral fibroma is local excision. Recurrence is rare'

\ote: Other variant forms ofthe peripheral fibroma includc:. The peripheral odontogenic fibroma: which is gingival mass composed ofa well-vascularized,flbrous connective tissuc. Thc distinguishing featlre ofthis variant is thc presencc ofstrands ofon-dorogenic epithelium, oftcn abundant, throughout the connective tissue. It is usrlally non-ulcerated.. Thc peripher.l ossifying fibroma: is a gingival mass in which calcified islands. presumed to be

bonc. are seen. The bone is found qithin a non-encapsulated prolifcration of plump benign fibrob-lasts. The surface is often ulccrated. See picture #85 in booklet

:. Focal fibrous hyperplasia is hlperplasia oforal mucosa. It is also callcd traumatic fibroma, irri-larion fibroma. and hyperplastic scar. It is a rcactivc lcsion caused osually by chronic trauma to oralmucous membranes. The giant cell libroma is a focal fibrous hlT'erplasia in which connective tissue

cells. many ofwhich are multinucleated, assume a stcllatc shapc.

The multipfe endocrin€ neoplasia syndromes (olso called MEN Syndrome./ have been

classified into three distinct syndromes, each is inherited as an autosomal dominant trait:

. \ten I: consists of tumors or hyperplasia of the pituitary parathyroids, adrenal

cortex, and ofthe pancreatic islets.. \len Il (also c'alled Sipple's Syndrome ard sabn?e lr: is characterized by parathy-

roid hyperplasia or adenoma, but no tumors to the pancreas. However, in addition, these

patients have pheochromocltomas ofthe adrenal medulla and medullary carcinoma ofthe thyroid gland.. \Ien III (elso kno$n os sub\tpe IIB): rs characterrzed by mucocutaneous neuro-mas, pheochron'rocytomas ofthe adrenal medulla and medullary carcinoma ofthe thy-roid gland.

Important: MEN I and II are related to MEN lll in that patients with types I and II syn-

dromes have neoplasms ofvarious endocrine organs, but do not have the oral manifesta-

rions ofmucosal neuromas. These oral lesions are most common on the lips, tongue, and

buccal mucosa.

\ote: The most important aspect of this syndrome is the medullary carcinoma of the

thvroid because of its ability to metastasize and cause death. Therefore, the detection

of the rnucosal neuromas may alert the clinician for early diagnosis and treatment.

Page 31: Oral Pathology

. Traumatic neuroma

. Neurilemmoma fsc hwannoma)

. Neurofibroma

. Nodular fasciitis

. Fibromatosis

. Neurolemoma

. Neurofibroma

. Neuroma

. Fibroma

30

Coplrigh O 20ll-2012 - Dental Decks

31

CopFight O 201l-2012 - Dental Deck

Page 32: Oral Pathology

Neurofibromas may appear as solitary Iesions or as muitiple lesions as part ofthc syndrome neurofi-bromatosis 6,()// ne. klinghdusen:\ disease of skin). lr rs a benign ncoplasm, howcver the etiology ofsolitary neurofibromas is unknown. Most researchers believe the cell oforigin is the Schu'ann cell; oth-ers believe dre pe neLlral fibroblasr is rcsponsiblc- For the solitary neurofibroma thc tongue. buccal mu-cosa, and vestibule are the oral regions most commonly affected.

Tumor Etiology Clinical Chrracteristics Treatm€ntand Prognosis

Traumaticpcnpheralnene

Most common sile over mentalforamen in edentulous mouths;nodule or swelling, which maybe painful lo digital pressure

Excision wilh small proxirnalponion of involved nen,e;recurTence uncommon

Neurilemmoma BcniSn neoplasm ofSchwann cells

Encapsulated mass that Fesentsas an aslmptomatic lump. Thetongue is the mosl commotlocalion.

Conservalive excision;

Neurofibrona Somc invcstigalorssay il is derived liomthe Schwann cell;olhers say it is de-rived l-.om

fibroblasts

Two formsl1. Solltary neurofibroma -asymplomaiic nodule, occurs ontongue, bsccal mtrcosa andvestibule2. Multiple lesions as pan ofthes)mdrome Deurcfi brohalosis

L Solitary: surgical excision2. Neurofibromatosislremoval is impractical. wrlchfor high rate ofmalignanttransformalion

\odular fasciilislpserdosarconatous

Reacti!e lesion;proliteration offibroblasts

P.esents as firm mass; exhibitsr8pid growth; pain andlendemess common

Surgical excision; recurrence

Benign fibrousproliferation offibroblasts

The mandible and conliguoussofi tissues are most liequenllyinvolved intraorally. Locallyaggressive and inliltrative.

Aggressive surgical approach isrecommended. Recurrence is

\bn Reckfinghausen's disease (neut'oJibromatosis) is an autosomal dominant disease

associated rvith loss ofa tumor suppressor gene (NF/ or NF2l. lt is characterized by mul-tiple neurofibromas of the oral cavity and on the skin, and pigmentations of the skinr Ccrfi-au-luit .rpor.s/, iris freckling (Lisch spots), and axillary freckling Crou'e )s sign.).

\ote: The presence ofsix or more caf6 au lait macules greater than 1.5 cm in diameter

is generally regarded as being indicative of this disease until proven otherwise

Radiographic features:. \\ ell-demarcated unilocular or multilocular radiolucency. Possible root disturbance. Possible jarv enlargement

Treatment:. Surgical excision but may be impractical due to the number of lesions. Best 1eft untreated because multiple recuffences may be associated with malignanttransformation to neurogenic sarcoma

\ote: The importance of the lesions is the high risk (5%' to l5%o.1 of mtlignanl trans-formation

Remember: The single neurofibroma presents at any age, it comnronly appears as a ses-

sile, firm, pink nodule that commonly occurs on the tongue, buccal mucosa and

lestibule. It is rerloved by surgical excision and rarely recur.

Page 33: Oral Pathology

Con Tis Les

A 5S-year-old patient com€s into your oflice for routine dental work.You se€ that he has a tooth fracture (due to decay) of tooth #31,

A smooth, firm, asymptomatic lesion is noted on the lateralborder ofthe tongue adjacent to the sharp enarnel oftooth #31.The patient states that the lesion has been there for years and is

annoying because sometimes he will bite it accidenta[y.Name this most frequently encountered intraoral benign

neoplasm of conn€ctive tissue origin,

. A leiomyosarcoma

. A traumatic fibroma

. A leiomyorna

. A rhabdomyoma

32

Copynghr C 2011-l0ll - Dental Decks

ORAL PATHOLOGY Con Tis Les

Scleroderma is a systemic disease that alfects many organ systems.The symptoms result from in{lammation and progressive tissue fibrosis and

occlusion ofthe microvasculature by excessive production and deposition of:

. Types II and IV collagens

. Types I and lll collagens

. Elastin and reticulin

. All of the above

Page 34: Oral Pathology

Traumatic fibroma, also know as initation fibroma, focal fibrous hyperplasia, and h)?erplastic scar, is

a reactive lesion caused usually by chronic trauma to oral mucorls mcmbranes. Ovcrcxubcrant fibrous

connective tissue repair results in a clinically evident submucosal mass.

*"* Otlrcr lcvels of nracromolecules found in the conncctivc tissue fe.g., glytosantinoglycans,:a,1u'(tn. lihronecti ) are also incrcascd.

scleroderma is an uncommon. autoimmune disease. It all'ects the connective tissues $hich sur-:rrund thcjoinls, blood vcsscls and intcmal organs bcneath the af}'ected area ofskin. Womcn arc af-i.tcd thrcc to ibur tinres morc oftcn than men. The diseasc usually starts between thc ages of25llnJ :0 It onl-'- oqcasionally bcgins in children or in the clderly. Although there is no curc. propcrlrearnlent and care can makc it possible for people rvith sclerodenna to lead full, productive livcs.\ote: \1a) occur concomitantly with other autoimmune diseases. Such as lupus erythemalosus,

rheunaroid arthritis. dermatomyositis, and Sjogren's sundrome. \ote: Thesc changcs in thc con-n.-iti\c trssue all'ect the fingers, trunk, face (producing u "purse-string" moil/i], and the nlorerr(]\]]'nal parts of the cxtrenlitieS.

Oral radiographs ol'a patient with scleroderma would show an abnormal rvidening of the peri-..Jrrnral ligament. This space is created by a thickening ofthe periodontal-membranc as a rcsult of:n increase in size and number ofcollagen fibers. The enlarged space is almost uniform io width,.rlrounds the cntirc root of the toolh and makes the tooth appear as if it is being cxlrudcd rapidly:ilrir its socket. See picture #31 in booklet Note: Other oral radiographic l'eatures may include]rlar.ral resorption ofthe angle ofthe ramus ofthe mandible or complete resorption 01-thc condylcs:nd or coronoid process ofthe mandible. Remember: The abnormal widening ofthc periodontal-:rembrane spacc is also a radiographic finding in osteosarcomas.

Thcrc arc tlvo major types ofscleroderma:

I Localized scleroderma; In this condition changes only occur in isolatcd areas of the skinand the tissues beneath it. [t is rclativcly mild and docs not aft'cct intemal organs. Cutaneouschanges includc induration and rigidity. atrophy, and telangiectasias.

L Systemic scleroderma: With this condition changes rnay occur in thc skin and also in a num-ber of intemal organs. These might include blood vessels, joints, the digcstivc syst"em (esopha-

gus, stomach and borlel/, and occasionally the lungs, hean, kidneys and muscles. Changes in the

connective tissue may affect the function ofany ofthese organs.

Painless, broad-bas€d swellinglhar is lighter rhan surroundingii$ue; fi€quenlly fotmd onbuccal mucosa, laleral borderoftongxe, snd tower lip

Reaclive lesion caused

tmuma to oral mucous

Wid€ surgical exc's'oni rc'Orat l€iomyomas arc rare;present as slow-growlng,asymptomatic subrnucosaloasses, usualiy in the longue,bard palate, or buc€al mucosa

Wide surgical excisioniOral leionyosarcomas have

been report€d in all ag€ groups

and most intraoral regions.

ExcisioniRare bul have a pr€dilectionfor the soli tissues olthe h€ada..d neck Aoor of the nouth,solt palate. totryue, and buccaln!.a.ra). Presents as anasymptomatic. well-d€fi ned

Combinition ofsurgery.radirtion, ard chemothempy.melaslasis not common

Rare in head and neck; mostcommonly affecled oml silesar€ th€ longu€ and soft palf,te.

Presents as a mpidly $owingmass, which, ifrhere isjawinvolvem€nt, may caus€ pa'n.

Page 35: Oral Pathology

Con Tis LesORALPATHOLOGY

A newborn baby girl was delivered via caesarian section due airway patencyconcerns. During ultrasound, there was th€ discovery of a tumor oftheoral cavity. Upon delivery, the pink, compr€ssible tumor of the anteriormaxilla was deemed to be a congenital epulis ofthe newborn. This lesion

is composed ofcells that are identical to those ofthe:

. A traumatlc neuroma

. A schwannoma

. A granular cell myoblastoma

. A lipoma

34

CopyriShr q 201 l-201 I - Dental Decks

ORAL PATHOLOGY lnfl Jaw Les

An emergency patient walks into your o{fice with swelling ofthe leftsubmandibular space. He says his lower left molar recently 'broke down"and has been very painful especially when something cold hits it or whcn

he chews down on it. What is the most likely etiology of this swelling.

. Orthodontics

. Trauma

. Infection of the pulp ofthe tooth

. Periodontal disease

Copyrighr C 20ll'l0ll - Dental Decks

Page 36: Oral Pathology

The congenital epufis of the newborn (also called congenital gingival granular celltumor).lt rsually appears on the gingiva (usually anteriof ofnewborns. It presents as a

noninflamed, pedunculated or broad-based mass. The maxillary gingiva is more often

involved than the mandibular gingiva, and ferrales are affected more than males. The

treatment is surgical excision with little possibility ofrecurence.See pictur€ #24 in booklet

The granular cell myoblastoma is a rare neoplasm of unknown etiology. Most re-

searchers believe its origin is from the Schrvrum cell. It presents as an uninflamed. asymp-

tomatic mass less than 2 cm in diameter. The most common location in the head and neck

region is the tongue. It may affect any age group and females seem to be affected more

than men. See picture #25 in booklet

Important: Both of these lesions are identical histologically. They both contain granu-lar cells, however, the congenital epulis of the newborn does not exhibit overlayingpseudoepitheliomatous hyperplasia. The pseudoepitheliomatous hyperplasia of the

or erlying epithelium is frequently seen in the granular cell myoblastoma.

*** This infection follows the carious involvement ofthe tooth. The cellular debris and/or

rnfection rvhich caused the tooth pulp to die, slowly filters out of the tip of the root and

produces an inflanmatory reaction around the root tip

\ote: ,\ periapical abscess can also occur after traumatic injury to a tooth, which results

in necroiis of the pulp, and in cases ol irritation of the periapical tissues, either by me-

chanical manipulation or by the application of chemicals in endodontic procedures'

Clinical features:. If acute, presents as an abscess: See picture #26 in booklet

Tooth is extremely painful to percussion

- May feel slightly extruded from its socket

- Tooth will exhibit mobilitY. If chronic, presents as a granuloma or cyst. There are usually no clinical features of

svmptoms

Radiographic features:. If acut€, only a slight thickening ofthe periodontal membrane is noticeable

.If chronic (granuloma or q,st), there will usually be a radiolucent area at the apex ofthe involved tooth. See picture #27 in booklet

Treatment: Establish drainage either by opening the pulp chamber or extracting the

tooth.

Note: If a periapical abscess is not treated, it can lead to serious complications such as

osteomvelitis. cellulitis and bacteremia.

Page 37: Oral Pathology

A [ealthy patient comes into your ollice for an inltial exam. On the full mouthseries of radiographs you see a radiopaque lesion perlaplcal to tooth #19. Tooth#19 has a deep amalgam restoration with r€curretrt decay underneath. You csn

Nee the entire outline ofthe mesial root oftooth #19 - the lesion seems to stemfrorn the tooth. What is the most llkely diagnosis of the lesion?

N.

. Focal sclerosing osteomyelitis

. Cementoblastoma

. Cementoma

. Fibrocementoma

CopyriShr O 201 1,2012 - Dental D€cks

may be psrt ofthe etiologic picture, althoughbeen the most fr€quently cited.

. Bacteroides and campylobacter

. Clostridia and corynebacterium

. Staphylococci ard streptococci

. Enterococci and lactobacilli

37Copyrighr O 201 l-2012 - Dental Decks

Page 38: Oral Pathology

Focal sclerosing osteomyelitis is a relatively common phenomenon that is believed to repre-sent a focal bony reaction to a low-gnde inflammatory stimulus. [t is usually seen at the apexof a tooth in rvhich there has been a long standing pulpitis. Note: Synonyms for focal scle-rosing osteomyelitis include bony scar, condensing osteitis, and sclerotic bone. The term focalperiapical osteopetrosis has also been used to descdbe the lesions associated with normalcaries-free teeth.

Focal sclerosing osteomyelitis may be found at any age but is typically discovered in youngadults. Patients are usually asymptomatic, and most lesions are discovered on routine radi-ographic examination. A majority are found at the apices ofmandibular first molars. The pe-riapical x-ray demonstrates the pathognomonic, well-circumscribed radiopaque mass ofsclerotic bone surrounding and extending below the apex ofone or both roots. The entire rootoutline is almost always visible, an important litature in distinguishing it from the benign ce-mentoblastoma, which radiographically, it may resemble. The tooth with this lesion may be

treated or exftacted, since the pulp is infected and the infection has spread past the immediatepenapical area. The sclerosing bone constituting the osteomyelitis is not attached to the tooth.and remains after the tooth is treated or removed.

DifTuse sclerosing osteomyelitis represents an inflammatory reaction in the mandible or max-illa. believed to be in response to a microorganism oflow virulence. lmportant in the etiologyand progression ofdiffuse sclerosing osteomyelitis is chronic periodontal disease, which ap-pears to provide a portal ofentry lorbacte a. The condition tends to occur most ftequently inmiddle-aged black females. The disease is typified by a protracted chronic course with acuteexacerbations of pain, swelling, and occasional drainage. Radiographically, this process is

dld'ure. 9*pically affecting a large part ofthejaw The lesion is ill-defined. Treatment consistst idetermining and addressing the cause. Antibiotics are the mainstay of trcatment. Low-dose.onicosteroids have also been used with some success. Hyperbaric oxygen therapy may prove

:.r be a r aluable adjunct.

Acutc inflammation ofthc bonc and bone manou'ofthe mandible and maxilla rcsults most fiequentlyfrom €rtension of a periapical abscess. The second most common cause of acute osteomyelitis is

phlsical injury as secn u,ith fractlte or surgery. Most cases of acute osteomyelitis arc infcctious.

Staphr-lococci and streptococci are the most frequcntly cited.

Pain is the primary feature of this inflammatory process. Fever, painful lymphadenopathy, leukocyto-

sis. and other signs and symptoms ofacutc infcction arc also commonly found. Paresthesia ofthe lotlerlip is occasionally seen with mandibular involvcmcnt. Important: Unless the inflammatow process has

bccn prcscnt for more than I wcck, radiographic cvidcnce ofacutc ostcomyelitis is usualll_ not present.\\'ith timc. diffuse radiolucent changes bcgin to appcar. Treatment includes antibiotics and drainage.

Chronic osteomyelitis (chronic osteitis) fiay bc one ofthc scquclac ofacule osteomychlis /ei er ult-I eatetl o, in.tdequateb lrealed), or rt may represent a long-tenr, lorv-grade inflammatory reaction that

nevcr $ ent rhrough a significant or clinically noticeable acute phase. Most investigators believe that

bacleria /e.g., staph ococci, \treptococci, bacleroiles. acli on\rer'/ are responsible for thc vast major-

iI ol chronic osteornyelitis cases. The mandible, especially the molar area, is much more lrequentlyatl'ected rhan js thc rnaxilla. Pain is usually present, s\relling ofthe jaw is a comrnonly encountered sign;

loose te€th and sinus tracts are less frequently seen. Anesthesia is very uncommon. Radiographically,chronic osteomyclitis appears primarily as a radiolucent lesion that may show focal zones ofopacifica-rion. The luccnt pattem is often described as "moth-eaten" because ofits mottled radiographic appear-

ance. Treatment includes antibiotics and scqucstrectomy.

Gar16's osteomyelitis or chronic osteomyelitis with proliferative periosteitis, is cssentially a subtype ofchronic osteomyclitis in which there is, additionally. a prominent periostcal inflammatory reaction. Itmosi olien results from a periapical abscess of a mandibular molar in a child. The child characteris-

ricali.v presents with an asymptomatic bony hard swelling with normal-appcaring overlying skin and

mucosa. Radiographicallli thc lesion appears centrally as a mottled, predominantly lucent lesion ina pattem consistcnt uith chronic osteomyelitis. The featrre that provides thc distinctive difference is theperiosteal reaction. This appears as an expanded cortex, oftcn uith concent c or parallcl opaque lay-els. Trabeculae perpendicular to the "onion skin" layers may also be present. Treatment includes toothextraction and antibiotics.

Page 39: Oral Pathology

. First to third week

. Fourth to sixth week

. Eighth to tenth week

. twelfth to fourteenth week

. Relatively normal torso and long arms and legs

. Short torso and long arms and legs

. Long torso and long arms with short legs

. Relatively normal torso and short axms ajld legs

38Coplright O 20ll-2012 - Dental Decks

39Coplrighr O 201l-2012 - Denral Decks

Page 40: Oral Pathology

Cleft palate occurs in the eighth to tenth week of embryonic life. Isolated clefts olthepalate are more common in females. It is characterized by a fissure in the midline ofthepalate, resulting from the failure ofthe two sides to fuse during embryonic development.The most severe handicap imposed by cleft palate is an impaired mechanism preventingnormal speech and swallowing. Note: It effects approximately I in 2000 births.

Cleft lip results when the medial nasal proc€ss fails to fuse with the lateral portions ofthe maxillary process olthe first branchial arch. Fusion normally occurs during the sixthand sev€nth weeks of embryonic development. The maxillary lip is most commonly af-fected. It may be bilateral (20a/o) or lunilateral (80%). Clefts ofthe lip are more frequentin males. Lip clefi involvement is more frequent on the left sid€ than the right.Note: It effects approximately I in 1000 births, but varies with race.

L Speech problems associated with both ofthe above are usually the result of\ot€< the inability ofthe soft palate to close airflow into the nasal area.

2. It is not unusual for teeth especially the lateral incisor to be missing in the

cleft area.

3. More than 250 syndromes have been identified that may be associated withcleft lip and palate, or cleft palate alone.

4. Lip pits are rare anomalies that can occur in the upper lip, lower lip, or the

oral commissure. Although lip pits may be seen near the oral commrssure ormidline upper lip, most occur on the Iower lip and are associated with van der\lbude syndrome. This syndrome is an autosomal dominant condition with80% to 90% penetrance consisting of lower lip pits and cleft lip and/or cleftDalate.

.\chondroplasia is the most common type of dwarfism. The upper an.ns and thighs aremore shortened than the forearms and lower legs. Generally, the head is large, the fore-head is prorrinent, the nose has a saddle-like appearance and the mandible exhibits prog-n al h ism.

Potential problems in children with achondroplasia include overcrowding of the teeth,speech problems (articulatio ), and frequent ear infectior:Ls (otitis media). Dental maloc-clusion is treated \\. ith orthodontics. All children with achondroplasia should be evalu-ared b\ a speech therapist by two years of age because ofpossible problerrs with thede\eiopment ofclear speech. Articulation problems may be caused by onhodontic prob-Iems. Due to the abnormal shape of the eustachian tube in an individual with achon-droplasia, they are very prone to ear infections (otitis media).

\ote: The teeth are of nomal size but there is limited space within the maxillary andmandibular arches for them to erupt into which causes overcrowding and subsequent mal-o!'cluS ron.

Page 41: Oral Pathology

.oI

. Marfar's syndrome

. Ehlers Danlos

. Cystic Fibrosis

. Acid phosphatase

. Vitamin K

. Alkaline phosphatase

. Phosphorus

40Coplrighl O 2011,2012 - Ltental Decks

41

Copyrighl c' 2011,2012 - DentatD€cks

Hypophosphrt.sia is a genetic metabolic disorder ofbone mineralizstion caused by a deficiency in:

Page 42: Oral Pathology

The characteristic features olosteogenesis imperfecta (OI) vary greatly from person toperson and not all characteristics are evident in each case. The chief clinical character-istic ofosteogenesis imperfecta is the extreme fragility and porosity ofthe bones, with aproneness to fracture. Other features include: pale blue sclera, deafness due to otoscle-rosis, abnormal teeth, loosejoints and low muscle tone. a triangular face, and a tendencytoward spinal curyature.

Clinically the teeth have:. Crowns that are bulbous: with a cervical constriction. Pulps that are obliterated, either partially or completely. Roots that are narrower and shorter*** The deciduous teeth are more s€ver€lv affected that the permanent dentition

T! pes of ost€ogenesis imperfecta:. Type l: most common and mildest fonn. Type ll: most severe lonn resulting in multiple fractures just from birth process. Type lll: most severe form beyond the perinatal period. Type IV: mild to moderately severe bone fragility

There is no known cure for osteogenesis imperfecta. Treatment is directed toward pre-

\ enting or controlling the symptoms.

\ote: The cause of osteogenesis imperfecta is believed to be due to a genetic defect that

causes imperfectly-formed, or an inadequate amount ofcollagen. The affected person has

either less collagen than normal, or a poorer quality of collagen than normal, leading to\r eak bones that fracture easilY.

Hr pophosphatasia is an inherited metabolic (chemical) bone disease that results from lowlerels ofan enzyme called alkaline phosphatase. This enzyme is essential to the calcifica-

tion ofbone tissue. The severity of hypophosphatasia is remarkably variable from patient to

patient. Some patients have blue sclera that resembles ost€og€n€sis imperfecta. There may

be delbrmity ofthe arms, legs and chest. Frequent bouts ofpneumonia can occur as well as

rccurrent tiactures.

J h pes of h) pophosphatasia:. \eonatal: severe manifestations, respiratory failure, marked hypocalcification of the

skeletal structures. Infantile: h-vpercalcemia, premature loss of deciduous teeth, skeletal malformations,

failure to grorv. Childhood: short stature. frontal bossing, usually normal calcium and phosphate levels. Odontoh! pophosphatasia: children and adults rvho have only dental problems

Important: The premature loss of teeth in children and adults is usually characteristic.

These teeth also exhibit hypocalcification. Radiographically, the teeth display enlarged

pulp chambers and pulp canals, deficient root development as well as alveolar bone loss.

Rememtrer: Patient's with Paget's disease also have high levels of serum alkaline phos-

phatase.

Page 43: Oral Pathology

. Gigantism

. Acromegaly

. Achondroplasia

. Dwarfism

42Coplrighi C 201 l'2012 - Denral Decks

A new 6 year old pediatric patient walks into your operatory with hls mother,You initirl physical assessment not€s ! prominent foreh""d

""d ;;;;;;;;;.

The prtietrt initirlly seems to have no eyebrows but you lrter rerlize thst thehair is just very fine and sptrse. When you shake her hand and she smiles,

you llso notice that she is n.lssing teeth and the ones she has are cone

shaped. Whst fu her most likely systemic condition?

. Piene Robin syndrome

. Ectodermal dysplasia

. Cleidocranial dysplasia

. Peutz-Jeghers syndrome

. Osteopetrosis

43Coptrighr O 201 l-2012 - D€ntal Decks

Page 44: Oral Pathology

In over 90% of acromegaly patients, the overproduction ol GH is caused by a benigntumor of the pituitary gland, called an adenoma. Whether or not the epiphyses of thelong bones have lused with the shaft is the main detenninant of whether gigantism oracromegaly will occur when there is oversecretion of growth hormone by the pituitarygland.

Remember:. Gigantism: tumor prior to adolescence (non-fusion o.f epip\,ses). Acromegaly: tumor after adolescence (fusion of epiphyes)

Oral manifestations ofacromegaly and gigantisrn include: enlarged tongue, mandibularprograthism, spacing of the teeth rvhich are usually tipped to the buccal or lingual side,

oNing to enlargement ofthe tongue. Roots may be lorger than normal. See picture #33in booklet.

\ote: Dn arfism (pttuitary dvtar/i) is characterized by arrested growth. Frequently thesepeople have limbs and features not properly proportioned or formed. It is caused by un-dersecretion ofgfowth homone. Oral manifestations include: eruption rate and the shed-

ding ofthe teeth are delayed, clinical crowns appear smaller as do the roots of'the teeth,rhe dental arch as a whole is smaller causing malocclusion and the mandible is under-der eloped.

Ectodermrl dvsplasia is an x-rinked recessive condition charactedzed by abnormar deveropmenr ofthe.i1n 3nd associated struct]lfes lhdir noi!!, and teeth, ond s$eat gldnd.rl. It involves all structures which are::nr ed tiom rhe €ctodcrm. It affects mares more than females. common clinical findings include hv-pothrichosis /./e./?d.re in hait,).

^nhidrosis ho s\\,eat gland.r, leoding to h(,at iuh )l..r,La. c/lanodontia t,r

oligodontia konplere or partiar absenk of teeth). depresscd bridgc of nose, Iack of salivary grandsl.,l the child appears much oldcr than rvhat he or shc is. There is no treatment lor the disease. hoNever;::rrures can be fabricated for these patients. Keep in mind that they will need to be replaced periodicallv:-r r..!rmmodate the palient's jaw growth. Scc picturcs #28 and #29 in booklet

Clcidocranial d]splasia is an autosomal dominant condition ofbony developmenr characterized by hy-poplasia or aplasia ofthe clavicles' cranial bossing, ocular hypertelorism, and dental abnonnalilies which::rrlrJe retaincd primar) teeth, malaligned leeth, the presence of multiple supcrnumerarv teeth andunerupted reeth- lmportant: The dentition itself, as obsen,ed by radiographs alone, often suggests the di-:5o.rs. Sec picturc #30 in booklet

Pierre Robin svndrome is an inherited disorder that presents the fblowintl in the neonate: severe mi-crognsthia and mfndit ular hypoplasia. se\ ere glossoptosis //),,, ariot rlt,fLttenent ofthe tongtrc), andhigh-arched or cleft pal:tte. This condition is characterized by respiratory problems.

Peutz-.Icgh€rs syndrome /PJSI is a genetic condition rnarked by hyperpigmentation (r"rfltrg ofthe lips.rnd \ometlmes other parts ofthe face, hands, and feet followed by the development ofbenign polyps calledarn'anomas throughout the intestines but primarily in lhe small intestine. See picture #35 in bookletosteopetrosis /a&o.a lled Albers-Se honberg disease or narble bone./iredr., is an uncommon bone con-Jiiron thal mav be inherited as an autosomal dominant /&rr--reno .!, or recessive trait 6drle seri.n/s./. The.r3racteristic feature ofosteopetrosis is an abscnce ofphysiorogic bonc resorption owing to rcduced os-(eoclastic activity' The lack ofbone resorption results in accumulation ofbone mass and manifesls itself:. ikeletal disturbances. including bone cavity occlusion. decreased hematopoietic activity, and growth re-1:rrdation. Bone pain is the mosl liequent symptom. Blindness and deafness from sclerosis ofostia. aneminriom sclerosis ofbone marroq and osteomyeliris due to diminished vascularity are aiso seen. Dental find-ings include delayed enrprion. congenitally absent teeth. unerupted and marormed teeth, and enamer hy-ooDldsla,

Page 45: Oral Pathology

ORAL PATHOLOGY M&GJawDis

An old patient presents to your oflice with ill-fitting dentures. Radiographs re-veal hypercementosis on roots and the patient is speaking embarrassingly loudly

to you that she is more conscious ofhis baldness now that his hat doesn't fitanymore." These signs all point to what possible diagnosis?

. Paget's disease

. Osteosarcoma

. Fibrous dysplasia

. Albers-Schoenberg

CopyriShr C 2011 :0ll Denral Decks

ORAL PATHOLOGY M & GJawDis

. Aneurysmal bone cyst

. Central Giant Cell Granuloma

. Tumor of hyperparathyroidism

. Cherubism

45Coplrighr ie 20ll-l0l: Denlal Decks

You are consulting on a pathological case for a fellow dentist. The biopsy ofthe lesion sho\fls multinucleated giant cells and perivascular collagen cuffing.After asking about the clinical signs, your colleague mentions that the youngpatient seems always to be (staring offinto spac€. and that she has.puffy

cheeks." The most likely diagnosis of this case is:

Page 46: Oral Pathology

Paget's discase ofbonc is a chronic, slouly progressive condition ofunknown ctiology(it appears

toie lanitiatl. The most frecluent sitcs ofinvolvemenl include thc spine' l'elnur' cranium' pelvis'

and stemum. Tbe bone may become dcnse, but liagilc, because of cxcessive breakdou'n and fbr-

mation ofbonc. The disease is morc conmon in males and affccts adults ovcr thc age of 50. Thc

signs and symptoms inclucle pain in thc aflectcd arca, defomlily of tlre bone in thc irffected area'

su"sceptibility to fractures in lire affected arca' and headache and hearing loss ifthe affccted arca

is the skull. Note: Thesc symptoms dcvelop slowly'

When the jarvs are affected:. Patient who rvears a maxillary denture cotnplains that thcy do not fit aS thc n-Iaxilla progres-

sively cnlarges. Ultimatelyl the alveolar ridge wiclens. r,irh a rclative flatrening ofthe palatal vault

. When tecth are prescnt, incrcased spacing as wcll as loosening is notcd

. Hypcrcemcntosis of tooth roots, loss of lamina dura, oblilcralion of the PDL space and re-

sorption ofthe roots

]licroscopically, ostcoblasts and multinuclcated osleoclasts arc lbund in abundance As the lcsion

adr ances. dense bone wlth numeious reversal or growth lines are sccn' giving the tissue a mosaic

parrem.

Other important features of this disease include:

. Palicnts may also give a history ofprogrcssively increasing sizc ofhats or nc'$ dentttrcs being

made at progiessively more frequent intervals. Note: This is due to bony changes'

. Boncs are warm to touch due to incrcased vascularity

.\-raysofthesku|landthejawsdcmonstlatethctypical''cotton-wooll.appearance

. Lab iests: Drastically increased serum alkaline phosphatase. Scrum phosphale and calcium

are normal. Urinary calciun and hydroxyproline are increased'

. Treatment: Recently. thc use ofcalcitonin and bisphosphonates as parathofmone antagonlsls

ha\ e bcen eilective in suppressing bone resorption and dcposition lt is seldom latal

*** Paqet's tlisease ofbone is also called Osteitis Deformans' See picture #32 in booklet

Cherubism is a benign, autosomal dominant condition ofthe maxilla and mandible, usu-

all-,- found in children by 5 years ofage (it olfects males 2:i). The vast majority ofcases

occur in the mandible. The bony expansion is most frequently bilateral, although unilareral involvement has been reported. The clinical appearance may vary from a barely dis-

cemible posterior swelling ofa singlejaw to marked anterior and posterior expansion ofboth jau s. resulting in masticatory, speech, and swallowing difficulties. Intraoralh', ahard. non-tender swelling can be palpated in the affected area. There are no associated

sl sterric manifestations. The deciduous dentition may be spontaneously shed prematurely,

beginning as early as three years of age. There is often delayed eruption of the penna-

nenr dentition uhich is olten defective with the absence ofnumerous teeth and displace-

mcnt of those present.

Radiographicalll', the lesions characteristically appear as multiple, well-deltned, multi-locular radiolucencies ofthejaw. The borders are distinct and divided by bony trabecu-

lae. \ote: An occlusal radiograph of the maxilla may give a "soap bubble-like" picturerr rrh m.rrillrq anrrum obliteration.

Histologicalll, the lesions bear a close resemblance to those seen in central giant cellgranulomas. There are nlrmerous fibroblasts and rnultinucleated giant cells with promi-

nent nuclei. A distinctive feature is eosinophilic perivascular cufling of collagen sur-

rounding small capillaries throughout the lesion. Although this is not always present,

perirascular collagen cuffing is regarded as pathognomonic for cherubism.

The treatment is cautious waiting as there is spontaneous regression of the tumors at

around 25 - 30 years ol age. With increase in age and size of the patient. the defotmityproduced is less noticeable.

Page 47: Oral Pathology

. Vitarnin A

. Vitamin C

. Vitamin D

. Vitamin K

. Toxic nodular goiter

. Graves'disease

. Hashimoto's disease

. Addison's disease

45Coprighr O 201 l'2012 - Dental Decks

17Copyright @ 201 t-2012 - De al D4ks

Page 48: Oral Pathology

Hypoparathyroidism is a rare disorder associated with insufficient production ofparathy-roid hormone, the inability to make a usable form ofparathy'roid hormone, or the inabil-ity ofkidneys and bones to respond to parathyroid hormone production.

Hypoparathyroidism can result from congenital disorders, iatrogenic causes (e.g., drugs,

rcmoval ol the porathyroid glands during thyroid or parath!-roid sto'qery, radiation), rn-filtration ofthe parathyroid glands (e.g., metqstatic cqrcinoma. Il/ilson disease. sarcoid),suppression of parathyroid function, HIV/AIDS, or idiopathic mechanisms.

Hypocalcemia is the most important consequence of hypoparathyroidism. Symptoms

occur when ionized calcium level drops to less than 2.5-3 mg/100 mL. The clinical man-ifestation is tetany. A positive Chvostek's sign (nitching oJ the.fdcial muscles v'hen

rapped on the facial nen e near the parotid gland) is characteristic ofhypoparathyroidism.

lmportant: The dental manifestations of hypoparathyroidism (i.e., delayed eruption,euantel ht,poplasia and blunted root apices) may be prevented by early treatment withvitamin D.

Remember: The term muscular dystrophy refers to a group ofgenetic diseases marked bythe progressive weakness and degeneration olthe skeletal, or voluntary muscles, which con-

fiol molement. Oral manifestations include an increase in dental disease iforal hygiene is

neglected. weakness in the muscles of mastication leading to decreased maxillary bitingibrce and a higher-incidence of mouth breathing and open bite.

The rerm hr..perthyroidism refers to any condition in which there is too much thyroid hormone

tln rorr'r, in the body. This most commonly results fron a generalized overactivity ofthe entire

ihlroid gland. a condition also known as diffuse toxic goiter or Graves'disease. Altematively,one or nrore nodules or lumps in the thyroid may become overactive, a condition known as toxic

nodular goiter or Plummer's disease, The primary role ofthyroxin is to stimulate cellular me-

rabolism. grorvth and differentiation of all tissues. In excess, it leads to high basal metabolism,

tilrgue. \! eight loss, excitability. elevated temperature and generalized osteoporosis. Oral man-

ifestations are not too remarkable, but ifthe disturbance begins in the early years of life, the

F.en'rature eruption of the teeth and the prernature loss of the deciduous dentition are common

dndings.

T! pes of hi perthJ-roidisml. (Note: Besic symptoms of htperth,'-/oidisn dre present as h,ell as

lJdi|io aI \nptonts).. Gra\'es'disease: is the most common form, occurs most frequently in women under 50

- goiter knlarged thyoid which moy catrse a bulge in the netklerophthalmos is common

- thickened skin over the shin area. Plummer's disease (toxi. nodular goiler; afibcts both genders usually over 50

Erophthalmos is rareOfien uni-system, may present with only cardiac disease

lmportant: Thyroiditis causes temporary hypcrthyroidism, usually followcd with hypoth-

l roidism. Thyroiditis is an inflammation ofthe thyroid gland.

Therc are three main types of thyroiditis:. Hashimoto's thyroiditis. Subacute granulomatous thyroiditis. Silent lymphocytic thyroiditis

Page 49: Oral Pathology

. Dwarfism

. Mlredema

. Cretinism

. Acromegaly

48Coprighr C 201 l-2012 - Dental Decks

The clinical features ofthe primiry form of which disease isclassically described as "stones, bones, groans, and moans?"

. Paget's disease

. Hypophosphatasia

. Hyperparalhyroidism

. Hyperthyroidism

49Coplright O 20ll-2012 - Denral Decks

Page 50: Oral Pathology

Hypothyroidism refers to a condition in which the amount ofthyroid hormone in the bodyis below normal. This is the most common form ofthyroid function abnormality, and isfar more common than hyperrhyroidism. This condition is considerably more comn.ron inwomen than in men. The most common cause of h)?othyroidism is Hashimoto's thy-roiditis. The second most common cause is the treatment of hyperthyroidism. Hypothy-roidism is characterized by pufiiness of the face and eyelids and swelling of the tongueand larynx. The skin becomes dry and rough and the hair becomes sparse. The individualhas a low basal-metabolic rate and a low body temperature. The affected individualsalso have poor muscle tone, low strength and get tired very easily. Mentally they are verysluggish. The treatment of hypothyroidism is straightforward and consists of adminis-tering thyroid hormone (t hyt oxin).

Se\€re hypothyroidism in a child is called cretinism. Due to a lack of thyroid hormone,there is a retardation of growth and an abnormal development of bones. Mental retardationis caused by the improper development ofthe CNS. If this condition is recognized early, itcan be markedly improved with the use of thyroid hormones. Note: Extreme hypothy-roidism in adults is called myxedema.

\ote: Dental findings in a child with hypothyroidism include an underdeyelopedmandible with an overdeveloped maxilla, enlarged tongue which may lead to maloc-clusion. delayed eruption ofteeth and deciduous teeth being retained longer.

H) perparathyroidism (von Recklinghausen's disease of bone) is a tnctabolic disorder in which the

parath.lroid glands produce too nruch pamthyroid honnonc. Too much parathyroid horrnone causes

loo much calcium to be released from bon€. It may be caused by a f'unctioning parathyroid tunoror conlpensatory parathyroid hyperplasia due to renal lailure, malabsorption, or vitamin D defi-crency. Thcrc is a ferralc prcdilection and it affccts middlc agcd adults. Thc symptoms include:Ioss ol'appetite, increasing thirst, lrequent urination, Jethargy and fatigue, muscle weakness, jointpain and constipation. Important: pathologic fraettre (due lo the marked rcsorplion of borc)mal bc thc first symptom ofthe disorder Intraorally, thcrc is diffuse bonc loss causing malocclu-

sron and shifting ofthc teeth.

The chie I radiographic finding is the appcarance ofwell-defined cystic radiolucencies ofthejaw,*hich mav be unilocular or nultilocular Partial loss ofthe lamina dura is seen around thc tccth.

See picture #34 in booklet

lmportant: Histologically, multinucleated giant cells are scattered within a dclicate fibrocellular

'iroma. Accumulalions ofhcmosiderin and extravasated red blood cells also rc present. As a result, thc

ussue-i appear rcddish-brown, accounting for the term "brown tumor." These lesions arc miuroscop-

rcall) idcntical to qentral giant cell granulomas.

Thc discasc spectrum ofprimary hyperparathyroidism rangcs from asynptomatic cases ldiag-nosetl fion routine senon calcium determiralrotrs/, to severe cases oflcthargy and occasional coma.

Earl) synrptoms include faligue. weakness, nausca, anorexia, polyuria, thirst, depression, and con-

stipation. Frequcntly, bone pain and headaches are present. There are several clinical f'caturcs ass-

ociated primary hyperparathyroidism, classically described as "stones, bones, groans, and moans."

Lcsions ofthe kidneys, skclctal system, CI tract, and nervous systcm are responsible lor this syn-

drome complex. The rcnal component includes the prcsenc€ ol'renal calculi.

Management of primary hyperparathyroidism is aimed at eliminating the parathyroid pathology.

Surgery is the treatmcnt ofchoice.

Page 51: Oral Pathology

. Vitamin A

. \4tamin D

. Vitamin C

. Mtamin E

50Coplrighr @ 2011,2012 - D€ntal Decks

. Difficulty with mastication and swallowing

. Higher incidence ofperiodontal disease and caries

. Attrition ofthe teeth

. Multilocular radiolucencies of the jaws

Coprrishr @ 201l-2012, Denral Decks

Page 52: Oral Pathology

Osteomalacia involves softening of the bones caused by a deficiency of vitamin D orproblems with the metabolism ofthis vitamin. This softening ofthe bones occurs because

the bones contain osteoid tissue which has lailed to calcily due to the lack of Vitamin D.Note: Osteomalacia may occur as a complication ofsteatorrhea secondary to chronic pan-

creatrtls.

Clinical findings:. Osteopenia. Bone softening/deformity: hourglass thorax, bowing of long bones. Increased fractures. biconcave vertebral bodies. Mottled skull

All bones are affected, specifically their epiphyseal growth plates. Osteomalacia appears

to be more common in women. This condition mav be asvmDtomatic until fracture occurs.

Blood tests may show:. Lou levels of vitamin D. \la) also show low calcium and phosphorus levels. .\lkaline phosphatase levels can be high

Rickets is osteomalacia in children. It causes skeletal deformities. It is usually accompa-nied bl listlessness, initability and generalized muscular weakness. A child with ricketsmay hare borvlegs and develop a pigeon breast and a protruding stomach. The teeth in a

child l ith rickets are affected as follows: delayed eruption, malocclusion and develop-mental abnormalities ofthe dentin and enamel alone with a hi{:her caries rate.

Cerebral palsy is a term used to describe a group ofdisorders affecting body movementand muscle coordination. It is due to an insult to or anomaly ofthe brain's motor controlcenters. This damage interferes with messages from the brain to the body, and from thebod\ to the brain. The effects vary widely from individual to individual. Cerebral palsyrs characterized primarily by spastic paralysis or impaiment of control or coordination!r\ er \ oluntary muscles and is often accorrpanied by mental retardation, seizures and dis-!rrders of Yision and communication.

\ote: No intraoral anomalies are unique to persons with cerebral palsy. However, severalconditions /r/rose /isted on the ltont ofthe card) are more comnton or more severe than inthe normal population.

Remember: Down syndrome is a congenital defect caused by a chromosomal abnor-rnaliy.- (trisomy 2 l). It is marked by various degrees of mental retardation and character-istic phvsical features such as a short, flattened skull, slanting eyes, a thickened tonguer.tissured), broad hands and feet and other anomalies. Other oral rnanifestations includemandibular prognathism. increased incidence ofperiodontal disease, delayed eruption ofteeth. higher incidence of congenitally missing teeth, malocclusion, and enamel dyspla-s ia.

Page 53: Oral Pathology

. Sjdgren's syndrome

. Cystic fibrosis

. Cerebral palsy

. Down syndrome

. Hepatitis A

. Hepatitis B

. Hepatitis C

52Cop,righr O 201 1,2012 - Dental D€cks

CoP)rght O 201l-2012 - Denlal Decks

Page 54: Oral Pathology

*** Important: There is a significantly reduced caries rate in patients with cystic fi-brosis. This is probably the result of alterations in saliva and the long{erm use of an-

tibiotics.

Cystic fibrosis is a congenital metabolic disorder that causes the exocrine glands (vhichare glands thqt seqete.fluids into a duct) to produce abnormal s€cretions, resulting in

several symptoms, the most important of which affect the digestive tract and the lungs. Insome glands, such as the pancreas and those in the intestines, the secretions are thick orsolid (an ext essivef;, viscous mucous/ and may block the gland completely. The mucous-producing glands in the airways ofthe lungs produce abnormal secretions that clog the air-rvays and allow bacteria to multiply. The sweat glands secrete fluids that have a highsodium and chloride content. Note: The staining olthe teeth is most likely due to the lactthat patients with cystic fibrosis are usually subjected to large amounts of tetracyclinesduring childhood.

Si mptoms of CF include:. Poor growth despite good appetite. Malabsorption and foul, bulky stools: steatorrhea. Chronic bronchitis (COPD) with cough. Recurrent pneumonia: respiratory infections. Clubbing offingers and toes. Banel-chested appearance

l4tlLl-l{epatitis A is a highly contagious infectious disease involving the liver It is usually trans-

rnined by dre fecal-oral route. However, it may also be transmitted parenterally. tlepati-ris -{ usually rcsults liom ingestion ofcontaminated food. milk. or water. Many outbreaks

of tl'ris tvpe are traced to ingestion ofseafood from polluted water. It is caused by an RNAenteroYirus. [t most often occurs in young adults. The initial symptoms (bver malaise,

,tbdoninal pain. anorexia, jaundice.)of Hepatitis A appear after an incubation period of-1-6 l eeks.

Darnage to the liver cells. also results in increased serum levels of enzymes, such as

transaminases, nonnally active in liver cells. The detection ofincreased serum levels ofrhese enzlrnes is used in diagnosing this disease. In most cases ofHepatitis A, the infec-rion is selt'-limiting and recovery occurs within 4 months.

Oral complications: The only oral complication associated with hepatitis is the poten-

tial for abnormal bleeding in cases ofsignificant liver damage. Ifsurgery is required, itis advisable to check with the patient's physician prior to scheduling the sLrrgery.

I . The presence of surface antigen (A or B) in a patient's serum indicates that\otei the patient is potentially infectious for Hepatitis (carrier state).

...,.:-,,,a:. 2. Hepatitis viruses are very heat-resistant (morc so than the AIDS virus).

3. Autoclaving properly will kill these viruses.

Page 55: Oral Pathology

. Hepatitis A

. Hepatitis C

. Hepatitis D

. Hepatitis E

54

Coplright O 201 l -20 12 - Deotal Decks

Atl of the following aid in wound healing Excepr one.

Which one is the EXCEPTIOM

. Highly vascular areas (i.e., tongue)

. Hyperthermia

. Younger age

. Cortisone

Copyrighr O 20ll-2012, Dental Decks

Page 56: Oral Pathology

The Hepatitis B virus (HB V) rs a double-stranded DNA virus with worldrvide distribution,transmitted by parenteral and sexual contact. Risk factors include multiple sexual partners,

intravenous drug abuse, and rcccipt ofblood products. Its incubation pcriod is 40 to 100

days, and it can be recovered from all body fluids, most importantly, blood, brcast milk, andamniotic fluid. The signs and symptoms are similar to hepatitis A (fever abdominal pain,nausea, etc.) but there is a longer incubation p€riod f6-8 ueefs). The symptoms arc slorverin devcloping but are ofa longer duration. Most patients rccover fully, however, some de-velop chronic liver disease.

Previously, viral hepatitis that was not caused by the type A or type B virus was callcd "non-A. non-B hcpatitis." Rccently three more viruses havc becn identilied that causc some ofthese non-A. non-B infections.

These new Yiruses includ€:. Hepatitis C: is a serum hepatitis that is caused by a virus antigenically different fromHepatitis viruscs A and B. Most cases olpost-transfusion hepatitides are ofthis type. Itis usually much milder than A or B but is otherwise clinically indistinguishablc fromthcm. Thcrc is a higher incidence of chronic disease (chroni< lrcp41llrt, cirrhosis andhepatocellular carcinoma. Note; Hepatitis C is now the most common reason for livertrirnsplantation in the U.S.. Hepatitis D is found only in patients with acute or chronic episodes of Hepatitis B,and it makes the Hcpatitis B infcction morc sevcrc. Drug addicts are at relatively high

. Hepatitis E: is transmittcd cntcrically, much like Hcpatitis A. It causes occasional epi-d.'nics similar to those caused by Hepatitis A. So far these epidemics have occurred onlyin undcrdcvcloned countries.

Other factors that influence the rate ofhealing:. Location of the wound: wounds in an area in which there is a good vascular bed

hc'al considerably more rapidly than wounds in an area which is relatively avascular. Phl sical factors: severe trauma to tissue is a deterent to rapid woLlnd healing. The

local temperature in the area of a wound influences the rate of healing. ln environrnental hyperthemia, rvound healing is accelerated; while in hypothermia, healing is

dela1,ed.. Circulatory factors: anemia and dehydration have been found to delay the healing

of q ounds. -{ge of patient: wounds in younger persons heal considerably more rapidly than

r ounds in elderly persons.Infection: bacterial invasion will retard healing. Hormonal factors ACTH and cortisone are substances that have been known to in-terlere rvith the healing of wounds. Diabetes mellitus (in.sulin deJiciencl) is one ofthemost $ idely recognized diseases in rvhich there is significant, clinically evident. retarda-

tion in repair of wounds.

\utritional factors such as the amount ofprotein a patient is consuming is one ofthe most

rmpofiant factors which may effect the speed ofwound healing. Hypoproteinemia has been

shos'n to delay wound healing, while having a high protein diet has been shown to acceler-

ate u ound healing. Vitamins, especially Vitamin C, have been shown to be important inproper wound healing.

Page 57: Oral Pathology

. 3 mm well-encapsulated fibroma

. Necrotizing sialometaplasia ofthe hard palate

. 2 mm papilloma of left commissure of lips

. Aneurysmal bone cyst

. Nasopharyngeal carcinoma

. Oral hairy leukoplakia

. Burkitt's lymphoma

. Koplik spots

Copyrighr O 20ll-2012 " Dental Deckr

Copright O 201l-2012 - Denral Decks

Page 58: Oral Pathology

A biopsy is a procedure perfonred to remove tissue or cells from the body for examina-tion under a microscope. Biopsies are usually performed to detemine whether a tumor ismalignant or to detemine the cause of an unexplained infection or inflammation.

When the entire tumor is removed, it is called excisional biopsy technique. lfonly a por-tion ofthe tumor is removed, it is called incisional biopsy technique.

An incisionaf biopsy (also called diagnostic biop.st) is done when lesions are too largeto excise initially without having established a diagnosis or are of such a nature that ex-cision would be inadvisable.

An excisional biopsy is preferred ifthe size ofthe lesion is such that it may be removed along

u ith a margin ofnomral tissue and the wound can be closed primarily. Erample: A l -cm ex-

ophl-tic mass flfrich is a lesion that grovs ouf,rordfrom an epithelial surface) on the cheek.

l. Biopsy is the most reliable technique to diagnose soft tissue lesions.\ote* 2. The fixative ofchoice to preserve biopsy specimens lor routine histological

examination is l0%o buffered formalin.3. The scalpel is the instrument of choice since it cleanly removes the tissueand does not dehydrate it as cautery or the high-frequency cutting knife may..1. The rationale for surgical removal and biopsy of a large periapical lesionsuspected to be of inflammatory origin is that a clinical diagnosis can be con-firmed microscopically. This is the only way to distinguish between a gran-uloma and a cvst.

The Epstein-Barr virus (EBV) is a member olthe herpes virus group. It causes infectiousmononucleosis and has been associated with the subsequent development of two formsof cancer: Burkitt's lymphoma and nasopharyngeal carcinoma. EBV is also associ-

ated rr ith hairy leukoplakia, an opportunistic infection resulting in white patches oftheIareral tongue. Hairy leukoplakia is a nonmalignant lesion seen almost exclusively inAIDS patients. The virus specifically infects Blymphocytes and some epithelial cells.

Ir is associated rvith the production of atypical lymphocytes and lgM heterophile an-tibodies identified by the heterophile test (also callecl the mononucleosis spot lest).

This antibody eventually appears in the serum of more than 80% ofthe patients with ir-teL-rious mononucleosis, hence it is highly diagnostic of the disease

l.There are no specific oral manifestations ol infectious mononucleosis, al-\otes though secondary Iesions do occur

2. Neck srvellings are cl.raracteristic ofinfectious mononucleosis, Hodgkin's dis-

ease and tuberculosis.

Remember:. Rubella viruses cause German measles (rubella),which present with a characteristic

'ash (flat, pink spots on theface and then spreads to other bodl,parts).. Paramyxoviruses can cause measles (rubeola) and mnmps. Rubeola is characterized

by fever. malaise and by the formation of Koplik's spots in the oral cavity. These spots

are small. bluish-white lesions sunounded by a red ring. They cannot be wiped offandoccur opposite the molars. Mumps cause enlargement of the parotid glands. Seriouscomplications include deafness in children and orchtlis (in/lammation ol the testis) inmales past puberty.

Page 59: Oral Pathology

Misc.

Epithelioid cells and giant cells are derived from macrophrgesand are important in the development of:

. Initial inflammation

. Granulomatous inflammation

. Acute inflammation

. Subacute inflammation

CopyriSht O 201l -201: - Dental Decks

ORAL PATHOLOGY Misc.

A patient you saw yesterday had minor swelling of the submandibular spaceassociated with a carious #31. You prescribed amoxicillin and sent him home.II€ called today to say the swelling has gotten worse. You squeeze him into theschedule and notice that he has trouble breathing. You call an arnbulance toescort him to the ER and tell the paramedics that he has Ludwig's angina,

Ludwig's angina is a severe and spreading infection that involv€s the:

. Submental and sublingual spaces only

. Submandibular, submental, and sublingual spaces unilaterally

. Submandibular and sublingual spaces only

. Submandibular, submental, and sublingual spaces bilaterally

59

Copyrighr (} 20ll 20ll DentalDecks

Page 60: Oral Pathology

Granulomatous inflammation is a subtype of chronic inflammation and is characterizedby granulomas, which are nodular collections of specialized macrophages referred to as

epithelioid cells. A rirn of lymphocytes usually sunounds granulomas. Granulomas are

produced by multinucleat€d giant cells (Langerhans giant cells and./brcign bod1, giantcells). All the other cell types characteristic of chronic inflammation, including plasmacells, eosinophils, and fibroblasts, may also be associated with granulomas.

Note: Granulomatous inflarnrnation is characteristically associated with areas ofcaseous necrosis produced by infectious agents, particularly Mycobacteriumtuberculosis,

. Tuberculosis: caused by the inhalation of Mycobacterium tuberculosis. Oral non-healing chronic ulcers lollow lung infection. Important: Characterized by caseatinggranufomas with multinucleated giant cells (Langerhans giont cells andJbreign bodygiqnt cells).

Etiologic agents associated with granulomatous inflammation;

. Infectious agents

- TB and leprosy, which are mycobacterial diseases

Fungal infections (blastomyco,sis, histoplasmosis, qnd coccidioidom)'cosis)Spirochetes (Treponema pallidum, which causes syphilis)Cat scratch disease (caused h; an unnamecl gyam-negdive organisnt)

. Foreign material (e.g., suture or talc)

. Sarcoidosis (unknown etiology; it is non-necrotizing)

. Crohn's disease (il ls non-caseating)

Lud$ ig's angina oflen results fiom an odontogenic infection. As a result, the bacteriol-ogl of theses infections generally involves oral flora, particularly anaerobes. Other rec-ounized etiologies of Ludwig's angina include poor oral hygiene, IV drug abuse, trauma,and tonsiliitis.

It is characterized by:. Rapid onset. The three facial spaces are involved bilaterally. Board-like swelling offloor olthe mouth and no fluctuance is present. Tlpical "open mouthed" appearance. FIP\ siiAn of the tnnore. Drooling. trismus, and fever. Ditllculty eating, swallowing, breathing. Tachvcardia. increased respiration rate. Can lead to glottal edema; asphyxiation

-\inr av management, massive antibiotic coyerage (IV), and surgical incision and drainageare the mainstays oftreatment.

Verv important: The most serious complication of Ludwig's angina is edema of theglottis (i'hich is a slit-like opening behreen the true vocal corcls).

Page 61: Oral Pathology

. Inflammation

. Administration of hydroconisone

. Necrosis

. Suppuration

. Pregnancy

60Coptrighr O 201 l-2012 - Dflld Decks

. l-2 mglkg

. 5- l0 mg/kg

.8-10 mg/kg

. t2-15 mg/kg

61

Copright O 20ll-2012 - Denbl Deck

Page 62: Oral Pathology

The ESR is the rate at which red blood cells settle out in a tube of unclotted blood, ex-pressed in millimeters per hour Blood is collected in an anticoagulant and allowed to sed-

iment in a calibrated glass column. At the end ofone hour, the lab techniciar measures the

distance the erlthrocytes have fallen in the tube. Elevated sedimentation rates are not spe-

cific for any disorder but indicate the presence of inflammation. Inflammation causes

an alteration ofthe blood proteins which makes the red blood cells aggregate, becomingheavier than normal. The speed with which they fall to the bottom ofthe tube correspondsto the degree of inflammation.

ESR rises during:. lnflammation. Tissue degeneration. Suppuration. Necrosis

l\Jote: Certain non-inflammatory conditions, such as pregnancy, are also categorized byhish sedimentation rates.

*** Death may result from ingesting as little as 2 g offluoride in an adult and l6 mg,&g in chil-dren. Symptoms may appear with 3-5 mg/kg offluoride.

S) mptoms of acute fluoride poisoning include nausea, abdominal pain, vomiting, diarrhea,

conr ulsions. and hypotension.

The treatment for acute fluoride poisoning includes: 1) Call poison control center, 2) Monitor vital

iigns. 3) Initiate basic life support as needed and 4) Get patient to the hospital

Fluoride poisoning may be ac\te (caused b1' a single larye close offluoride) or chronic (caused

hr long-term ingestion offluoride). The characteristic signs ofchronic fluoride poisoning are:

l. Osteosclerosis of the bones: which results fiom long-term ingestion of water with l0to 15 ppm offluoride.:. Dental fluorosis (enamel hypoplasia): which is due to fluoride intake during the cal-cification stage oftooth development. This can occur in permanent and deciduous teeth.

\ote: [t has been estimated that the average American diet contains about 0.2 to 0.3 mg offluoride per day. lf I ppm of fluodde is added to the drinking watel about I to 2 mg of flu-oride will be added to the diet daily. Balance studies have shown that when quantities offlu-oride ingested do not exceed 4 to 5 mg daily, little is retained by the body. The finding in-

dicates the safety of the preventive dentistry programs based on the addition of fluoride todrinking water in concentrations of approximately 1 ppm.

Important: Fluoride normally accumulates slowly in bones as a person ages. However, ifingested in very high amounts, it accumulates rapidly. The intake of calcium in high doses

rvill reduce the absomtion ofdietarv fluoride.

Page 63: Oral Pathology

. Hereditary

. Medications

. Tooth decay

. Mouth breathing

. Tongue and mandibular alveolar ridge

. Buccal mucosa and pharyngeal pillars

. Palate and maxillary gingiva

. Mucobuccal fold of lower lip

62Coplright O 201 1-201 2 , Dental Decks

63

Copyriglr @ 201l-2012 - D€ntal Decks

Page 64: Oral Pathology

Xerostomia (dry nouth) is not a disease, however, it can be a symptom of certain dis-eases. Many times xerostomia is caused by failure ofthe salivary glands to function nor-mally, but the sensation can also occur in people with normal salivary glands. Xerostomiacan cause health problems by affecting nutrition, as rvell as psychological health. At itsmost extreme, it can lead to rampant tooth decay and periodontal disease.

Perhaps the most prevalent cause of xerostomia is medication. The main culprits are

antihistamines, antidepressants, anticholinergics (e.g., atropine and scopolamine),anorexiants, antihypertensives, antipsychotics (e.g., chlorpromazine and prochlorper-azine), anti-P arl<tnson agents, diuretics and sedatives.

The most common disease causing xerostomia is Sjiigren's syndrome /S.9/, a chronicinflammatory autoimmune disease that occurs predominantly in postmenopausal women.Sarcoidosis and amyloidosis are other chronic inflaurmatory diseases that cause xeros-tomia. Other systemic diseases that can cause xerostomia include rheumatoid arthritis,sl stemic lupus erythematosus, and scleroderma. Remember: Xerostomia is the mostcommon toxicity associated with radiation therapy to the head and neck.

Comrnercially available saliva substitutes in general, contain an agent to lncrease vlscos-iq. such as carboxymethylcellulose or hydroxymethylcellulose, minerals such as calciumand phosphate ions and fluoride, preservatives such as methyl- or propylparaben, and fla-\ oring agents. Examples include: Xero-Lube,i Salivarti and Optimoist.s

\lalignant melanoma accounts for only 4% of all skin cancers; however, it causes the

createst numbcr ofskin cancer rclatcd dcaths worldwide.

Clinical features:. Malc' morc oftcn than fcmalcs. L sually palalc or rnaxillary gingiva. 5th decade or older. A. B. C. D's of melanoma:

-\ = asymmctryB = border inegularityC : color variability (brown, black, blue, gray, pink)D = diameter of /o/ief greater than l/4 inch

Excessive exposure to UV radiation from the sun may bc the primary cause ofmclanoma.\falignant melanoma has bcen linked to both a lot of sun exposure over a lifetime and topainful sunbums during childhood.

\{alignant mclanoma is an uncommon neoplasm ofthe oral mucosa. It exhibits a definitepredilcction for the palat€ and the maxillary gingiva/alveolar ridge. Unfortunately, oralmucosal melanomas have a dismal prognosis. See picture #36 in booklet

\ote: A nevus is a mole. Almost all moles are normal. Atypical (dl,splastic) nevi are un-

usual moles that are gencrally larger than normal moles and are eithcr flat or have a flat part.

They have irregular bordcrs and oftcn are variable shades olcolor, particularly brown. Thepresence ofdysplastic nevi may mark a greater risk ofmalignant mclanoma developing on

apparently normal skin. See picture #46 in booklet

Page 65: Oral Pathology

Name the malignant, epithelial cell tumor thrt chancteristically begins asa papule and enlarges peripherally, developing a central crater thaterodes, crusts and bleeds. An example ofthis tumor is shown below.

ORALPATHOLOGY

A 62-year-old patient comes into your oflice complaining of his loose lowermolars. The health history reveals recent lower back pain. On a hunch, you

send him down the hall to the orthodontist to take a lateral skull radiographwhich reveals "punched-out" radiolucencies. You will refer this patient to the

physician suspecting a diagnosis of:

. Non-Hodgkin's lymphoma

. Hodgkin's lymphoma

. Multiple Myeloma

. Langerhan's Cell Disease

Copyrighr C 20ll-2012 - Dental Decks

Page 66: Oral Pathology

Basal cell carchoma is a common low-grade skin cancer that rarely metastasizes, but thelocal invasion by direct extension destroys the underlying and adjacent tissue. lt frequentlydevelops on the exposed surfaces of the skin, face and scalp in middle-aged or elderlypersons, it is very rare in mucosa. The primary cause ofthe cancer is excessive exposureofthe sun or to x-rays.

Clinical features:. Presents as a nonhealing, indurated chronic ulcer. Males are more affected than females. 4th decade of life or older. Located primarily on sun exposed areas ofhead and neck with the nose being the mostcommon srte

Risk factors: childhood sun exposure, blistering sunbums, fair skin, blue eyes, blondeor red hair.

Important: It is locally destructive and metastases are exceptionally rar€.

The treatment for basal cell carcinoma is eradication ofthe lesion, often by electrodessi-

cation or cryotherapy. Recurrence is unconmon ifproperly treated.

r'-ote: MOHS micrographic surgery is the most effective and state-of-the-art procedure

for skin cancer today, which offers the highest potential for recovery - even if the skincancer has been previously treated by another method and recurred. With the MOHS tech-

nique. physicians are able to see beyond the visible disease, to precisely identify and re-

moVe the entire tumor layer by layer while leaving the surrounding healthy tissue intactand unharmed. As the most exact and precise method of tumor removal, it minimizes the

chance ofre-growth and lessens the potential for scarring or disfigurement.

\Iultipfe mleloma f.r lso known as "Plastna Cell Mveloma') is a disease ofabnormalplasma cells that

mosr often build up in the bone marrorv. The cells form tumors in many bones ofthe body.

Clinical Features:. \Ien 2:l (40-70 rears o/r// Vcrtcbrae, ribs, and skull are most frcquently involvcd; pain inlumbar or thoracic region is a common early symptom.. Jaws are rarely a primary site, but become involvcd in 70% of the cases, molar ramus area

most common site. Symptoms include swelling, pain, loosening ofthe teeth, and paresthesia.

Radiographic features: variable; slight demineralization to extensive dcstruction, characteristic

finding rs multiplc. small, discreet "punched out" radiolucencies in involvcd bones ln a patient

:u:pccted ofhaving multiple myeloma. a lateral skull radiograph is best to confirm the diagnosis.

Laborator-'- fi ndings: important in establishing diagnosis:

. Abnormal monoclonal immunoglobulin protein peak known as an M spike. The immunoglobulinis usualll ofthe IgG or IgA class. with a monoclonal Jight chain cofrponent.. Lrinary monoclonal light chains, so-called Bence Jones proteinu a

. Anemia

Treatment and Prognosis: chemotherapy, radiation; prognosis poor.

Important: A form ofamyloidosis occurs in l0% of multiple myeloma patients. It is due to the forma-

rron of complex proteins in which immunoglobulin light chains are precursors. Amyloid ptotein is de-

posited in various organs an can lead to organ dysfunction f€speciallr kidnq', hedfi, GI ltucl, liver and

I Plasmacytoma is a localized collection ofmonoclonal plasma cells. The discasc is dividcd

\61*,-'intoprimaryplasmacytomaoftheboneandextramedullaryplasmacytoma.Thcimportance- ,.:rr' ofthe diagnosis restswith thc potential forthese disorders to progress to multiple myeJoma.i.';$t:l: 2 Primary presentation of intraoral non-Hodgkin's lymphoma is uncommon. In gencral,

the oral manifestations occur secondary to a morc widespread distribution throughout the

bodY3. Hodgkin's disease involving the oral cavity is also considered a raity, but it still can ap-

pear here initially.

Page 67: Oral Pathology

. Superficial spreading

. Lentigo maligna

. Acral lentiginous

. Nodular

66

Cop"ighi @ 201 I '2012 - Dental Decks

. The presence of Nikolsky's sign

. The presence ofnodules

. The presence ofregional lymph node involvement

. The presence of erythroplakia

CopFight O 20ll-2012 - Dental D€cks

Page 68: Oral Pathology

Malignant melanoma involves the cells /melqnoc),tes) that produce pigment (zrelarirl, whichis responsible for skin and hair color Melanoma can spread very rapidly and is the most deadlyform ofskin cancer. lt is the leading cause of death from skin disease. Melanoma may appearon normal skin, or it may begin at a mole frevusl or other area that has changed in appearance.

Some moles present at birth may develop into melanomas. The development of melanoma is

related to sun exposure, particularly to sunbums during childhood, and is most common amongpeople with fair skin, blue or green eyes, and red or blonde hair.

There are four major types ofmelanoma;

I . Superficial spreading melanoma is the most common type of melanoma. It is usuallyflat and irregular in shape and colol with varying shades ofblack and brown. lt may occurat any age or site and is most common in Caucasians.2. Nodular melanoma usually starts as a raised area that is dark blackish-blue or bluish red,although some lack color Poorest prognosis,3. Lentigo maligna melanoma usually occurs in the elderly. It is most cornmon in sun-damaged skin on the face, neck, and arms. The abnormal skin areas are usually large, flat,and tan with intermixed areas of brown. DeveJops from pre-existing lentigo maligna(Hutchinson freckle).,1. Acral lentiginous melanoma is the least common form of melanoma. It usually occurson rhe palms, soles, or under the nails and is more common in African Americans.

Important: Malignant melanoma exhibits either initial radial (do not metastqsize) or yertr-c^l lnetastasis mdf occur) gtowthpattems within the skin. Radial growth phase melanomas

include the superficial spreading, lentigo maligna, and acral lentiginous melanomas. Verticalqro\\th is characteristic of nodular melanoma.

\ote: Skin cancer is a very common malignancy in the United States. Olthe different types,basal cell carcinoma is most common followed bv souamous cell carcinoma and the least

common is malisnant melanoma.

Clinical staging of malignant neoplasms:

The T:-Nl method of assessing the prognosis and thempy of malignant neoplasms is based

upon 1) the size ofthe primary tumor, 2) the presence ofregional lymph node involvementand 3) the presence of distant metastases. This is represented as:

. T : size of the primary tumor

. \ = presence ofregional lymph node involvement

. \I : presence ofdistant metastases

\ote: The use of this classification system is becoming less common. A major short-corring ofthe TNM classification is the lack ofquantification of depth/penetration oftheumor.

Page 69: Oral Pathology

. Dorsum

. Ventral surface

'Tip

. Posterior lateral border

. Squamous cell carcinoma ofthe palate

. Squamous cell carcinoma of the nasopharynx

. Squamous cell carcinoma ofthe oropharynx

. Squamous cell carcinoma ofthe maxillary sinus

68Coplrighr O 201l-2012 - D€nral Decks

69

Coplaight @ 201l'2012 - Dental D€cks

Page 70: Oral Pathology

Squamous cell carciloma i'SCC) accounts for greater than 9070 oforal cavity and oropharyngeal malig-nancics. Lip carcinomas account for 25,q/o b 30o% ofall oral cancers. Approximately 909/0 occur on the

lower lip due to increascd sun exposure. IntraorallJ', the most common site is thc posterior lateral bor-der of the tongue, it is particularly uncommon to have lesions develop on thc dorsum or in the tip ofthetonglle. Metastases from tongue cancer are relativcly common at thc time ofprimary trcatmcnt. In gcn-eral, metastatic deposits from SCC ofthe tongxe are found in the lymph nodcs ofthe neck. usually on

the ipsilateral side. The first nodes to become involved are the submandibular orjugulodigastric nodes

at the angle ofthe mandiblc. Noter The floor of the mouth is the second most common intraoral lesion

of SCC.

f,liolog/ Clinic!l Chrrrcterirtics

Lipsurlighl- pjpe

NIen betwee. 50 10. rermil-lion oflower lip. chronrcnonhe!liig ulcer, keratotic

Lo*er liP,

Upper lip -

Tongue Men oler 60. posterio.Irter|l border . indurarednon healing ulcersith

leukoplak'a, erytbroplakia

Fair;

indurated ulcer: leukoplakia.

and girgivaMe! in their 70 s: painless

exophyl'c'nas verrucons

Soft palate-10-20 %asympbmatic. red or sh'teplaqres as ul.erared and

Locrtion Incidence Etiology Clinical Charrcteristics Treatment

\asopharyrL\ Less than 2% of allcancers in fhe U.S.

Tobacco, alcohol Men 30-40 years old, roofor latcral wall is most com-mon site, cervical mass,

earache sore throat, nasal

obsnucion

Su.gery andirradiation

Palate l0% ofalloral Tobacco, alcohol,d€ntue irritation

Men over 60, soft palatc ismore common than hardpalate, painful ulce., Ieu-koplakia, cxoph'1ic mass

Surgery andinadialion

Orophartr{ l0% ofall hcadand neck cancers

Tobacco, alcohol Men over 50. sore throat,dysphagia, paintul ulcer,cewtcal mass

Surgery andirradiation

\laxillary sinus 30% ofallheadand neck cancers

Unknown Men over 40, chronicsinusitis, butging of thepalate, loosening of teeth,paresthesia in cheek

Surgery andiradiation

Page 71: Oral Pathology

. "Horizontal" growth phase

. "Circular" growth phase

. "Vertical" growth phase

. "Radial" growth phase

' Lip

. Tongue

. Floor of mouth

. Buccal mucosa

70

Cop)'righl O 20ll-2012 - Dental Decks

71

Coprrighr O 201 | -2012 - Dental Decks

Page 72: Oral Pathology

Melanomas develop initially as a flat phase without competence for metastasis calledthe "radial growth phase". This refers to the initial growth of a melanona in a horizon-tal plane. It is clinically macular or only slightly elevated. They then may evolve focallyan elevated part, the vertical growth phase, with metastatic competence.

The "vertical" growth phase is the phase that begins when neoplastic cells populate theunderlying dermis. lt is characterized clinically by an increase in size, a change in color,nodularity and, at times, ulceration. Metastasis is possible once the melanoma reaches thisphase. Overall, patients with vertical growth phase melanomas have a cure rate of 7070.

Note: This phase predoninates in nodular melanoma.

Radial growth phase melanomas, although invasive, have a cure rate that approaches1009'o with surgery alone. Radial growth phase melanomas include the superficial spread-

ing. lentigo maligna, and acral lentiginous melanomas.

Generally, the radial growth phase is described by the A,B,C,D's of melanoma:

A : asynmetryB : border irregularityC : color variability (brolvtr, black, blue, gray, pink)D = diameter of (olexl greater than 1/4 inch

Remember: Malignant melanoma is an uncommon neoplasm of the oral mucosa. It ex-hibits a definite predilection for the palate and the maxillary gingiva/alveolar ridge.Untortunately. oral mucosa melanomas have a dismal prognosis. The five-year survivalrate for such tumors is < 20olo (/br skin lesions it i,s > 65%o).

Cancer ofthe tongue causes more deaths than do malignant lesions in other regions ofthe head

and neck. This has been attributed to the fact that it is a highly mobile organ that is richly en-

dorved rvith lymphatics and blood vessels which facilitate metastases. IntraorallY. the most

c'ommon sjte is the posterior lateral border of the tongue, it is panicularly uncommon to

har e lesions develop on the dorsum or on the tip ofthe tongue. Remember: Metastases tiomronsue cancer are relatively common at the time ofprimary treatment. In general. meiastatlc

deposits from SCC ofthe tongue are found in the lymph nodes ofthe neck, usuaily on the ip-silateral side. The first nodes to become involved are the submandibular or j ugulodigastricnodes at the angle ofthe rnandible.

Cancer ofthe lips: lip carcinomas account for 25 to 30% ofall oral cance$. Approximately900 o occur on the lower lip due to increased sun exposure. Pipe smoking has also been im-phcated in the etiology.

Cancer of the lloor of the mouth: is the second most common intraoral location of SCC. ltoccurs most comlronly in the anterior segment on either side ofthe midline, near the orificesofthe salivary glands. Metastasis to submandibular lymph nodes is not uncommon for lesions

tbund here. Note: Prognosis is very poor for lesions found here.

Cancer of the buccal mucosa: generally occurs along the plane of occlusion, midway an-

teropostenorly.

Cancer of the gingiva: is more common in the mandible than in the maxilla, and posterior

siles are seen more frequently than anterior

Note: Overall the 5-year suwival rate for SCC is 45% to 50%; with neck metastasis it is 25%.

Page 73: Oral Pathology

ORAL PATHOLOGY

A 47-year-old IIIV* patient is referred frorn his physician to you because ofanexophytic growth in the maxillary l€ft mucobuccal fold. Unable to find an

odontogenic source, a triopsy was done. A starry-sky morphology wasfound and a dismtl diagnosis of Burkitt's lymphoma was made.

Which virus is thought to be responsible for this lymphoma?

. Herpes virus

. Epstein-Barr virus

. Cytomegalovirus

. Human papillomavirus

72

CopyriShr lil20l l l0l: DenralDecks

ORAL PATHOLOGY Neo

At the age ofnine, poet Lucy Grealy was diagnosed with Ewing's srrcoms ofthe jaw. Although rare in fernales, and rare in the jaw, Ewing's sarcoma

most oft€n presents radiographically as:

. Multiple radiolucent/radiopaque lesions resembling "cotton ball" or "cotton wool"appearance

. Multiple "punched-out" radiolucencres

. "Moth-eaten" destructive radiolucencies ol medulla and erosion of the conex witlrexpanslon

. Lytic lesion that rnay be ill-defined or sharply defined

73

CoDriShr C 20ll l0ll ' Dental Decks

Page 74: Oral Pathology

Burkitt's lymphoma is a high-grade, non-Hodgkin's lymphoma that is endemic inAfrica and occurs only sporadically in North America. It is manifested most often as a

large osteolytic lesion in the jaw (African fom) or as an abdominal mass (Non-African

form).

Burkitt's llmphoma is the first human cancer with strong evidence of a viral etiology. TheEpstein-Barr virus (t herpes-4tpe virus) has been isolated from cultures of tumor cellsand patients with Burkitt's lymphoma have high titers ofantibodies against EBV Also, an

antibody against a surface antigen on the tumor cells has been demonstrated,

Note: The Epstein-Ban virus is also associated with infectious mononucleosis, oralhairy leukoplakia, and nasopharyngeal carcinoma.

Two forms of Burkitt's lymphoma:

1. African: younger (mean age 3), male predominance, typically involves the jaws.

2. \on-African: older (mean age I I), no sex predilection, presents most often as an ab-dominal mass.*** Both forms are histologically identical.

\ote: The jaw lesions usually present as expanding intraoral masses on the palate and

ringir,a. Lesions appear as soft tissue nodular masses and many are hemorrhagic.See picture #37 in booklet. Radiographically, there is a moth-eaten, poorly marginated

destmction ofbone. See picture #38 in booklet

Eling's sarcoma is an uncommon, highly lethal, round cell sarcoma ofbone ofuncer-tain origin. The most common sites for Ewing's sarcoma are the pelvis, the thigh, and the

trunk ofthe body. When the jaws are involved, there is predilection for the ramus ofthemandible. Ninety percent of Ewing's sarcoma occur between the ages of5 and 30 years.

and over 607o affect males. See picture # 39 in booklet

Pain. usually ofan intermittent nature, and swelling ofthe involved bone are ollen the ear-

liest clinical signs and symptoms of Ewing's sarcoma. Involvement of the mandible olmaxilla may result in facial deformity, destruction of alveolar bone with loosening ofreerh. and mucosal ulcers. Feveq leukocytosis, raised ESR, and anemia are also present.

Radiographically. the most characteristic appearance is that ofa moth-eaten d€structiveradiolucency of medulla and erosion ofthe cortex with expansion. A variable periosteal

"onion-skin" reaction may also be seen. See picture #40 in booklet

Histologically, it is often difficult to distinguish this tumor from a neuroblastoma or areticulum cell sarcoma, however, the cells of Ewing's sarcoma contain glycog€n.

The highly malignant nature ofthis sarcoma is reflected in its propensity for metastasis,

especially to lungs, other bones, and the lymph nodes. Multiple method treatment proto-cols. including surgery or radiation for local control and chemotherapy for systemic mi-crometastases have dramatically improved the formerly dismal l07o 5-year swvival rate

to a 60olo 5-year suryival rate.

Important: The most common osseous malignancies are osteosarcomas, followed bychondrosarcomas, fibrosarcomas and Ewing's sarcoma.

Page 75: Oral Pathology

ORAL PATHOLOGY Neo

A {ifteen-year patient presents to his physician because of localized painin his right femur and rapidly enlarging swelling. A radiograph

ofthe area shows a "sun-ray" app€arance, Which is a likelydiagnosis for this patient based on incidence?

. Chondrosarcoma

. Osteosarcoma

. Sclerodema

. Chronic osteomyelitis

74

Copyrighr..C 20ll':012 - Dental Decks

ORAL PATHOLOGY Neo

While doing a dental mission trip in f,thiopia, you notice purplisb-brownnodules on th€ hard palate of a 32-year-old female patient. You notice more ofthese spots on her arms and legs. Given that HMnfection has an almost 59lo

prevalence in urban Addis Ababa, what neoplasm are you suspicious ol?

. Nicotinic stomatitis

. Hemangioma

. Kaposi's sarcoma

. Leukemia

Cop).right ia: 20ll-:012 - Dentd Decks

Page 76: Oral Pathology

Osteosarcomas bl.so called osteogenic sdrcomas) account for approximatcly 20Yo of all sarco-mas and, alier plasma ccll rnycloma, arc thc most common primary bone trnnors. Conventional os-

tcosarcomas involving thc rnandible and maxilla display a predileclion for malcs f629l,,/. Thosc

arising in the jaws have a mean age of34 years. The majority (60o/o) ofnrandibular osteosarcomas

arise in thc body ofthe mandible. In the maxilla, there is nearly equal incidcnce ol-tumors involv-ing the laveolar ridge and maxillary antrum, u,ith feu, a1'ficcting the palate.

Osteosarcomas involving the mandible prcscnt most commonly with swelling and localized pain.ln some cases, there may bc loosening and displacement of teeth as well as paresthesia duc toinvolvement of the inl'crior alveolar nerve. Maxillary tumors display similar clinical symptoms

but nlay cause paresthesia ofthe infraorbital nervc, epistaxis. nasal obstnlction, or cyc problems.

Radiographicalln early ostcosarcomas nray be characterized by localized widening ofthe peri-odontal ligament space of onc or two tccth. Advanced tumors can be visualized as moth-eatenradiolucencies or irrcgular. poorly marginated radiopacities. See picture #42 in booklet

Osteosarcomas are best treated by radical mandibulectomy or maxillectomy, with radiothcrapyand chemotherapy for rccuncnces, soft tissue extension, or metastatic diseasc. As with most ma-

Lgnanl jaw tumors. initial radical surgery resulls in superior survival ratc of 807o compared withl'o D sunival lbr local surgery. Osteosarcomas ofthc jaws frcqucntly recur l/.;/0 lo 709'0), \Nith as

n'relastalic rate of 25 to 50o/o. The most common sites ofmetastases arc lung and brain.\ote: Prognosis is better for mandibular tumors than lor maxillary tunors.

\ote: Chondrosarcomas arising in the mandible and maxilla are extren'tely mrc and havc countcdli'r ipproximately l9lo of all chondrosarcomas ofthe entire body. Cartilagc is produced by tumor.ells. The most common signs are a painless swelling and expansion ofthe affected bones, re-

:rltrng in loosening ofteeth or ill-fitting dentures. Radiographically, the lesions vary fiom moth-

laten radiolucencies that arc solitary or multilocular to diflusely opaquc lcsions. The trealment isir rdc Iocal or radical excision. Note: They are radioresistant.

Kaposi's sarcoma is a malignant ncoplasm ofcndothelial cells. It is characterized by abnormal vascularprofiferation (it is o concer oftl1e lining of blood t'esselsi. It occurs on multiple sites, cspecially thclo\ er extremities. lnitial lcsions are srnall, red papules, which enlargc and fuse to form purplc_to-broq'n.

ipLrngl nodules. It spreads to lymph nodes and intemal organs. Note; Human herpes virus 8 has an eti-

oloclc role.

Kaposi s sarcoma is a uniquc form of angiosarcoma and is the most common cancer associated *ithAIDS patients and has a predilection for the palate.

Tbree different clinical patterns:. Clas\ic lfediterranean type. Endcmic African type

' I mmunodeficiency rJpc (AIDS-tetuted)

Classic Type Africeo Typc lmmunodeficienc! T\pe

Gcogmphy Mediterranean basin Afiica

Rare Endenic Relari\ely (ommon

Older nren Children and adul$

Skin tesions Extremilies

Oml lesions Rare Rare Common

Olher organs Occasionally Occasionally Frequently

Coulse Indolent Prolonged

Prognosis l:air Fair

\ote: Inrraorally. the hard palate is the most common localion, followcd by the gingiva and buccal mu-

R€memberAIDS is caused by the RNA refovirus, HIV (d&o tdllel HTLV-lll). The HIV infection is

acqnircd with IV drrrg use as we1) as by scxual conlact (homosexual and heterosexnal) and contaminated

blood Droducts.

Page 77: Oral Pathology

. Lipoma

. Ranula

. Lymphoma

. Oral lymphoepithelial cyst

76

Coplrighr O 201l-2012 - D€nral Decks

. They may be completely asymptomatic

. The patient is usually aware ofslight discomfort or pain

. The maxilla is affected far more frequently than the mandible

. The molar region is predominantly involved

77Coptright @ 20 t 1,20 t2 - Denral Deck

Page 78: Oral Pathology

Almost all oral lymphoepithelial cysts arc lcss than 0.6 cm. in diameter at thc timc ofdiagnosis, whichis usually during the teen years or the third decade oflife. The site most colnmonly affected is thc floorof the mouth, but the lateral and ventral tongue are not uncommon sites ofoccurrcnce. nor is thc softpalarc, espccially lhe mucosa abovc thc pharyngcal tonsil. This cyst has a clinical appearance similar tothat ofan epidermoid cyst or a dcmroid cyst ofthc oral/phanngcal mucosa, but its growth potential is

much lcss than ihc othcr cysts.

Lymphomas arising rvithin the oral cavity account for less than 59/o oforal malignancies. In the head andncck. most lymphomas start in the rcgional Iymph nodes or rvithin cxtranodal lymphoid sites in areas

known as mucosa-associated lymphoid tissues (MALT h'nphonrur./. Within thc oral cavity. lymphoidtissue is chiefly represented in Waldeyer's ring fu ring oflvnphdtic tissue fonied br lhe tvo palatinetonsils. Ihe phanngeal tonsil,theli gual tonsil, dnd inteh^ening htnphoid tissre). Ofimponancc rcla-tive to lymphoma is that in both nodal and extranodal sitcs, neoplasia can occur A ftrndamental aim is

Io separatc lymphomas into two grollps:. Ilodgkin's t)pe: is charactcrizcd by painless enlargcmcnt of lymph nodes or extranodal tissue.\\'ithin Ihc oral cavity, tonsillar enlargement. usually unilateral. may be seen in the carly phascs. His-tologicalll-, cominon to all forms ofHodgkin's diseasc is the prcscncc ofmalignant l)'rnphoid cells andnon-ncoplastic inflamrnatory cells. including Iymphocytes, macrophages, eosinophils. and plasmacclls. Ofgreatest significance is thc identification ofthe Reed-Sternberg cell, which must be pres-ent fbr thc diagnosis of Hodgkin's disease to be established. Note: The Ann Arbor system of clini-cal stsging is uscd to determine the clinical extent ofthe disease and is an impofiant factor fbr dcciding!he tJ-pe and intensity oftherapy and helps dcterminc thc prognosis.. \on-Ilodgkins tlpei when primary oral soft tissues lesions are present (\'hich i.t rare). thcy arecharactcnzed by an absence ofsymptoms and by a relatively soft charactcr, oftcn $'ilh overlying r.rl-

.erations. If bone is the primary site. alveolar bone loss and tooth mobility are oftcn prcscnting

.rgns. S\!elling. pain, numbncss ofthc lip. and pathologic fracture may also be associatcd with bone

lesrons. In general. thcrc arc t\r'o groups ofnon-Hodgkin's lymphona noddar (bllicular"/ and diffuseibrms. For non-Hodgkin's Iymphomas. numerous classification schemes have cvolved. Thcsc in-;lude the classifications ofLuke and Collins, the WHO, Rappaport, and thc NIH International Work-lns Fonnulation.

*** This is false; the mandible is affected far more ftequently than the n.raxilla.

The most cornmon malignancy affecting skeletal bones is metastatic carcinoma. How-e\ er. netastatic disease to the mandible and maxilla is unusLra'l (onl.v about I %o). Most im-portantll, a tumor of the jaws may be the first evidence of dissemination of a kno$'nrurlrrr from its primary site.

\ote: \letastases to the jaws most commonly originate from primary carcinomas of the

breasr (ade o(arcinona), kidney (renal cell carcinona).lur'g (carcinona), colon (ade-

tr()Ltr(i .) t.t). proslale (adenocarcinona) and thyroid gland (ode ocarcinonn), in de-

creasing order of fiequency.

Clinical features of metastatic jaw lesions:

. Ofder adults (average age is 561,ears)

. The mechanism ofspread to the jaws is usually hematogenous from the primary vis-ceral neoplasm. \\'ithin the jar.v, the angle and body of the mandible are more commonly involved. Bone pain, loosening of teeth, lip paresthesia, bone swelling, gingival mass, andpathologic fracture may be clinically evident. The radiographic appearance of mostjaw metastases is poorly marginated' radio-lucent, irregular, moth-eaten, expansile defects

\ote: Microscopicalll', the diagnosis of metastatic carcinoma in difficult cases can be

r erified u,ith an immunoperoxidase stain for cytok€ratin, which is present in all carci-noma cells.

Important: The prognosis for patients with metastatic carcinoma to the jav's is grave,rvith a disrnal l0olo 5-year survival and over two-thirds dead within a year.

Page 79: Oral Pathology

Neo

Explain what is meant by a carcinoma ofthe oralcavity having the following TNM designation:

Tl, N2, Ml

7A

Coptrighr e 2011 201? Denral Decks

ORAL PATHOLOGY Neo

A 65-year-old patient ofEast Indian origin presents to your clinic with acomplaint of a lesion on the maxillary alveolar tuberosity. The lesion is a

thick white, exophytic mass with a cauliflower appearance. A social historyreveals that this woman has been chewing a betel-nut concoction from hernative India for over 40 years. \Yhat is the likely diagnosis of this lesion?

. Papilloma

. Erythroplakia

. Vemrcous carcinoma

. Hyperkeratosis

79Cop)righr O 20ll 201? Denral Decks

Page 80: Oral Pathology

Clinical Staging of Carcinoma of the Oral Cavity

. T = Size of the primary tumor

TX: Primary tumor can not be assessed

TO: No evidence of tumor

- Tis: Carcinoma in situT1: less than 2 cm in greatest diameter

- T2: 4 cm in greatest diameter

- T3: greater than 4 cm in greatest diameter

. N = Regional lymph node involvement

- NX: Regional lymph nodes can not be assessed

NO: No clinically palpable lymph nodes, or lymph nodes palpable but metastasesnot suspected

Nl: Palpable homolateral lymph node(s), not fixed but metastases suspectedN2: Palpable centralateral/bilateral lymph node(s), not fixed but metastases

suspectedN3: Palpable lymph node(s), fixed metastases suspected

. \I = Distant metastasis

- !tX: Presence of distant metastasis can not be assessed

lfo: No distant metastasis

fI1: Clinical and/or radiographic evidence of metastasis other than regionalll mph nodes

-\ r'errucous carcinoma is a well-differentiated squamous cell neoplasm ofsoft tissue ofthe oral or laryngeal cavity. The lesion may invade or infiltrate the borders of adjacent

structures but it rarely metastasizes. Verucous carcinoma may transform into an

invasile fom of carcinoma or coexist with other squamous cell carcinomas. It is often

misdiagnosed histologically as a benign lesion.

Clinical Features:. \{ale predilection. Broad-based, exophytic, indurated lesion. Diffuse, whitish, cauliflower or coral papillary mass. \{andibular mucobuccal fold, alveolar mucosa and palate. Slo*-growing, continuous enlargement. Painless

See picture #,13 in booklet

\ote: Tobacco and human papillomavirus (subtypes 16 and 18)may be etiologic fac-

tors. It is treated by surgical excision and has a good prognosis.

Page 81: Oral Pathology

Neo

A 73-year-old patient pr€sents to your office with complaint of a lastingulceration on th€ right side of his tongue. H€ has a history olhypertension

and high cholesterol and a 40-year pack history of srnoking. The patient takesanti-hypertensives and anti-lipidemics and has no allergies. You cannot lindany sources oftrauma. After two weeks, the ulcer has grown in size. What is

the likely diagnosis of this most common malignancy ofthe oral cavity?

. Adenoid cystic carcinoma

. Mucoepidermoid carcinoma

. Basal cell carcinoma

. Squamous cell carcinoma

80

CoDrighr Lq 20ll l0ll Denral Decks

ORALPATHOLOGY Nrv & Mus Disord

While attempting to give in inferior alveolar nerve block, ifyou inject theanesthetic solution into the capsul€ of the parotid gland, you may cause a

Bell's palsy like feeling for the patient by anesthetizing the:

o Triuern inrl newe

. Glossopharyngeal nerve

. Hl.proglossal nerve

. Facial nen'e

81

Copynghr C 201 l,:01 2 - Dentat Decks

Page 82: Oral Pathology

Squamous cell carcinoma is a malignant neoplasm ofstratified squamous epithelium that is ca-

pable oflocally destructive growth and distant metastasis. It is the most common type oforalcancer, accounting for over 90%o ofall malignant neoplasms ofthe oral cavity. lt is two times

more prevalent in m ales (40-65 years of4ge./. See pictures #44 and #45 in booklet.

Possible sites:. Lower lip (most common site). TottgU.e (posterior leteral border is the most common loccttion, dorsum least common lo-cqtiotr). Floor ofthe mouth (least favorqble plognosis). Sofl palate (u conmon). Gingiva/alveolar ridge. Buccal mucosa

Clinical f€atures:. Early presentation ofleukoplakias and erythroplakias. Painless ulcer, tumorous mass, or verrucous (papillary gro\tth). Occasional loosening or loss ofteeth. Possible paresthesia ofthe teeth and lower lip

Risk f'actors identified include smoking, alcohol consumption, painful and ill-fitting dentures,

chronic inflammation and the use ofsmokeless tobacco. Important: Tobacco use is the pri-mary risk factor Note: Causes ofgenetic alterations which results in loss ofcell cycle con-

trol. abnonnal signaling, increased cell survival, and cell motility include tobacco, human

papillomavirus (subq,pes 16 and l8), and heredity.

Remember: The most reliable histologic criterion for a diagnosis oforal squamous cell car-

cinoma is invasion,

Bell's palsy is a form offacial paralysis resulting from damage to the facial n€rve.It can

strike at any age; however, it disproportionately attacks pregnant women and people whohave diabetes, influenza, a cold, or some other upper respiratory ailment.

Clinical signs include a unilateral paralysis ofall facial muscles with loss ofeyebrow and

forehead wrinkles, drooping ofthe eyebrows, flattening ofthe nasolabial furrow, sagging

of the comer of the mouth and the inability to frown or raise the eyebrows. The upper

and loler lips may also be paralyzed on the side affected.

.{tier its sudden onset the paralysis begins to subside within two or three weeks, and grad-ual. complete recovery occurs in over 85yo of patients.

Triggering events related to Bell's palsy are acute otitis media, atmospheric pressure

change. exposure to cold, ischemia of the facial nerve near the stylomastoid foramen,

\lelkersson-Rosenthal syndrome, and multiple sclerosis. Note: Melkersson-Rosenthalslndrome is the term used when cheilitis occurs with facial palsy and plicated tongue.

Page 83: Oral Pathology

A 2s-year-old college student comes into your olfice cornplaining that when shewakes up she has trouble opening her mouth. When conducting a TMJ exam,you note tenderness ofthe right lat€ral pterygoid and non-reciprocal clicking

of the right TMJ. What is the most likely cause of the patient's myofacial pain?

. Trauma

. Muscle spasm

. Periodontal disease

. Tumor

a2

Copyrighr O 20ll'2011- Dental Decks

ORAL PATHOLOGY Nrv & Mus Disord

A S3-year-old patient comes to your o{Iice and notes that sometim€s whenhe swallows, he gets a sharp 'jolt" on the right side of his throat. He says thepain is sey€re and he can even feel it in his ear. The most likely diagnosis is:

. Postherpetic neuralgia

. Orolingual paresthesia

. Frey's syndrome

. Glossopharyngeal neuralgia

83

Coplriglrt C 20ll ?012 Denral Decks

Page 84: Oral Pathology

Such spasm may arise in one ofthree ways: muscular overextension, muscular over- con-traction or muscle fatigue. The most frequent cause ofthe spasm seems to be muscle fa-tigue. This syndrome is seen predominantly in women, usually in the 20 to 40 age range,and generally occurs unilaterally.

There are four cardinal signs and symptoms ofthe syndrome:

1. Pain2. Muscle tendemess

3. A clicking or popping noise in the TMJ4. Limitation in jaw motion (especially in the morning)

The pain itself is usually unilateral and is described as a dull ache in the ear or preauric-ular area, rvhich may radiate to the angle ofthe mandible, temporal area or lateral cervi-cal area. The muscle most apt to exhibit tenderness is the lateral pterygoid muscle.

\ote: There are no radiographic findings associated with MPS (mtofacial pain syn-

droDte)

Treatment: Most cases are selflimiting. Soft diet. limited talking, no gum chewing,moist heat. NSAIDs and Diazepam help relieve symptoms.

Glossopharyngeal neuralgia refers to pain similar to that of trigeminal neuralgia, which

arises frorn the glossopharyngeal newe (CN IX).lt is not as common as trigeminal neu-

ralgia. but the pain may be as severe when it does occur. It occurs in both sexes, in mid-dle-aged or olcler persons and is described as sharp, jabbing, electric, or shock-like pain

located deep in the throat on one side. lt is generally located near the tonsil although the

pajn may extend deep into the ear. It is usually tiggered by swallowing or cherving. It is

almost ahvays unilateral.

Frel 's sl ndrome (also called the auriculotemporal syndrome) ts an unusual phenome-

non. * hich arises as a result of damage to the auriculotemporal nerve and subsequent

reinnen ation of the sweat glands by parasympathetic salivary hbers.The syndrome fol-lo* s some surgical operation such as removal ofa parotid tumor or ramus ofthe mandible,

trr an infection of the parotid that has damaged the auriculotemporal newe (a branch ofI

---l /.

lmportant: Gustatory sweating is the chiefsymptom ofFrey's syndrome. The patient

rypically exhibits flushing and sweating of the involved side of the face during eating.

This syndrome is not a common condition.

Postherpetic neuralgia is a persistent burning, aching, itching and hyperesthesia alongdistribution of a cutaneous nerve following an attack ol herpes zoster. It may last for aferv rveeks or many months. lnvolvement of the facial nerve and geniculate ganglionproduces the Ramsey Hunt Syndrome, which is characterized by facial paralysis and

otalpia karache).

Page 85: Oral Pathology

ORAL PATHOLOGY Nrv & Mus Disord

Which of the following is a relatively rare autoimmune disorder ofperipheral nerves in which antibodies form against acetylcholine

(ACh) nicotinic postsynaptic receptors rt the myoneural junction?

. Myasthenia gravis

. Myelofibrosis

. Multiple sclerosis

. Graves' disease

E4

Copynghr C 201 l'201 2 - Dental Dects

ORALPATHOLOGY Nrv & Mus Disord

A 34-year-old dentophobe is your patient for the morning. After giving a

very hesitant health history, you decide to begin your oral exam, As youreach towerd her face, she immediately flinches and puts her hands up.

She lets you know that ifyou tough a particular point above her lip,she gets sharp, stabbing jolts ofpain. You let her know that a

neurologist can work her up for:

. Glossopharyngeal neuralgia

. Trigerninal neuralgia

. Postherpetic neuralgia

. Diabetic neuralgia

Copy.ighr (] 20ll ?011 Denral Decks

Page 86: Oral Pathology

Myasthenia gravis is an autoimmune disorder in which antibodies form against acetyl-choline (ACh) nicotinic postsynaptic receptors at the myoneural junction. The musclesare quickly iatigued with repetitive use. lt is tlpical for a myasthenic patient to have a flat-tened srrile and droopy eyes. with slow papillary light responses. Xerostomia and ram-pant caries may accompany myasthenia gravis. The acetylcholine that is necessary for the

proper transmission ofnerve impulse is destroyed, with the result that salivary glands donot receive adequate stimulation. Note: Head and neck manifestations include inabilityto focus eyes, drooping eyelids, double vision, difficulty in chewing and swallowing, andslurring ofwords.

Multiple sclerosis is a chronic, often disabling disease that randomly attacks the CNS

lhrain ancl spinal cord). It is believed to be due to an autoimmune response in which theinxnune system attacks a person's own tissue. Twice as many women as men have MS,s ith the onset of symptoms occuring most often between the ages of 20 and 40. Symp-toms may range from tingling and numbness to paralysis and blindness. Patients withmultiple sclerosis sometimes have facial andjaw weakness. In addition, both Bell's palsyand trigeminal neuralgia may develop more frequently in patients with MS.

Trigeminal neuralgia is an excruciating painful illness in which the afl-licted f'eels sud-

den stabJike pains in the face. The pains usually last only moments, but are among the

most severe pains humans can feel. The pain is provoked by touching a "trigger zone,"n picallv near the nose or mouth. lt is caused by degeneration ofthe trigeminal nerve or

b1 pressure being applied to it. Pain distribution is unilateral and follows the sensory dis-

tribution of cranial nerve V, typically radiating to the maxillary (I'-2i or mandibular/ I --l/ area. At times, both distributions are affected. The momentary bursts ofpain recur

in clusters. lasting many seconds. Paroxysmal episodes ofthe pains may last for hours.

\eccssary criteria for diagnosis of trigeminal neuralgia:. Onser ofpain initiated by a trigger point. Pain extreme. paroxysmal, lanclnatlng. Durltron is less thrn 2 minutes. -{ refractory period experienced for several minutes after attack. Pain limited to krown distribution of one or more branches of trigeminal nerve withno motor deficit in that area. Pain diminished with use ofcarbamazepine. Spontaneous remissions occur lasting tnore than six months during the early phase ofthe disease

The dnrg ofchoice for treating trigeminal neuralgia is carbzmazepine (Tegrelor. lt is an

analgesic and anticonvulsant. It is also prescribed in the treatment ofcefiain seizure dis-

orders. When used for the trcatment of trigeminal neuralgia, it usually rclieves the pain

u'ithin 48 hours aftcr treatment is startcd.

Page 87: Oral Pathology

. Median alveolar cyst

. Globulomaxillary cyst

. Nasolabial cyst

. Nasopalatine cyst

86

Copliiglt O 20l l-2012 - Dotal Deck

Afl ofthe followlng cysts are congenit EXCEPT one.Which one is the EXCI'PIIOM

. Thyroglossal duct cyst

. Branchial cyst

. Globulomaxillary cyst

. Dermoid cyst

87CopynShr O 20ll-201? - Dental Decks

Page 88: Oral Pathology

The nasolabial cyst has been called, inaccurately, nasoalveolar cyst. This latter designa-tion is inappropriate because the entity is not a true cyst ofthe maxilla. Rather, it repre-sents a soft tissue cyst without involvement ofthe alveolus, hence the prelerence for thedesignation "nasolabial cyst." Note: Because thjs cyst is extraosseous, it is not likely tobe seen on a radiograph.

Ouick reference for develoomental cvsts:

Nasopalatine- most common- heart shaped

Median AlYeolarII

Hard Palate

lII

- IAIE- teeth vital

*Nasolabial

- in sofi tissue

Median Palatal- rare

. Thc thl roglossal duct cyst, which may arise fiom any portion of the thyroglossal duct. Thiscy st is therefore lound in a midline position and is usually dark in color It may bc vascular as torcsenrblc a hemangioma. One lrequent important symptonr is hemorrhage into the mouth, re-sulting from thc rupture of the overlying veins. Complctc cxcision of the tracl to the base of theIongue. frequently including a portion ofthc hyoid bone, is nccessary tbr a curc. Note: Whcn mi-rration ofthc thyroid gland lails and the gland rernains in thc basc ofthe tongue. it is called lin-gual thl roid or eclopic lingual thyroid. Lingual thyroids are lour tjmes more comnon in females

than in rrales.

. The branchial /.1eli/ cyst or cervical lyrnphoepithelial cyst is located in the latcral portion ofthencck. usuallv anterior to the sternocleidomastoid muscle. Thcy may also appear in the submand-rbLrlar area. adjaccnt to the parotid gland. or around lhc stcmocleidomastoid muscle. The majorilyrr: lhese t-vpes arise ftom remnants ofthe second branchial cleft or pouch. This cyst is lincd with.rlified and slratified squamous epithelium and conlains a l11ilky or mucoid fluid. The treatment

aon5isrs ofconlplele surgical excision. Note: The branchial cyst has an intraoral countcrpart knowna: Ihe lt mphoepithelial cyst. The floor ofthe moulh is the most common site lor thcse lcsions, foll-o\\ ed bv the tongue

. Thc dermoid cyst is rclativcly uncommon in the oral cavity. This cyst frequcntly contains hair,\ebaceous and sweat glands, as well as tooth structures. The most common site is thc midline ofthe floor ofthe mouth ifabove the mylohyoid muscle.lt appcars as a mass in the upper neckif belol\ the mylohyoid muscle. The treatment is the surgical removal ofthe entirc tulnor

Important: Globulomaxillary cysts are thought to dcvclop fiom epithelial rcmnants remaining tbl-los ingjoining ofthc globular portion ofmcdian nasal process with thc maxillary process. They arccharacterized by a large "pear-shaped" radiolucency between the maxillary lateral incisor and

cuspid. Thcy arc asymptomatic, all regional teeth are vital. They tcnd to cause divergence ofthcroots. However, thcrc is considerable controversy as to whether this cyst actually exists. Many ofthese are, in reality apical cysts associated with non-vital lateral incisors. Olien thcsc apical cystshavc a tcndcncy to cxtcnd betwccn thc two tccth to simulatc a globulomaxillary cyst.

Page 89: Oral Pathology

All of the following cysts are developmenttl (or ftssural)EXCEPT one. Which one is the EXCEPTIOM

. Nasopalatine duct (canal) cyst

. Nasolabial (nasoalveolar) cyst

. Branchiogenic cyst

. Median palatal cyst

. Median alveolar cyst

88

Copynghr q 20ll'2012 - Dental Decks

ORAL PATHOLOGY N-O Cysts

The circular radiolucent area seen in this radiograph is clinicallyseen as a marked swelling in the region of the palatine papilla. It issituated mesial to the roots of the central incisors. The pulps of theanterior teeth in this patient tested yital. These findings would be

compatible with what diagnosis?

89

Copyrighr C 2011'2011- Dental Decks

Page 90: Oral Pathology

. Nasopalatine duct (canal): "hearrshaped" radiolucency in midline, most frequenttype of nonodontogenic cyst. Usually asymptomatic or may produce an elevation in theanterior part ofthe palate.Teeth are vital, Treatment is enucleation. Note: It is caused bycystification ofthe nasopalatine duct remnants. Remember: It is also known as incisivecanal cyst and may be located within the nasopalatine canal or within the palatal soft tis-sues at the point ofthe opening ofthe c anal (in this loccrtion it is called a cyst ol the pala-tine papilla).

. Nasolabial (nasoalveolar): is superficially located in soft tissues of the upper lip. Thisis an extraosseous cyst. Treatment is surgical excision.

. Nledian palatal: rare, may occur any'rvhere along median palatal raphe. May produces$ elling on palate. Treatment is enucleation. Note: Many oral pathologists now believethat this cyst represents a more postedor presentation of a nasopalatine canal cyst .

. \Iedian alveolar: rare. occurs in bony alveolus between central incisors. Distinguishedliom periapical cyst by the fact that adjacent teeth are vital. Treatment is enucleation.

Of the cysts ofthejaw, those that arise from epithelial remnants in the incisive canal are

the most common type of maxillary developmental cyst. Histologically, this cyst is linedrvith vessels, nerves and mucous glands in the wall. They most often remain limited as to

size and are asymptomatic. Some of them, however, become infected or show a

tendency to grow extensively. When tl'tis occurs, surgical intervention is indicated.

\ote: The soft tissue (ancl.far less common) v^ri^nt of the nasopalatine canal cyst is the

cr st of the palatine papilla.

\\'hen making a diagnosis ofthis cyst, the following two cysts should be ruled out:

. The globulomaxillary cyst: usually appears between the roots of the lateral inci-sor and those ofthe canine. lt is "pear-shaped" and often causes the roots ofinvolvedreerh to diverge. See picture f87 in booklet. The median palatal cyst: usually situated in the midline of the hard palate, poster-

ior to the premaxilla. Clinically, Ihis lesion presents as a firm swelling, which is usu-ally painless. Note: Some investigators now believe that this cyst represents a moreposterior presentation ofa nasopalatine duct cyst, rather than a separate cystic degen-

eration ofepithelial rests at the line ollusion of the palatine shelves.

See picture ff48 in booklet

Page 91: Oral Pathology

The soft tissue, and far less common, varlant ofthe nasopalatine canal cyst is the:

. Median mandibular cyst

. Nasolabial cyst

. Cyst ofthe palatine papilla

. Aneurysmal bone cyst

90

Coprighr O 2011,2012, DeninlDecks

. Heart disease

. Polyostotic fibrous dysplasia

. Cafe-auJait spots

. Endocrine dysfunction

91

Coplright O 201 I -20 l2 - Denral Decks

An &year-old girl who looks like she is 14 years old comes with her fathorinto your ollice. Her frther states during the health history that she hNs

Mccune-Albright syndrome. Which of the following would youNOT EXPECT the p.tient to have?

Page 92: Oral Pathology

The nasopalatin€ dvct cyst (NPDC) is a developmental non-odontogenic cyst that only oc-cuIS in the anterior maxilla from the embryonic remnants ofnasopalatine ducts. lt is usuallylocated between teeth #s 8 & 9 but can be as posterior as the mid-palate and as anterior as theincisive papilla. It is the most common non-odontogenic oral cyst of the oral caviry lt oc-curs at any age, but is most common in the fourth to sixth decade oflife and is more commonin males. It is usually asymptomatic and is discovered during routine dental examination. Ifinfected, the patient complains ofpain with swelling and even drainage; however, swelling is

not common. When present, swelling is often in the incisive papilla area in smaller lesions butcan be in the mid-palate in larger lesions and may be buccal and mid-palatal in very large le-sions. Radiographically it usually presents as a bilateral, well-circumscribed, round, ovoidor heart-shaped radiolucency. Tooth displacement or diverging ofthe roots ofthe central in-cisors is commonly identified. Depending on the size and clinical symptoms, treatment ranges

tiom no treatment to surgical curettage ifthe cyst is infected or interlering with a prostheticappliance. Recurence is rare and prognosis is good. Note: The cyst of th€ palatine papillais identical to that of its intrabony counterpart.

The median mandibular cyst, like the globulomaxillary cyst, was once considered a fissuralc.vst. Ho$'ever, embryonic evidence rcfutes thrs (there are no epithelial lined processes).It is

a \ ery rare cyst that appears in the midline ofmandible. Most of them are periapical, lateralperiodontal cysts or odontogenic keratocysts.

Remember: The nasolabial cyst has been called, inaccurately, nasoalyeolar cyst. This larter designation is inappropdate because the entity is not a true cyst ofthe maxilla. Rather, itrepresents a soft tissue cyst without involvement ofthe alveolus, hence the preference for thedesignation "nasolabial cyst." Note: Because this cyst is extraosseous, it is not likely to be

seen on a radiograph.

-\lbright's syndrome (also cqlled McCtne-Albright s-vtdrcrre/ is the most severe formof poll-ostotic librous dysplasia. It affects young people (males andJbmales equally). Itis characterized by Caf€-auJait spots on the skin, and endocrine abnormalities (the most

conunon ol u,hich is precocious sexual development infemales). The extent to which each

ofthese problems exist in those with the syndrome is quite variable. The hallmark of Al-bright's syndrome is premature puberty in the female. Early sexual development in the

nrale is less comnon than the female.

Clinical features:. Early childhood. \{ultiple, slow-growing, painless expansile bone lesions confined to the craniofacialarea or throughout the skeleton. endocrine manifestations; in females often sexual precocit-v.lrregular shaped Cafe-aulait spots on the torso and sometimes intraorally. Disfiguring. Increased level ofserum alkaline phosphatase

*** Pathologic fractures are frequently associated with this syndrome.

There is no specific treatment for this syndrome. Drugs that inhibit estrogen production,

such as testolactone. have been tded with some success.

Important: An additional complication is the malignant transformation potential ofboth the polyostotic (nainly) and monostotic fibrous dysplasia into osteosarcomas.

Page 93: Oral Pathology

An &year-old chlld has complained of pain on the left sido ofhis head for 5weeks. There are no abnomal findlngs on physical examination. A prnoramlrradiograph reveals multiple radiolucent lesions on the left side ofthe moxillo.

The lesions give ahe appearance of teeth thot rre "floating in space." Tbelesions are sharply circumscribed, with a punched-out appearance.

Which of the following is the most likely di.gnosis?

. Langerhans cell disease

. Hyperparathyroidism

. Cherubism

. Paget's disease

92Cop)rishr O 201I -2012 - Dental Decks

. Is a common lesion and appears more frequently than does the peripheral giant cell gran-uloma

. Is found predominantly in children and young adults

. Affects males more than females

.Is present almost exclusively in the small bones ofthe hands and feet

93CoDright A 20ll-2012 - Dental Decks

Page 94: Oral Pathology

ldiopathic histiocytosis or Langerhans cell disease, also formcrly known as histiocylosis X,is a disorder charactcrized by a prolifemtion of cells exhibiting phenotypic charactcristics ofLangerhans cells, The clinical manifestations of this process range from solitary or multiplcbone lesions to disseminated visccral, skin, and bonc lcsions.

Traditionally, idiopathic histiocytosis has been used to encotrlpass thrcc disorders: eosinophil-ic granuloma, Hand-Schullcr-Christian syndromc, and Lellcrcr-Si$,e disease.

. Eosinophilic granuloma (cltronic loculized Jbnn) has refened to patients with soJitary or rnul-tiplc bonc lcsions only. Hand-Schuller-Christian syndrome (chronic listelnitvl?d lbrn.) has represented a specificclinical triad of lytic bone lesions, exophthalmos, and diabetes insipidus. Many ol thesc paticnts

also cxhibit lymphadenopathy, dermatitis, splenornegaly, or hepatomcgaly.. Letterer-Silve disease (.rcute dissemitvted fonn) has been charactcrizcd by a rapidly progrcs-sir e. usually latal, clinical coursc. Widcsprcad organ, bonc, and skin involvement by the prolif-Lrdti\e lrocc:'s rn Inl'anls has been the comnton presenlalion.

ldiopathic histiocytosis is generally regarded as a condition ofchildren and young adults. Oralchanges may bc the initial presentation in all fonns ofthe disorder. Tenderness, pain and srvellingarc liequcnt patient complaints. Loosening ofthe teeth in thc arca ofthe affected alvcolar bone is

a conlnlon occulrcncc. Thc gingival tissues are frequently inflamed, hypcrplastic, and ulceraled.Theja\\ s mal exhibit solitary or multiple radiolucent lesions. The lesions frcqucntly ali'ect the alveo-lar bonc. rcsu)ting in the appearance of teeth that arc "floating in space." Bone lcsions rvitb a

.h:rrpiy- circumscribcd, punched-out appearance may also occu. in the central aspects of the

:randiblc or naxilla.

\licroscopicall!, eosinophils are mixed with thc tumor Langcrhans cells. some ofwhich arc mult-rnuc lci e d.

Prognosis is Very good when the disease is localized, however, the disseminated form is usually:...te1.

The central giant cell granuloma is a benign process that occurs almost etclusively within the jaw

bones. The tumor typically prescnts as a solitary, radioluccnt lesion ofthc mandible or maxilla- It is an

uncommon lesion and occurs less frequently lhan does the peripheral giant cell granuloma. It is

loL)nd prcdominantly in children and young adults. Fcmalcs are atltcted lnorc frcquently than males.

Lcsions occurmorc frequenlly in thc mandible than in themaxilla. These lcsions tend to involvc thcjawsentarior to thc molar teeth. Thc ccntral giant coll graiuloma typically produces a painlcss cxpansion or.\\ elling ofthc affected jaw

(-entral giant cell granuloma is classified into aggressive and non-aggressive types; the aggressrvc

lre tends to occur in younger patients and is known to cause disfiguration, especially after surgcry.

Radiographically, it consists ofa multilocular or, less frcqucntly. unilocular radioluccncy ofbonc. The

r::rgins of the Iesion are relatively well demarcatcd. often presenting a scallopcd border. Roots of the

i3.th nlay br displaced and. less commonly, rcsorbed. See picture #,ll in booklet

Important: Thc lcsion is composed ola proliferation ofspindled libroblasts in a slroma contain ing \ ari-

.rb.c anlounts of collagen. N{ultinucleated giant cells are ptesent lhtoughout the conneclivc tissuc

\otei Thc differential diagnosis includes amcloblastoma. odontogenic mlxoma, odontogenic kerato-

.l !t. and aneurysmal bone cyst.

Thc usual treatment is surgcry however. treatment altematives to surgery have emcrged with success_

1u1 results ranging from stcroid injections to calcitonin injcctions or nasal spray to interferon alpha-2a

injections . \\,hich are administered 2-J times pcr rvcck for several months.

. L The microscopic appearance ofccntral giant ccll granulona is vimrally identical to ihe

\otei' giant cell lesion /B/?rtri /rrrol/ associated lvith hyperparath] roidism. However. blood. . tests \r'ill shotv an increase in serum calcium and alkaline phosphatase and a decrcasc in--'

serum phosphorus in hypcrparathyroidism.2. The giant cell tumor of bone may present with similar clinical and mjcroscopic features.

However, the Siant cell tunor is regardcd as rare in thejaws in comparison to the cenffal giant

ccll granuloma. They occur most frequently in long bones. See picture #96 in booklet

Page 95: Oral Pathology

ORAL PATHOLOGY N-O Tirm

A 21-year-old male patient is home from colleg€ and came to you becaus€ his

"bite seems off " A quick physical assessment seems to indicate thathis chin is deviated to the right. Taking a panoramic x-ray and

comparing to the previous panoramic radiographs you have in hischart, you notice that the l€ft condylar neck seems to have elongated.

What condition does this patient most likely have?

. Condylar agenesis

. Condylar hyperplasia

. Condylar hypoplasia

. Hemifacial microsorria

94

Coplaight aq 2011-:01l - DenitlDecls

ORAL PATHOLOGY N-O Thm

A 6-year-old boy is a patient in your practice who has been hospitalizedmultiple times for broken bones. During routine lab tests, it was noted that

his alkaline phosphatase levels were quite high. Dentally, you notice multipleimpacted teeth. If this patient has a form offibrous dysplasia,

which radiographic feature may you lind?

. The lesions are usually radiolucent, well-circumscribed, and may have a "cotton wool"appearance

. The lesions are usually a saucer-shaped radiolucency

. The lesions are usually radiopaque, not well-circumscribed, and may have a "ground-glass" appearance

. The lesions are usually well-demarcated unilocular or multilocular radiolucencies

95Copyrighr'-C 2011 20ll - Denlal Decks

Page 96: Oral Pathology

Congenital and developmental anon.ralies ofthe temporomandibularjoint, although rela-

tively rare, are important to identiry early to reestablish normal midface growth centers.

The more common entities include condylar agenesis, condylar hypoplasia, and condy-lar hyperplasia.

Condylar agenesis is the absence of all or porlions of the coronoid process, condylarprocess, ramus and mandibular body. Other first and second arch abnormalities are com-monly seen. Early treatment is indicated to limit the degree of deformity, with the pri-mary objective being to re-establish the condylar growth center. This is best done with a

costochondral graft with or without orthodontic surgery and facial plasric augmentation.

Condylar hypoplasia may be congenital, but is usually the result of trauma or infection.The most common facial deformity is shortness of the mandible with deviation of the

chin towards the affected side. Treatment ofthe child involves the placement ofa cos-

tochondral graft. In the adult, treatment involves either shortening of the normal side orlengthening ofthe involved side. Both result in an acceptable cosmetic and functional re-

sult. Onhodontic therapy is necessary in all cases to establish proper occlusion.

Condylar hyperplasia is an idiopathic disease characterized by a progressive, unilateralo\ergro\rth olthe mandible. The chin is deviated towards the unaffected side. Presen-

tation is common in the second decade. Radiographic findings are usually a normal

condlle but an elongated neck- Treatment depends on whether the condyle is still grow-ing. lts growth is occurring, condylectomy is the treatment- If growth has ceased, or-thognathic surgery is performed.

Fibrous dysplasia is an idiopathic condition in which normal mcdullary bone is gradually replaced byan abnormal fibrous connective tissue proliferation. The mesenchymal tissue contains varying amounts

of ostcoid and osseous material that prcsumably arises through mctaplasia.

The discase most commonly presents as an asymptomatic, slow enlargement of thc involved

bcrne. It may involve one bone or several boncs concomitantly.

Forms of fibrous dysplasia:. \Ionostotic: is the most common, comprising 70o% ofcases. There is an equal male to fcmalc ratio,and this fclrm is the most likely to quiesce at puberty. A t!?ical monostotic lcsion will involve thcfemur tibia. or ribs, uith 25% occuring in the bones ofthe skull. See picture #49 in booklet. Pol] ostotic fibrous dysplasia applies to cases in rvhich nrore than one bone exhibits evidcncc ofthcdisorder. It is relatively uncommon; however, many patients have lesions of the skull, facial bones,

t r.ia$ s. as a componcnt ofthe condition. Note: Alkaline phosphatase may bc elevated in up to 30%oipatients $ith polyostotic fibrous dysplasia, and a dramatic dse may herald malignant degeneration.

Calcium and phosphorus tcnd to be normal.

l. Craniofacial librous dysplasia is a form offibrous dysplasia charactcrized by maxillary

\otes- lesions that extend to involve the maxillary sinus, zygoma, sphcnoid bone. and floor oftheorbit.

Itfccune-Albright syndrome is a designation that has been applied to patients withpolyostotic fibrous dysplasia, cutaneous melanotic pigmentation fcafe au lait macules), and

endocrine abnormalities (most commonlr precocious sexual development infenales).3. Jaffe-Lichtenstein syndrome has been used to describe patients with multiple bone le-sions offibrous dysplasia and skin pigmentations.4. Osteoblastoma is an uncommon primary lesion of bone that occasionally arises in the

rnaxilla or mandible. It is a benign process that may exhibit a seemingly rapid onset and

cause pain. The mandible is the most liequent head and neck sitc. Young adult malcs arc

most commonly affected. Radiographically, is a well-circumscribed lesion that varies fiomradiolucent to radiopaque. May have a "sun-ray" paftcm ofnew bone production.

Page 97: Oral Pathology

. In the retromolar pad region

. On the lingual surface ofthe mandible, most often in the premolar region

. On the lingual surface ofthe mandible, inferior to the mylohyoid ridge

. Along the midline ofthe hard palate

96CopriglrtO 20ll-2012 - D€nral D4ks

All of the following are clinical features of the ossifyingfibroms EXCEPT one. Which on€ is the EXCEPTION!

. Slow growing expansile lesion

. More often in maxilla

. Asymptomatic

. Common in young adults around 35 years ofage

97

Cop}.right O 201 l-2012 - De al Decks

Page 98: Oral Pathology

Tori and exostoses are nodular protuberances of mature bone whose precise designationdepends on anatomic location. These lesions are of little clinical significance; they arenon-neoplastic and rarely are a source ofdiscomfon. The precise etiology ofthese lesionsremains obscure, although evidence has been presented to suggest that the torus may be

an inherited condition. Note: The etiology ofexostoses is also unklown. It has been sug-gested that the bony growths represent a reaction to increased or abnormal occlusalstresses ofthe teeth (i.e., bruxism) in the involved areas.

Mandibular tori (also called torus mandibularis) are bony. exophytic growths that occuralong the lingual surface ofthe mandible superior to the mylohyoid ridge.

Vandibular tori may occur singly, however, there is a marked tendency toward trilateraloccurrence, and the lesion is not necessarily confined to the premolar region. Unlikepalatal tori. the mandibular tori are more readily demonstrated radiographically.

forr huoxillary and mandibulor) are ofno pathological significance and rarely are theyerf clinical significance r.vhile the normal teeth are still present. If, howeyer, a complete

denture needs to be made, they should be carefully removed.

*** This is false; these lesions, with rarc cxccptions, arise in tooth-bearing regions ofthciau s. most ofien in the mandibular premolar-molar area. See picture #89 in booklet

Thc ossifling libroma is a bcnign fibro-osseous lesion of the jaws that is considercd bynran\ in\estigators to dcvclop fiom undiffercntiated cells ofthe periodontal ligament. The

:imilaritics between this lesion and thc ccmcntifying fibroma are numerous. Both tumors

occur in similar age groups and locations and manifest comparablc clinical characteristics.

The ossif-ing fibroma is a slow-growing, expansile lesion that is usually asymptomatic whend:scorered. Thcsc lcsions, with rare exceptions, arise in tooth-bcaring regions ofthejaws,most oflen in the mandibular premolar-molar area. They are uncommon lesions that tendro occur in fcmales during the third and fourth decades of life.

The most important radiographic feature ofthis lesion is the well-circumscribed radi-rrluccncy \r'ith a sharply defined border Thcy prcscnt a variable appearance dcpcnding onthc maturation or thc amount olcalcification present. As thc lcsion matures, it eventuallybecomes a relatively uniform radiopaque mass.

The tumor consists ofa collagenous stroma that contains varying numbers ofunifbrm spin-dled or stellate cells. Irregular trabeculae of woven immature bone are most consistentlyrored in th€se tumors. The treatment ofossifying fibroma is most often accomplished by sur-gical removal utilizing curettage or enucleation. Recurrence is uncommon.

.\ r ariant of ossifoing fibroma, thejuvenile ossifying fibroma, has been described in childrenand teens. This rare lesion behaves in a more aggressive fashion than does the ossifliing fi-broma, and it may require more extensive therapy when encountered.

Page 99: Oral Pathology

A l?-year-old patient ofyours comes in for a routine examination. A head andneck examinrtion reveals multiple cysts of the skin. Her panoramic exarns

have always shown multiple impact€d teeth and today shows multipleradiopacities ofthe jaws especially at the angle of the mandible. You

suspect Gardneris syndrome. What complication should she mostbe concerned with wh€n consulting her physician?

. Odontomas

. Osteomas

. Adenocarcinoma

. Epidermoid cysts

. GI polyposis

9E

Copyfl ghr a., 201 1,201 2 - Dental Decks

ORAL PATHOLOGY N-O Tirm

A l0-year-old boy comes with ber mother to the dental office because of apainless swelling ofhis maxilla. Radiographic exam reveals an irregularlyshaped radiopoque mass with a ground glass appearance. No other bulgeshave been noted by the mother. A biopsy reveals fibrous tissue in the bone.

What is the most likely diagnosis?

. Monostotic fibrous dysplasia

. Polyostotic fibrous dysplasia

. Albright's syndrome

. Jaffe syndrome

99Copyrighr,(] 20l l'2011 Dental Decks

Page 100: Oral Pathology

Gardner's syndrome is an autosomal dominant disorder and is characterized by intestinalpolyposis, multiple osteomas, fibromas ofthe skin, epidermal and trichilemrnal cysts, im-pacted pemanent and supernumerary teeth, and odontomas. The most serious complica-tion of Gardner's syndrome is the multiple polyps that affect the large intestine. Theinevitable outcome ofthis disease is invasive colorectal €anc€r.

Clinical features:. Onset early puberty. Polyps ofthe colon ultimately change into adenocarcinoma by the fourth decade oflife. Abnormality ofthe retina ofthe eye. Development olmultiple epidermal cysts usually on face, scalp, and extremities

The oral findings ofGardner's syndrome include:. Vultiple impacted and supernumerary teeth. \4ultiple jaw osteomas which give a "cotton-wool" appearance to the jaws. These

osteomas appear as dense, well-circumscribed radiopacities. Osteomas most olten dev-eiop first rvithin the angle ofthe mandible. \4ultiple odontomas

*** \\rhen Gardner's syndrome is suspected based on oral findings, the patient should

be referred to a gastroenterologist for consultation. Note: Multiple desmoid tumorst.tibt onatosis) and epidermoid cysts ofthe skin are also characteristic of the disease.

Remember: Multiple impacted and supemumerary teeth are also seen in Cleidocranialdl splasia.

Fibrous dysplasia is an idiopathic condition in which normal medulJary bone is gradually rcplaccd byan abnormal fiborus connective tissue prolifcration. The mesenchymal tissuc contains \ arllng amounts

ofosleoid and osseous mateial that presumably arises through metaplasia.

Thc discase most commonly presents as an asymptomatic. slo$' enlargcmcnt of the involvedbone. I! may jnvolve one bone or scvcral bones concomitantly.

. \Ionostotic fibrous dysplasia: is the most common, comprising 707o of cases. There is an equal

male to female ratio, and this form is the most likcly to quiesce at puberty. A typical monostotic les-

ion Nill involve the fcmur, tibia or ribs, with 2570 occu.ring in the bones ofthe skull flle mdiil/a r.r

.atnnnlv involved). A panorex will show a radiopaque mass with inegular bordcrs that has a "groundglass" appearance. See picture #,19 in booklet. Poltostotic fibrous dysplasia applies to cases in which more than one bone exhibits evidence ofthedisLrrder. It is rclatively uncommon; horlcver, many patients have lesions of the skull, facial boncs,

or ii\\ s. as a component ofthe condition. Note: Alkaline phosphatase may be elevated in up to 307o

oipalicnts $ ith polyostotic fibrous dysplasia, and a dramatic risc may herald malignant degeneration.

Cal.rum and phosphorus tcnd to be normal.

Important: Fibrous dysplasia has a variable radiographic appearance that ranges from a radiolucent le-

sron to a densely radiopaquc mass. The classic ptesentation has been describcd as radiopaque with nu-

merc'us bony nabeculae imparting a "ground-glasss" appearance. An important distinguishing featureL)ifibrous dysplasia is the poorly delined radiographic and clinical margins ofthe lesion The process

iecms lo blend into the surrounding normal bone without cvidence of a circumscribcd border.

\lalignant degeneration occurs in less than l7o ofcases offibrous dysplasia. Malignancies are almost ex-

ciusi\,ely osleosarcoma. For unknown reasons, monostotic and craniofacial Iesions have the greatest po-

tential formalignant dcgeneration, and radiation therapy has been found to incrcase the risk by 400-fold.

\ote: The differential diagnosis of fibrous dysplasia of the jaws includes the ossilying fibroma, how-eyer, radiographically thc ossifying fibroma has a well-circumscribed appearsnce.

Remember: N{cCunc-Albright syndrome is a designation that has bccn applied to patients with polyos-

totic fibrous dysplasia, cutaneous melanotic pigmcntation fcate .r lait macules), and endocrine abnor-

rnaTlties (rnost cotwtronly precocious sexual devlopuent in.lbmales).

Page 101: Oral Pathology

Odont CystORAL PATHOLOGY

A new patient walks into your oflice. Your initial physical assessment revealsthat his eyes are set wide and that he has multiple lesions of the skin. Whenshaking his hand, you notice that the skin ofhis palm is very thick and has

palmer pitting. When doing a health history, he reveals that he sees aneurologist and that he has some calci{ied structures "in his brain."

A panoramic radiograph may likely reveal:

. Osteomas

. Odontogenic keratocysts

. Odontomas

. Dentigerous cysts

100

Copynghr i,e 20ll'2011- Denral Decks

ORALPATHOLOGY Odont Cvst

A mother brings her 2-year-old boy into the dental oflice because of a "swelling"on his alveolar ridge. Your exam reveals a smooth-surfaced bluish lesion with

fluctuance where tooth #K will be erupting. The most likely diagnosis ofthis is:

. Dentigerous cyst

. Eruption cyst

. Hematoma

. Hemangioma

10'l

Coprrighl !, 2011-2011 - Denlal Decks

Page 102: Oral Pathology

This patient has nevoid basal cell carcinoma syndrome, which is an autosomal domi-nant disorder characterized by oral, systemic and skeletal anomalies. It is an inheritedgroup ofdefects which involve abnormalities ofthe skin, eyes, nelvous system, endocrine,glands and bones. The condition is characterized by an unusual facial appearance and apredisposition for skin cancer. It is also krown as the basal cell nevus-bifed rib syndrome,the basal cell nevus syndrome and the Gorlin and Goltz syndrome. Note: Recently, mu-tations in the human homologue ofDrosophila patched fPICIl), a tumor suppressor gene

were identified as the underlying genetic event in this syndrome.

Possible abnormalities include:. Cutaneous anomalies: including multiple basal cell carcinomas, other benign der-

mal cysts and tumors, palmer pitting, palmer and planlar keratosis and dennal calcino-sis.. D€ntaf and osseous anomalies: including odontogenic keratocysts (o.ften multiple)mild n.randibular prognathism, rib anomalies (often bifid), and venebral anomalies.. Ophrhalmologic abnormalities: including hypertelorism with wide nasal bridge an

congenital blindness.. Neurologic anomalies: including mental retardation, dural calcification, agenesis ofcorpus callosum and congenital hy&ocephalus.. Sexual abnormalities: including hypogonadism in males and ovarian tumors in fe-males.

Radiographic features:

. Calcification ofthe falx cerebri

. Presence of odontogenic keratocysts

An eruption cyst is essentially a soft tissue variant ofthe dentigerous cyst. It is invariably

associated with an erupting tooth (asaa lly prim^ry but o(casionally a permanent tooth).

The effects are mostly limited to the overlying gingival tissues rather than bone.

Clinically, the lesion usually appears as a smooth-surfaced, reddish-pink or bluish-black,tluctuant. Iocalized swelling on the alveolar ridge over the crown ofan erupting primary

or permanent molar tooth. The intense bluish coloq which is often characteristic, is due

ro an accumulation of blood. Due to this appearance, it may be mistaken for a heman-

gioma or hematoma.

\o treatment is necessary as the cyst often ruptures spontaneously. In a few rare cases,

incision or even the removal ofthe overlying tissue may be necessitated by pain or ten-

demess associated with lhe lesion.

Page 103: Oral Pathology

Odont Cvst

Upon viewing a panorex ofa l4-year-old patient, you see a well-de{inedmultilocular radiolucency with scalloping around the roots on the left side

ofthe mandible rpical to the canine and first premolrr. No clinical symptomsare present. Teeth are not carious and respond normally to vitality tests.M€dical history is unremarkable. Upon opening into the area, no fluid or

tissue is evident. What is the most probable diagnosis?

. Dentigerous cyst

. Traumatic (simple) bone cyst

. Primordial cyst

. Residual cyst

. Stalne /s/a/i., bone defect

102Copyflghr O 20ll 201: Dental Decks

ORAL PATHOLOGY Odont Cvst

A healthy l9-year-old patient presents to your olTice for a routine exam. Takinga panoramic radiograph, you see a well-corticated, unilocular radiolucencysurrounding the crown of impacted tooth #17. The lesion is asymptomatic.

What is the most likely diagnosis?

. OKC

. Dentigerous cyst

. Cystic ameloblastoma

. Central ossifying fibroma

103

Copyaghr,e 20ll :01: Denral Decks

Page 104: Oral Pathology

Remember: This cyst may be completely devoid ofsolid or liquid material. lt occurc most fre-quently in younger persons rlith no sex predilection. The usual location is in the mandible bc-tween the canine and ramus. The regional teeth are vital.

Clinical features:

. Children and adolescents. usually belbre the age of20

. Usually asynlptomatic

. Prinarily seen in mandiblc

. Slight ilun) conieal bonc crpansion

. Associated with vital teeth, no displaced teeth

The dentigerous cyst (or lbllicular c.vst) contains a crown ofan unerupted tooth or denlal anonr-alv such as an odontoma. Enlarged dentigerous cysls can cause Drarked displaccmcnt oftccth. Prcs-

sure of accumulatcd fluid usually displaces the tooth in an apical direction.See picture #50 in booklet

The primordial cyst diflers from the periodontal and dentigerous cysts in that it conlains no cal-crlied structures. These cysts are lined by stratified squanous epithelium and may be either uniloc-ular. multilocular or multiple.

The residual cyst rcl'ers to a situation in which a tooth associated with a radicular cyst is cxtractcdbut the c)st is lcft undisturbed, it persists within thejaw and this lcsion is called a residual cyst.

\ote: \bu must curette the socket ofa tooth with a radicular cyst aftcr extraction. See pictures #86 and #87 in booklet

Ihe st^Ine (stqtic) bone defect is radiolucency ofthe mandible due to invagination ofthc lingual.urlace of the of the jaw. lt is locatcd in thc posterior mandible below the mandibular canal.

\ote: Thc focal osteoporotic bone marrow defect is a radiolucency in the jaw that conlainshenratopoietic bonc marrow. 11 is olten seen in an extraction site.

Clinical features:.lt usually contains a crown ofan unerupted tooth. Usually not clinically visible rvithout radiographs. .\\\ nrptomalic, occasionally pain or swclling. Lsually involve uneruptcd mandibular third molars. olher frequenl sites includc ma\illary canines.

marillary third molars and mandibular second premolars

Rrdiographic feature: \\'ell-circumscribed, unilocular radiolucency around crown ol'looth. See picture

=50 in booklet

Remember:

I The lateral periodontal cyst may bc dcfincd as a non-kciatinized, non-inflammatory dc-\ clopmcntal cyst occurring adjacent o. lateral to thc root ofa tooth. See picture #91 in booklet

:. The fissural cysts ltthich are also called developmental .,l.irtr) are non-dental in oiigin, thcy in-

clude nasoah colar. median palatal, and nasopalatine cysis.

-1. Th!- primordial cyst contains no calcified structures.

-1. Thc traumatic (simple) bone cyst n].ay contain blood, 11uid, dcbris or be completely empt-l.',

Comrnonly found in young persons, in the mandible between the caninc and ramus.

The residual cyst is often found in edentulous areas. This cyst refers to a situation in which a tooth* ith a radicuiar cyst associated with it was extracted. and the socket wasn't curelted. The radicu-lar clst persists in thejaw as a residual cyst.

The gingival cyst ofthe newborn has also been designaled as the dental lamina cyst ofthe new-born or Bohn's noduels. Such cysts appear typically as multiple nodulcs along the alveolar ridgein neonales. They are due to cystification ofrcsts ofthe dcntal lamina. In the vast majority ofcascs.these cvsts degeneratc and involute or rupture into the oral cavity. Note: Similar epithelial inclu-sional cysts may occur along the midline ofthe palate fpala/i/te q'sts ofthe nerborn or Epstein's

2earl9. Thcse are ofdevelopmental origin but are not dcrivcd from odontogenic cpithelium. Notreatlnent is necessarv.

Page 105: Oral Pathology

. Henwig's epithelial root sheath

. The reduced enamel epithelium

. Remnants ofthe dental lamina

. A pre-existing osteoma

. Residual cyst

. Traumatic bone cyst

. Primordial cyst

. Periodontal cyst

10,t

CopFiShr O 201 I '2012 - Dnral Decks

105

Copynght O 201l-2012 - Denhl Deck

Page 106: Oral Pathology

Keratocysts dillcr fiom olher odonlogenic cysts in their nricroscopic appearancc and clinical be-havior They may resemblc periodontal, primordial or follicular cysts. Usually they cannot be dis-tinguished radiographically. See picture #88 in booklet

Clinical features:. Widc age range. peak occurrence in 2nd and 3rd decades. [,csions found in children are often reflective of nrultiple odontogenic keratocysts as a com-ponenl ol'the nevoid basal cell carcinoma syndrome. More common in males than fcmalcs. The chrefsite ofinvolvcmcnt is thc mandible, in approximatcly a 2 to 1 ratio. In thc nrandiblc, most occur within the posterior portion ofthe body and rarnus rcgion. Typically asymptomatic

Radiographic features:. well-demarcated area ofradioluccncy with a sca)lopcd, radiopaqr,re margin. Unilocular or multilocular

\licroscopicallyi. The lininging epithclium is thin and parakcratinizcd. The basal layer is palisadcd u,ith prominent, polarized, and intensely staining nuclei ofuniformdramctcr. The luurcn may contain large amounts ofkeratin debris or clear fluid similar to serum transu-

dalc. The parakeratotic type forms 85 10 95% ofall odontogenic keratocysts; thc balance is made

op of the .'rthoker.ltinized vaiant. lmportant: The orthokeratinzed variant is less aggressive,

\\ ith a rnuch lorver rate ofrccurrence and is not syndrome-associatcd.

Important: The most remarkable feature of keratocysts is their great tendency toward recur-rence.

A primordial cvst arises from cystic changes in a developing tooth bud before the for-mation ofenamel and dentin matrix. Since the primordial cyst arises from a tooth bud, thetooth \\'ill be missing from the dental arch unless the cyst arose from the tooth bud of asupenrumerary tooth. The mandibular third and fourth molar regions are the mostcommon locations for a primordial cyst. lt is usually found in children and young adultsbetri .-en l0 ard 30 years of age. Radiographically, the primordial cyst is a circular ra-drtrlucencv * ith a radiopaque border and lound at the site where the tooth failed to de-\elrrp. \lany investigators have reported that most prirrordial cysts have the sameaharacteristic l'eatures as those of odontogenic keratocysts. However, until conclusiveprtrtri is established, primordial cysts and odontogenic keratocysts are considered separateiit trtles.

\\ h.n a tooth having a radicular cyst at its apex is extracted, the radicular cyst is leftbehind in bone and is now called a residual cyst. A residual cyst can also rise from rem-n3nts ofthe epithelial rests after tlre extraction ofa tooth. This cyst occurs in older indi-r iduals. the average age is 50 years. The radiographic appearance is that of a circularradiolucency sunounded by a radiopaque border and occurring in an edentulous area. Aresidual cyst can easily be misdiagnosed as a primordial cyst. The latter arises in lieu ofa looth r\hereas a residual cyst arises in relation to an extracted tooth.

Page 107: Oral Pathology

. Lateral periodontal cyst

. Dentigerous cyst

. Odontogenic keratocyst

. Based on symptoms

. Radiographically

. Histologically

. By an electric pulp tester

106

CoplriSht O 201 1,201 2 - Dental Decks

107

Copright O2011,2012 - Denral Decls

Page 108: Oral Pathology

Cyst Clinical Characteristics Radiogmphic Chrracteristics

Dendgerous

{follicular cyt). Children and teenagers. Mandibular third molar and maxillary canine

area 00%o) in nandible). Associated with impact or unerupted teeth. Sccond most common odontogenic cyst

. Well-defined usuallyunilocular RL associated withthe crown of !n uneruptedtooth

Odontogenick€mtocysl

Usually occurs between the ages of 10-30Often associated with an impacted tooth50o/o mandibular third molar areaOver 307o recurrence rate

. Well-circumscribed RL withsmooth margins lnd thinradiopaque borden

LateralPeriodontal cyst

95% mandibular canine-premolar areaApposition with rooi ofvifal toothUsually symptomless

. Well-defined, round orteardropshaped RL with anopaque margin along lateralsurface of tooth

L The lateral Dcriodontal cvst mav be deflncd as a non-keratinizcd. non-inflammatorv de-

\otcs, velopmental cyst occurring adjacent or latenl to the root ofa tooth.2. Thc gingival cyst ofadulthood is the sofl-tissue oounterpart ofthis lesion.

3. The origin ofthe lateral periodontal cyst is related to proliferation ofthc rests ofdentallamina.4. The calcifying odontogelic cyst (COC./ is believed to be dcrived from odontogenic ep-

ithelial remnants within thc gingiva or within thc mandible or maxilla. Ghost cell kera-tinization is the characteristic microscopic fe.ture of the COC. Radiographical)y, the

COC may prescnt as a unilocular or multilocular radiolucency with discrctc. wcll-demar-cated margins. within the radiolucency, ghost cells may undergo calcification and appear as

a "salt and peppcr" typc of pattcm. Note: This cyst has a cutaneous countcrpart known as

calcifu ing epithelioma of Malherbe or pilomatrixoma.5. The glandular odontogenic cyst, or thc sialo-odontogenic cyst is rare cyst that occurs

most often within the mandible. This lesion is locallv asgressivc and recurence is com-mon.

LesionUsual

LocationClinical and

Radiogriphic Characteristics Treatment

Dental gaanuloma(h is one ofthe moslcoamon ofall sequelaeo{p lpiti,

Apex oftooth . Asymptomatic. Circumscribed radiolucency at

apex of tooth. Tooth is non-vital. May be sensitive to percussion

Root canal rcatmentor extraction ofinvolved tooth

Radicula. cystl..llso called apicalperiodontal & periapicalfls, It is the most commonodontogenic cyst

Apex oftooth . Asymptomatic. Circumscdbed ndiolucency at

apex of toothr Tooth i: non-vital. May be sensitrve to percussion

Root canal treatmentwith apicoectomy orextraction wilhcurettage ofsocket

\ecrotic pulp causes periapical inflammation:. lt acute. a periaprcal abscess forms. If chronic, a dental granuloma forms

\ote: This inflammatory cyst derives its epithelial lining from prolitbration of small odon-

rogenic epithelial residres (rest of Malcssez) within the periodontal ligament.

Page 109: Oral Pathology

ORAL PATHOLOGY Odont T[rm

A 37-year-old patient comes into your office with the complaint of a slowly-growing, painless swelling of his lower left jaw. A panoramic shows a

multilocular radiolucency with well delined and sclerotic margins alongthe left mandibular molar-ramus area. All teeth test as vital. A triopsy isconducted and shows odontogenic epithelium. What is the most likely

diagnosis for the most aggressive type of this turnor?

. SoIid (multicystic or pol!"cystic)

. Unicystic

. Extraosseous (p e rip heral)

108

Copyrighr'a 201 l-2011 - Dental Decks

ORAL PATHOLOGY Odont T[m

A 30-year-old patient com€s into your oflice complaining ofa painless swellingof his lower left jaw. A panoramic radiograph shows a welltircumscribedmultilocular radiolucency with a 'honeycombed" pattern at the locationofthe lower left molars. The teeth have been displaced. The pathology

report calls this an odontogenic mlroma, This tumor:

. Is composed of large polyhedral, neoplastic, epithelial cells

. Is composed ofneoplastic epithelium and mesenchyme

. Arises from follicular connective tissue resembling pulp tissue

. ls composed of spindle-shaped mesenchymal cells and aggregates of multinucleatedgiant cells

109

coplrighr ill 20ll-201: - Denral Decks

Page 110: Oral Pathology

Ameloblastoma consisrs entirely ofodontogenic epithelium, which at sites shows the differentia-tion ofthc familiar, histologic layers ofthe enamel org.n. It is most often seen in the mandibularmolar-ramus area. Clinically, it is the most aggressive odontogenic tumor. Important: The ameloblas-

toma is the most common epithelial odontogenic tumor.

Clinical features:. No sex or racial predilection. Mean age is commonly between 35 and 45 years. 2 biologic-microscopic subtypes:

I ) Solid or multicystic: is more aggressive and high recunencc rate if treatcd consen'ativcly.2) Unicystic: is less aggessive and less likely to recu. A histologic variant ofthis type is the plex-iform unicystic ameloblastoma.

. Slow growing and painlcss swclling

. Locally aggrcssivc

. Capable ofcausing large facial deformities

. Teeth are vital

\oter Rarely, extraosseous peripheral ameloblastomas arc found in tlre gingiva and buccal mucosa.

The) exhibit a benign non-aggressive course.

Radiographic characteristics: Multilocular or unilocular radioluccncy with well defincd and slerotic

margins. Notei In the mandible it appears similar to thc central giant cell granulona. See picture #51

in booklet

Treatment: Each casc should be considered on its own basis. The solid hnullicristic or polvcl'stic) le-.ion requires surgical excision. Rescction should be reserved for larger lesions. Unicystic Iesions usualh require only enucleation. Thcy should not bc ovcr trcatcd. Note: You nccd to know thc.rmeloblastoma ertremely well for thc cxam!l!

\olei }lalignant behavior by amcloblastoma is rarely encountered. These lesions occur in a younger age

group (JOr/ and appcar in the mandible more frequently than the maxilla. Malignant lesions have been

ll\ rded into nvo subtvpes: the malignant ameloblastoma and ameloblastic carcinoma.

The odontogenic myxoma is mesenchymal in nature and origin. mimicking microscop-icall! the dental pulp or follicular connective tissue. The mean age in which this lesion ap-

pears is J0. Radiographically. this lesion is always radiolucent, although the pattem

mav be quite variable. It may appear as a well-circumscribed lesion or as a diffuse lesion.

h rs often multilocular. frequently with a "honeycombed" pattem- Important: Cortical ex-

ltnsion (rother than perforation) and root displacement (rqther lhan resorplion) are the

rule. Surgical excision is the treatment ofchoice. These lesions exhibit some aggressive-

ness and have a moderate recurrence rate. See picture #90 in booklet

\ote: \\:hen relatively large amounts ofcollagen are evident, the term "myofibroma" is

used to designate the odontogenic myxoma.

Ihe central odontogenic fibroma is a rare lesion that is regarded as the central counter-

part to the peripheral odontogenic fibroma. It is seen in all age groups, and is found in both

rhe n.randible and maxilla. It appears as a well-defined radiolucent lesion that is usuallymultilocular, often causing cortical expansion. It is treated with surgical excision and re-

currence is very uncommon.

l. The calcifying epithelial odontogenic. (pindborg) ttrmor is composed oflarge\otcr polyhedral,neoplastic,epithelialcells.. t,,.,.r,rr' 2. The ameloblastic fibro-odontoma and ameloblastic fibroma are composed

of neoplastic epithelium and mesenchyme.3.The central giant cell granuloma is composed ofa proliferation ofspindledfibroblasts in a stroma containing variable amounts ofcollagen. Multinucl€atedgiant cells are present throughout the connective tissue stroma.

Page 111: Oral Pathology

. In the mandible than in the maxilla. and more often Dosterior than in the anteriorreglons

. In the mandible than in the maxilla, and more often anterior than in the posteriorreglons

. In the maxilla than in the mandible. and more often Dosterior than in the anteriorregions

. In the maxilla than in the mandible, and more often anterior than in the posteriorregl0ns

'fi0Coplrighr O 201 1-201 2 - D€nral Decks

. A predilection for persons younger than 20 years old

. A predilection for middle-aged caucasian men

. A predilection for middle-aged black women

. No age, racial or sex predilection

111

Coptrighr e 201 l '2012 - D€ntal Decks

Page 112: Oral Pathology

The cementoblastorna, also known as the true cementoma, is a rare benign neoplasm ofcementoblast origin. It occurs predominantly in the second and third decades, typically be-fore 25 years ofage. It is more often seen in the mandible than in the maxilla, and moreoften posterior than in the anterior regions. It is intimately associated with the root of a

tooth, and the tooth remains vital. It may cause cortical expansion and, occasionally, low-grade intermittent pain. Radiographically, this is a well-circumscribed radiopaque lesionthat replaces the root ofthe tooth. It is usually surrounded by a radiolucent ring. Because

ofthe intimate association ofthis lesion with the tooth root, this lesion cannot be removedwithout sacrificing the tooth. There is no recurrence.

Note: To distinguish this lesion from condensing osteitis - in CO you can distinguish

the root outline.

This lesion also known as periapical cemento-osseous dysplasta represents a reactlve

,uth". thun a neoplastic proiess. This lesion appears to be an unusual response of peri-

apical bone to some local factor This is a relatively common phenomenon that occurs aI

the apex ofvital teeth. Women, especially black women' are affected more than are men'

iiopp"urc in *iOat e age (around i0 ,earsl and the mandible' especially the anterior pe-

riapical region, is farrnor" .o.-only uffected than is the maxilla More often' the apices

of trvo or r.riore teeth are affected. There are no symptoms and the teeth are vital' As the

condition progresses or matures, the lucent lesion develops into a mixed or mottled pat-

i"* "ttl.g

,. i"t. repair The final stage appears as a solid opaque mass often surrounded

by a thin iucent ring. No treatment is required for this condition'

\ote: A rare condition called florid osseous dys plasit (FoD) appears to be an exuber-

ant fonrl ofperiapical cemental dysplasia'

The cementiffing fibroma occurs chiefly in adults around the age of 40 There are

fr.iii".tio* ior ine uody of the mandible antl for females' The Iesion may cause tooth

rr.ro. ement or cortical expansion. Radiographically, it is well-circumscribed and may ap-

pea, ..tatively radiolucent, lucent with opaque foci, or diffusely opaque lt is treated with

cu..etag. o. e^cirion. Recurrence is very rare Note: This lesion can be considered sim-

ilar or identical to the ossifying fibroma'

Page 113: Oral Pathology

. TuberosibT area

. Maxillary anterior area

. Mandibular premolar area

. Molar - ramus area

112

Coplright O 20ll-2012 Denral Decks

\Th€ smeloblrstic libromr and ameloblastic librordontoma eppeer to bevariations of tbe same process. These neoplasms occur predominantly in:

s-,

. Adults with a mean age of 40

. Elderly people with a mean age of 75

. Young adults with a mean age of25

. Children and young adults

.113

Copyright O2011,2012 - Dentat Deck

Page 114: Oral Pathology

The mandible is affected twice as often as is the maxilla. There is a predilection for the

molar-ramus region. although any site may be affected.

Note: A unique microscopic pattem typifies the CEOT - sheets of large polygonal ep-

ithelial cells are usually seen v,ith areas of amyloid that have concentric calcified deposits( [, ie s e g an g ri ngs ) tl]roughout.

E\cept for the presence of an odontoma, the 1wo lesions are tlte same. The mean age is

lround 12 years, and the upper age limit rnay extend as high as 40 years. The mandibu-iar molar-ramus area is the favored location for the two lesions. Radiographically, these

lesions are rvell circumscribed and are usually surounded by a sclerotic margin. Theymal be either unilocular or multilocular and may be associated wiih the crown of an im-pacred toofi. Important: An opaque focus appears within the ameloblastic fibro-odon-tona o\\ ing to the presence ofan odontoma. This lesion therefore presents as a cornbined

lucent-opaque lesion; the ameloblastic fibroma is completely lucent radiographically.

\ote: These lesions are treated conservatively by curettage or excision. Recurrence is

rare.

The other tumor of mixed, (epithelial and mesenchynal) origin is the odontoma. These

;alcilied iesions take one or two general configurations. They may appear as multipleminiature or rudimentary teeth, in which case they are known as compound odon-tomas, or they may appear as amorphous conglomerations of hard tissue, in whichcase thel are known as complex odontomas. As a group they are the most common odon-

togenic tumors.

Teens; females:anteriorjaws; inassociation wilh thecrowns ofimpactedteeth

Well-circumscribedunilocular RL lesion; mayhave small opaque foci

Resection ofaffcctcdarea; recurrence commonif inadcquately treated

Adults f,/0 l"dr'timolar-ramus region;

law expansron

Unilocular or multilocularRL lesior! may have opaquefoci

Adolt qhears);anterior maxilla andposterior mandible;no symptoms

Well-circumscribed, oft enscmilunar RL lcsionassociated with the roots ofteeth

Resection of affectedareai locally aggrcssivc;metastases lo lung andregional lymph nodes

Females over 60;mandible and maxilla

Page 115: Oral Pathology

How would you refer to the group of small radiopacities between thernandibular canine and first premolar on the periapical x-ray below?

'114

Cop)rrghr ae :0ll :012 Dertal Decks

ORAL PATHOLOGY

A 4s-year-old African Amerlcan female presents to your oflice for a routineexam. Periapicals of the mandibular incisors show multiple radioopacitieswith radiolucent rims. Teeth #23 through #26 test as vital. There is no pain

on percussion or palpation. Tfeatment for these lesions should be:

. Do nothing (obserue)

. RCT treatment lor teeth #23 through #26

. Surgical excision of lesions

. None ofthe above

115

Cop)'right C 201 l-2011 - Dental Decks

Page 116: Oral Pathology

Tumoa Clinical l'eaturesRadiogrxphic

Charactedslics\{icroscopic

CharacteristicsTreatment

aud Prcgnosis

Complex Children and young adultsitendency to occur in theposreriofja\r's

Amorphous opaquemasses iYpically in a

tooth-beanng ar€a, be-tween roots or over tne

impacted tooth

Nomlal appealngenamel. dcntin. ce-

mentum. and pulpmay be sccn.

Enuclealion;

Compound Children and young adults;tendency to occur in thcanteriorjaws

Multiple small tooth-llke structurestypically in a tooth-bearing area, berweenroots or over the qownofan impacted tooth

Normal-appearingenamcl. dentin.cementum. and PulPmay bc seen.

Enucleation;docs not rccur

Remember: A compound odontoma looks like a tooth, and a complex odontoma does

not lit is a disorganized arrangement oJ tubular dentin, enqnrel' and thin la1'ers of ce-

nemunl. See picture #52 in booklet for picture ofcomplex odontoma

l.'ote: A rare variant known as ameloblastic ondontoma has been described. This is es-

sentially an ameloblastoma in which there is focal differentiation into an odontoma.

Periapical cemental dysplasia, also known as periapical cemento-osseous dysplasia (and

lornerly knov'n as cementoma) represents a reactive rather than a neoplastic process. Thetenr "cernentoma" is a misnomer as thc opacities are not cementum but bone. While theyappear lo arise lrom the teeth, the lesions apparently arise within the bone instead. It ap-pears to be an unusual response ofthe periapical bone to some local factor (br example,

It dunntic occIusion or infbction).

Clinical features:. Occurs at the apex ofvital teeth. A pre dilection fbr middle-aged black women. As] mptomatic, usually multiple, small periapical areas ofradiolucency in the mandibu-lar incisorarea. See picture #53 in booklet

\ote: Depending on stage, it may appear mixed radiolucent and radiopaque or totally ra-

dropaque.

Important: Age. gender, Iocation, radiographic appearance, and tooth vitality consid-ered together are diagnostic of this condition.

Three stages:L Osteolytic stage; radiolucency appears on radiograph.l. Cementoblastic stage: beginning of calcification in the radiolucent area (mixed ra-diolucent and rqdiopaque appearance).3. llature stage: radiopacity appears on radiograph with a thin radiolucent line aroundarea.

\ote: No treatment is required for cementomas. Once this stage is reached, the lesion sta-bilizes and causes no complications.

Page 117: Oral Pathology

. ORALPATHOLOGY PigLesofOralCav

A S-year-old boy presents with his mother for his lirst dental exam.Your exam reveals a normally developing dentition but you notice

multiple "freckles" on his lower lip and on the buccal mucosa.What condition should vou be concerned about?

. Gorlin-goltz syndrome

. Gardner's syndrome

. Peutz-Jeghers syndrome

. Cleidocranial dysplasia

115

Coplriglrr o 20ll-l0l: - Denral Decks

ORAL PATHOLOGY Pig Les of Oral Cav

What most likely caused the discoloration of thegingiva as seen in the picture b€low?

117

Copyrighr C 20ll'l0l: Denral Decks

Page 118: Oral Pathology

Peutz-Jeghers syndrome (also called hereditat! itesti al polyposis syndrome) is an unusual

condition which is ofinterest to the dentist because of the oral findings. The pigmentations

usually appear at an early age, often during the first decade of life and at this time are re-

stricted to the oral region. Intraorally, these pigmentations may be located anywhere on the

mucosa, but are most frequently seen on the buccd mucosa, gingiva and hard palate. Themucosal surlace ofthe lower lip is almost invariably involved. These spots or macules, whileof variable intensity. may mnge through shades of brown, blue and black. During the suc-

ceeding decades of the patient's life, pigmentations may arise elsewhere on the skin, espe-

cially on the extremities. In addition, intestinal polyposis is seen, most commonly in the

small intestine (jejunum) and may produce signs and symptoms of abdominal pain, rectal

bleeding, and diarrhea. It should be noted that the pigmentations ofPeutz-Jeghers syn-

drome may occur without demonstrable evidence of polyps and, also that multiple pollpsmay be encountered without pigmentations. See picture #35 in booklet

Important: It is significant to note that although the oral pigmentations per se are harm-less. their presence is important in that they indicate a necessity for investigating the pos-

sible presence of multiple polyposis which may prove harmful. There is a strong tendency

for these multiple polyps of the colon to undergo malignant chang€.

\ote: It appears to be caused by a mutation ofa gene known as LKBI ofchromosomeI 9 that encodes a multifunctional serine-threonine kinase.

.\n amalgam trttoo is the most common oral pigmented lcsion The tattoo has been mistaken for a

mclanin-pigmented lesion. The most common looations for amalgam tattoos are the gingiva. buccal mu-

cosa and alveolar mucosa,

Trauma and chemicals are frequent causes oforal lesions:

. '\n aspirin burn occurs when patients placc the tablet against an aching tooth, allowing the check

or lip to hold it in position while it dissolves slowly. Within a few minutes a buming sensation ofthcmucosa will bc noted and thc surface becomes blanched or whitened in appearancc The caustic ac-

rion ofthe drug causes necrosis ofthe oral mucosa, with subsequent sloughing ofthe necrotic epiih-

eliurn. \ote: Aftempt to locate amalgam on x_ray if tattoo is suspcctcd; ifyou can not locate any

amalgam. a biopsy may be needed to rule out a melanocytic neoplasm See pictur€ #54 in bookletRemember: Chcmical bums may also be caused by hydrogen peroxidc. silver njtratc, phcnol' etc.. The use ofthe heary mctal bismuth is still common in treating certain dermatologic disorders as well

as larious other diseases. Bismuth pigmcntation appears as a "bismuth line," a thin' blue-black linein the marginal gingiva which is sometimes confined to the gingival papilla Note: Lead ingestion also

l.avcs dark marks on the gingiva.. Traumatic ulcer: very common; may mimic oral cancer and chronic infectious ulcers. Fo.al (frictionat) keratosisi common whitc lesion caused by chronic friction on the mucosa.

Important: Differcntiated from idiopathic leukoplakia becausc cause is known. Linea alba: a type offrictional keratosis that appcars as a linear whitc line in buccal mucosa. Smoking-associated melanosis: caused by a chemical in tobacco smoke that stimulates melanin pro-

duction. Note: If thc pigmentation is localized, an ulceration is present or thc lesion is elevalcd, a

biops.v is necessary to exclude other pigmented conditions feg, nevi, melanoma). Although smoker's

mclanosis is an abnormal deposition ofmclanin, the lesion itsclf is not associated with an increascd

risk ofmclanoma or carcinoma.. Nlelanotic macufei most common mclanocytic lesion. Gingiva a commonlocatton (called an orulnelonotic macule - See picture #55 in booklet/ and the lips, most frequently on the lower lip (called

a labial melanotic macrle). Can be associated with Peutz-Jegher's syndromc. Drug-induced pigmentation: Most commonly associated with minocycline' chloroquine' cyclo-

phosphamide, and azidothymidine (AZT).

Page 119: Oral Pathology

. Buccal mucosa

. Tongue

. Hard palate

. Alveolar mucosa

. Addison's disease

. Albright's syndrome

. Cushing's syndrome

. Peutz-Jeghers syndrome

118

Copright O 201l-201: - Dental Decks

't19

CoD,right C 201 I -20 12 - Denral Decks

All ofthe following conditions demonstrate pigmentation of th€ intraoralmucous membranes .EXCfPl one. Which one is the EXCEPTION2

Page 120: Oral Pathology

Moles (nevi) are small, usually dark, skin growths that develop frotn pigmenlproducingcells in the skin (called melanocltes). While nevi are fairly common on the skin, intrao-rally they are quite uncommon. When present, they are usually on the hard palate, butcan also be seen on the gingiva and lips. Congenital nevi (commonlt knou,r as birth-marks) are usually large (greqter thqn I 0 cm) and with the passage of time, may change

ftom flat, pale tan macules to elevated, vemrcous, hairy lesions. Approximately 150/o occur

on the skin ofthe head and neck. Congenital nevi have a higher incidence of malignanttransformation (as opposed to acquiretl nevi).

Acquired nevi are microscopically classified into five subtypes:

l. Intramucosal nevus: most common in oral cavity2. Blue nevus3. Compound nevus: rare in oral cavity,1. Junctional nevus: rare in oral cavity5.Intradermal nevus: is the most common lesion of skin. known as the common mole

\ot€: Acquired nevi are much more common than congenital nevi both intraorally andertraorally.

Important: The B-K mole syndrome and the dysplastic nevus syndrome are both char-

acterized by having numerous large, pigrrented atypical nevi which have a high risk forder eloping malignant melanoma.

Addison's disease (qlso called chronic adrcnocortical itsuflicienq,) results from hypofunc-

tion ofthe adrenal cortex. lt is chamcterized by bronzing ofthe entire skin. Oral signs consist

of diffuse pigmentation ofthe gingiva, tongue, hard palate and buccal mucosa. Although cu-

raneous pigmentation will most likely disappear following therapy, pigmentation ofthe oraltissues tends to persist.

-{lbright's s1'ndrome (also celled McCune-Albright syndronre) is a severe form ofpollostotic fibrous dysplasia, involving nearly all bones in the skeleton. In addition tothe bone lesions there are brown patches ofcutaneous pigmentation (called caf6-au-laitspots, and endocrine dysfunction, especially precocious puberty in girls.Important: There is an increased incidence of osteosarcoma seen with polyostotic fi-brous dysplasia.

Peutz-Jeghers syndrome (also known as Hereditary Intestinal Pol!-posis Syndrome) rs

an inherited disorder that is characterized by having multiple intestinal polyps and in-rraoral melanin pigmentations. These pigmentations usually appear during the firstdecade of life and at this time are restricted to the oral region. They may be located any-u here on the mucosa, but are most ftequently seen on the buccal mucosa, gingiva and

hard palate. The mucosal surface of the lower lip is almost invariably involved.

\ote: Cushing's syndrome is a hormonal disorder caused by prolonged exposure ofthebody's tissues to high levels ofthe hormone cortisol. It is relatively rare and most com-monly affects adults aged 20-50. The symptoms vary but most people have upper bodyobesity, rounded face, increased fat around the neck ("buflblo hunp"), and thinning arms

and legs.

Page 121: Oral Pathology

rA 40 year otd patient presents to his physician with complaints of muscleA /ru yeer oro pauena pnesenrs ro nrs pnysrcran wrtn comptaints ol musce

, ' werkness and loss of appetite. II€ has noticed a loss of weight and also t[rthis skin hss started to "bronze,' His labs show lowered blood glucose and

sodlum and an increased potasslum. One condition likely to be crusing this is:

. Peutz-Jeghers syndrome

. Cushing's syndrome

. Addison's disease

. Albright's syndrome

. Surgical excision is required

. Radiation is required

. No treatment is necessary

. Antibiotics are required

120CoplriShr @ 201 1,2012 - Dental Decks

121

Copliight C 20ll-2012 - Dental Dects

Focal melanosis is a common circumstanc€ in which brownish areasof pigmentation occur in the oral cayity; once properly diagnosed:

Page 122: Oral Pathology

Addison's disease occurs when the adrenal glands do not produce enough cortisol /a g/ri-cocorticoid). Cortisol's most important function is to help the body respond to stress. The

failure to produce adequate levels ofcortisol can occur for different reasons. The problcmmay be due to a disorder ofthe adrenal glands themselves (prinary adrcnal irrsulJiciencv)or to inadequate secretion ofACTH by the pituitary gland f.secondary adrcnal insullicienc,").The symptoms ofAddison's disease usually begin gradually.

These include:. Muscle weakness. Loss ofappctitc. Weight loss. Skin changes with areas of hyperpigmentation covering cxposed and nonexposedparts ofthe body. This darkcning ofthc skin is most visiblc on scars, skin folds, pressure

points such as elbows, knccs, knucklcs, and tocs as wcll as thc oral mucous membranes

-there is diffuse pigmentation ofthe gingiva, tongue, hard palate. and buccal mucosa.

. Nausea, vomiting and diarrhea

. Low blood pressure*** Clinical features do not begin to appear until at least 9070 of glandular tissuc has

been destroyed.

Laboratory tests show:. Lou'blood concentrations ofsodium and glucose. Increased serum potassium. Dccreascd urinary output ofcertain steroids

Important: The main concern when performing dental procedurcs on a patient with Ad-djson's diseasc is that the adrenal cortex has no capacity to put out cxtra cortisol as a re-sponse to strcss.

Focal melanosis is a disorder of increased melanin pigmentation that develops withoutpreceding inflammatory disease. lt is a condition characterized by abnormal deposits ofnelanrn (especially in the skin).

f tiologl': DevelopmentalLocation(s): Any mucosal site; gingiva a comrnon location (cqlled an oral melqnoticnracrile

-See picture #55 in booklet) and the lips, most frequently on the lower lip

rc?lled a labial melanotic macule)Clinical Features: Brownish coloration to the oral mucous rlembraneRadiographic Features: None\Iicroscopic Features: Melanin pigment within rnelanocytesComplications: NoneTreatment: NonePrognosis: ExcellentPathogenesis: Increased numbers olmelanocytes locally

Important: Melanosis may be observed in adrenal insufficiency (Addison s disease),-{CTH-producing tumors, malignant tumors metastatic to the pituitary gland, or metasta-tic malisnant melanoma.

Page 123: Oral Pathology

. Intradermal news

. Compound nevus

. Junctional news

. Blue news

. Intramucosal nevus

122Coplright O 20ll-2012, Dental Deck

. Well-defined unilocular or multilocular radiolucency with scalloping around the roots

. Not well-circumscribed radiopaque lesion which may have a "ground-glass" appear-ance

. Saucer-shaped radiopaque lesion

. Not well-defined multilocular radiolucency with a "pear-shaped" appearance betweenthe maxillary central incisors

123CopJright O 20ll-2012 - Dental Decks

Page 124: Oral Pathology

Most pigmented skin tumors are composed of ner.us cells and are a result of a develop-mental anomaly ofmelanocytes; they are rare in the oral cavity. The initial, flat, raised

lesion can become nodular, with an increase in consistency. Spontaneous involution mayoccur and malignant transformation is a rare complication. When found intraorally theymost frequently occur on the hard palate (See picture #58 in booklet) but can also be

seen on the gingiva and lips. lntramucosal nevi are the most common variety found in the

oral cavity followed by blue nevi. Compound and junctional nevi are very rare.

Subtypes of Acquired Nevi:.Intradermal nevus: most common lesion olskin. Known as the common mole.Nevus cells lie exclusively within the dermis. See picture #57 in booklet. Junctional nevus: nevus cells are located at the inteface between the epithelium andlamina propria. They are flat and not detected by palpation. Some regard as premal-ignant, may undergo transformation into malignant melanoma.See picture #59 in booklet. Compound nevus: nevus cells are located at the epithelium/lamina propria interface

and also deep in the dermis. They are raised and solid. See picture #60 in booklet. Blue nevus: congenital, painless; color based on the deep cutaneous or subcutaneous/submucosal deposits of melanin. See picture #56 in booklet. Intramucosal nevus: nevus cells are located in the connective tissue or lamina pro-pria ofthe oral mucosa. Under palpation these nevi appear solid and are slightly raised

or er the surface ofthe mucosa.

Important: Ifa pigmented lesion shows ulceration, an increase in size, darkening in color,

erc.. a biopsy should be performed--+his may indicate transfomation into a malignantmelanoma.

The traumatic bone cyst is an asymptomatic intmosseous empty cavity ofyoung patients located

p:imanll' *,ithin the mandible, lined by a thin loose connective tissue membrane.

TrarLmalic bone cysts are non-cysts fre/ened lo as pseudocysts) and have many names, whichinclude the simple bone cyst, hemorrhagic bone cyst, unicameral bone cyst, extravasationt'one cyst. idiopathic bone cyst and solitary bone cyst.

Clinical features:. Children and adolescents, usually before the age of20. \lore conmon in males. Usually asymptomatic, may produce enlargement ofthejaw. Conmonl,"- found in posterior mandible *** See picture #61 in booklet. Regional teeth are vital teeth

The treatment of traumatic cysts is relatively easy. lt consists of opening the l€sion, curettage

and closure. The resultant blood clot soon undergoes organization and the bone defect is soon re-

oarred.

\ote: This cyst is a closed compartment that has a connective tissue lining (iro epithelial lin-ilg,r ofvaried thickness. It may contain blood, serosanguineous fluid, debris composed chieflyofa blood clot, or it may be completely devoid ofsolid rnaterial.

The following are also not true cysts -they

are called "pseudocysts":. Latent bone cyst. Lingual mandibular concavity. Aneurysmal bone cyst

Page 125: Oral Pathology

ORAL PATHOLOGY Pseudocyst

All ofthe following statements are true concerning the aneurysmalbone cyst.EXCEPI one. Which one is the EXCEPTIOM

. It is an uncommon expansile osteolytic lesion of bone consisting of a proliferation ofvascular tissue that forms a lining around blood filled cystic lesions

. Most aneurysmal bone cysts occur in patients under 20 years of age, and it is uncom-mon after the age ol30

. It commonly involves the jaws

. The lesions are usually tender or painiul, particularly upon motion ofthe bone affected

. Upon entering the lesion surgically, excessive bleeding is encountered

124Copvnglu e 2011-:012 ' Dental Decks

ORAL PATHOLOGY R-B Les

A patient pres€nts with an asymptomatic, elongated, erythematouspatch of atrophic mucosa of the mid-dorsal surface of the tongue due

to a chronic C. albicans infection. The most likely diagnosis is:

. Thyroglossal duct cyst

. Lymphangioma

. Hemangioma

. Median rhomboid glossitis

.125

Copyiight O 20ll-1012 - Dental Decks

Page 126: Oral Pathology

*** This is false; it commonly involves the proximal humerus, femur, tibia and pelvis. Itis not as common in the jaws, but it does appear here as well.

The aneurysmal bone cyst (ABC) is classified as a pseudocyst because it appears radi-ographically as a cysrlike lesion but microscopically exhibits no epithelial lining. It rep-resents a benign lesion ofbone that may arise in the mandible, maxilla, or other bones.

It is typically seen in patients under 30 years old, with a slight female predilection. Whenthe mandible and maxilla are involved, the more posterior regions are affected, chiefly themolar areas. Pain is described in approximately half the cases, and a firm non-pulsatingswelling is a frequent clinical sign. On auscultation, a bruit is not head, and on firm pal-pation, crepitus may be noted.

The radiographic picture ofthe lesion is often distinctive. Usually a unilocular radiolu-cency is found with slightly irregular margins. The bone is expanded and appears cysticwith a "honeycomb" or "soap bubble" appearance. Teeth may be displaced with orwithout concomitant extemal root resorption. See picture #62 in booklet

Histologically, the lesion consists ofa fibrous connective tissue stroma containing manycavemous or sinusoidal blood-filled spaces. Fibroblasts and macrophages (histiocytes)

line the sinusoids. Multinucleated giant cells, similar to those ofa central giant cell gran-

uloma, are dispersed throughout.

Surgical curettage or excision is the treatment ofchoice, with little chance ofrecunence.

\ote: The differential diagnosis should include ameloblastoma, central giant cell granu-

loma and a central vascular lesion fthis can be ruled out uoon auscultation since a bruitis often heard).

This entity was once thought to be a congenital abnormality related to the persistence ofthe tuberculum impar, however, it is now believed that this condition is a permanent end

result of a chronic Candida albicans infection. Diabetics, immunosuppressed patients

and patients on long-term antibiotic therapy are more susceptible to this condition.

Median rhomboid glossitis usually presents as a smooth, denuded, beef,, red lesion de-void offiliform papillae. The most common location is the midline ofthe dorsum ofthetongue, just anterior to the circumvallate papillae. It is generally asymptomatic. Generallyno treatment is necessary, however, topical or systemic antif,rngal drugs to manage the pre-

disposing factors may be helpful.

Median rhomboid glossitis

Page 127: Oral Pathology

. Children

. Teenagers

. Post-menopausal women

. Middle-aged males

. Wegener's granulomatosis

. Rendu-Osler-Weber

. Sturge-Weber

. Juvenile nasopharyngeal angiofibroma

126

CoDrighr O 201l-2012 - Dental Decks

127

Cop)'righr O 201 I -2012 - Dental Deck

Page 128: Oral Pathology

Burning mouth syndrome (BMS) is a relatively common condition that is characterized bya complaint of an abnormal sensation of the lining of the mouth that most patients de-scribe as feeling like their mouth has been scalded. Usually this sensation develops in theftont part of the mouth, typically affecting the inner surfaces of the lips, the roof of themouth and the sides and tip of the tongue. In some patients, only the tongue will be af-fected, however, any combination ofthese sites may be seen. Some patients may have a

decreased taste ability or altered taste sensation (bitter or salt!). Other patients may feelthat their mouths are dry or sticky. In all cases, howeveq the lining of the mouth clini-cally appears normal. Note: BMS is a "diagnosis of exclusion."

The etiology is unknown. There are a few common diseases that should be tested lbr, such

as anemia. diabetes and oral yeast infections. Buming mouth syndrome is diagnosed bydoing blood tests and cultures to make certain that one of the other problems mentionedpreviously is not present. Ifthose tests are all negative, and ifthe lining ofthe mouth ap-

pears normal, then the diagnosis ofburning mouth syndrome is made.

Unfortunately, no one has developed a medically proven treatment for buming mouthslndrome. For about halfofthe affected patients, the condition will resolve after a periodof tin.re. but no one can predict how long that will be for a particular individual. For themost part. this problem is a nuisance, and it is a frustrating situation for both patients and

doctors. Note: Some individuals find relieffrom axiolytics, antidepressants, or low doses

ofcapsaicin, applied 3 or 4 times topically on the area(s) where the pain is localized. ap-

pear to be quickly effective in alleviating the pain in BMS.

Rendu-Osler-Weber syndrome, also known as hereditary hemorrhagic telangiectasia, is a con-

gcnrtal hcrcditary form of hemangioma. It is characterized by numerous spiderlike telangiec-

tases on the face, neck. chest. lips, gingiva, buccal mucosa and tonguc. One ofthe earliest signs ol'rhe disease is epistaxis (ho:tehleeds). Arteriovenous fistula, cspecially ofthe lungs and liver. are a

\ ariablc conlponent. Blceding from the tclangicctases may be recunent, lifethreatening and in-

;rease in sevcrity with aging. The onset often is in childhood. Both sexes are affected equally

Rememberi. Encephalotrigeminal angiomatosis fslurge-lleber disease) is an uncommon congcnltal syn-

dronrc ofunknown etiology (somelines classdied as a variant.forn ofhemangioma). It consists

of a l'acial lcsion, known as the port-wine stain, rvhich is distributed ovcr the trigeminal nerve

accompanicd by a similar vascular disorder ofthe underlying mr;ninges and cerebral cortex ltusuall! occurs unilaterally.. Juyenile nasopharyngeal angiofibroma is a rare, benign neoplasm that ncarly always aflects

adolescent males. It characteristically produces a mass in the nasopharynx that leads to ob-

struction or epistaxis. Treatment is surgery. Recurrences are conmon.

\\ egener's granulomatosis is a disease involving granulomatous inflarnmation, necrosis and vas-

cuLtis that most frcquently targets thc uppcr respiratory tract, lower rcspiratory tract, and kidneys.

The diagnosis ofWegener's granulomatosis is established most securely by biopsy specimcns show-

ing the triad ofvasculitis, granulomas. and large arcas ofnecrosis (lototvn a.s geographic necrosis)

admixed with acute and chronic intlammatory cells. Also CANCAs (otlttueulophil c)'toplashicantibodies) are highly spccific for Wegener's granulomatosis. The treatment includes prednisone and

cyclophosphamide.

Note: Midline granuloma is a unifocal destructive process, generally in the midline of the

orofacial region, that does not affect other organ systems. lt clinically mimics the lesions ofWegener's granulomatosis. Most cases represent peripheral T-cell lymphomas of the upper

respiratory tract or m ovth (perforation of the lwrd palate na!- be seen). There ls a good prog-

nosis when treated early with radiation.

Page 129: Oral Pathology

A clinical term delined as a red patch that cannot be clinicallyor pathologically diagnosed as any other condition is callsd:

. Leukoedema

. Psoriasis

. Erythroplakia

. White sponge nevus

124Cop)righr () 20ll 20ll Denral Decks

ORAL PATHOLOGY R-B Les

The arrow in the picture trelow is pointing to a fast-growing reactiveproliferation ofendothelial cells which is commonly found on the

gingivo and usually forms in response to chronic irritation.The most likely diagnosis is:

129Copy.ighr e 20ll 201? Denral Decks

Page 130: Oral Pathology

The term "erythroplakia'r like the term "lerkoplakia" has no histologic connotation. Itis a clinical diagnosis. Almost all true erythroplakias exhibit a microscopic picture ofepithelial dysplasia, carcinoma in situ, or invasive squamous cell carcinoma.\ote: Biopsy is mandatory.

Rem€mber: Carcinoma in situ exhibits all of the histologic characteristics of malig-nancy (pleomorphism, hyperchromatism, abnonnal niloses, onaplasia, e/c.), but does notshorr invasiveness or extension into adjacent structures.

Enthroplakias may be located anywhere in the mouth, but are most likely to be found inrhe mandibular mucobuccal fold, oropharynx and floor of the mouth. There is no sex

predilection and patients over 60 years old are most commonly affected.

This lesion represents an overexuberant connective tissue reaction to a known stimu-Ius or injury. The term "pyogenic granuloma" is somewhat ofa misnomer in that it is notpus producing, as "pyogenic" implies. It is, however, a "tumor" ofgranulation tissue, as"granuloma" implies.

Pl ogenic granulomas may also occur on the lower lip, tongue and the buccal mucosa.

Thel rarel.v occur on other areas ofthe oral mucous membrane. It is generally believedlhar rhe pyogenic granuloma arises as a result of some minor trauma to the tissues fce-neutdtion of q crotvrt, calculus, etc,),which provides a pathway for the invasion olnon-specitic t-vpes of microorganisms.

Pr ogenic granulomas present as soft pedunculated broad-based growths that have asmooth red surface. This red appearance is due to the presence ofhyperplastic granula-tion tissue, which contains many capillaries. They are often ulcerated, bleed easily and

ma1 have a raspberry-like appearance. Treatment consists ofexcision. They are benign,

but rnav occasionally recur.

\ote: Pregnant patients are prone to these lesions (sorreliz es called "pregntncy tumor").

Page 131: Oral Pathology

Peripheral giant cell granulomas are seen €xclusively in:

. Buccal mucosa

. Alveolar mucosa

. Bone

. Gingiva

't 30

coplright o 20ll-2012 'DmialDects

ORAL PATHOLOGY

131

Coplyighi C 201 1,2012 - Dentnl Decks

The picture below shows a benign, soft, moderately well-circumscribedprinless mass which is deep red or blueish red in coloration.

The most likely diagnosis is:

Page 132: Oral Pathology

The peripheral giant cell granuloma has an unknown etiology, with some dispute as to

whether this lesion represents a reactive or neoplastic process. However, most authoritiesbelieve the peripheral giant cell granuloma is a reactiv€ lesion. Local irritation due to

dental plaque or calculus, periodontal disease, poor dental restorations, ill-fitting dental

appliances, or dental extractions has been suggested to contribute to the development ofthe lesion. They are seen exclusively in the gingiva, usually in the area between the firstmolars and the incisors. See picture #63 in booklet

Peripheral giant cell granulomas typically present as red to blue broad-based masses.

They usually bleed easily and may occur at any age and tend to be seen more frequentlyin females than in males.

When this process occurs on the edentulous ridge, a superficial, cup-shaped radiolucencymay be seen. Histologic sections are diagnostic and are identical to those of a centralgiant cell granuloma. The lesion is composed of a proliferation of spindled fibroblasts

in a stroma containing variable amounts ofcollagen. Multinucleated giant cells are pres-

ent throughout the connective tissue stroma. Conservative excision is typically curative,although the lesion must be completely removed to prevent recurrence.

\ote: Generally, this lesion is clinically indistinguishable from a pyogenic granuloma.

-{lthough the peripheral giant cell granuloma is more likely to cause bone resorption than

rs the pvogenic granuloma, the differences are otherwise slight. Biopsy will provide de-

tinitir e results.

Hcmangiomas are benign vascular tumors composed ofcells that nonnally line the blood vessels

rentlothelial cells). They arc the most conmon tumor ofchildhood, occurring in up to l0% ofin-1ants. Henangiomas may be visible at birth or may not be recognized until the first few weeks or

cr en months of lif'e. ln general, however, most hemangiomas becomc evident by 2 to 3 weeks 01'

age. -\lthough they comrnonly occur on the head or ncck (60%). they can occur in any region ofrhc bodl ). Some lesions are small and hardly visible, whereas others are large and rcadily obscrv-

:b1c'. \lost hemangiomas appear as single tumors, though l57o present with more than one lesion.

1 . Congenital hemangioma (slrawberry revr.r'./: local proliferation of capillaries; ex-

\otes hibit a rapid growth phase followed several years latcr by an involution phase; persist-

ent lesions are excised.2. \'ascular malformation: a pcrsistcnt malfonnation of blood vessels; do not invo-lutc; exhibits a thrill (palpate a pulse) andbrdt thear a pulse). A typc ofvascular nal-lomation is known as Sturge-Weber disease (encephalotrigeminul angiomatosis)

w hich consists of a facial lcsion. known as the port-rvine stain, which is distributcd

over thc trigeminal nervc accompanied by a similar vascular disorder ofthe underlyingmeninges and cerebral co ex. lt usually occurs unilaterally.3. Remember: Rendu-Osler-weber syndrome, also known as hereditary henlorrhagic

telangiectasia, is a congenital hcreditary lbrm ofhemangioma. 11 is charactcrizcd by nu-

merous spider-like telangiectases on thc face, neck, chest. lips, gingiva, buccal mu-

cosa and tongue. One of the earliest signs of the disease is epistaxis (nosebleeds).

Arteriovenous listula, especially ofthe lungs and liver, are a variable component. Bleed-

ing from thc telangiectases may bc rcculrent, lil'e{hreatening and increase in sevcrity

rvith aging. The onsct oftcn is in childhood. Both sexes arc aflected equally.

4. lmportant: How to distinguish between a hemangionra and a hcmatoma -heman-gioma will blanch on diascopy. hematomas do not blanch.

Page 133: Oral Pathology

. Uncommon and represerfiz4yo ofhead and neck neoplasms

. Common and represent 75-80% ofhead and neck neoplasms

. Uncommon and represent 25-30olo of head and neck neoplasms

. Common and represent 95-98% ofhead and neck neoplasms

132Coplright @ 201 I -2012 - Denral Decks

. White sponge nevus

. Lichen planus

. Necrotizing sialometaplasia

. Focal hyperkeratosis

133

Coplright O 201l-2012 - Denral Decks

Page 134: Oral Pathology

Salivary gland tumors may be broadly categorized into benign neoplasms, tumorJike condit-ions, and malignant neoplasms. The glands are divided into major and minor salivary gland

categories. The major salivary glands are the parotid, the submandibular. and the sublingualglands. The minor glands are dispersed throughout the upper oral-digestive submucosa(i.e., palate, lip, pha4nx, nasopharl,rrx, latynx, paraphatyngeal space).

Benign Salivary Gland Tumors:. Pleornorphic adenoma lbenign mLxed tumor./ is the most common tumor of major and

minor salivary glands. It is a mixture of epithelial and mesenchymal elements (pleomor-phi.).85yo occnr in the parotid gland. Within the oral cavity, the majority are found in thepalate. Pathologically, it is characterized by slow growth and few symptoms.. Monomorphic adenomas: The term "monornorphic adenoma" refem to a group ofraresalivary tumo$ that includes the basal cell, canalicular, sebaceous, myoepitheliomas and on-cocytomas. Ofthese, the basal cell adenoma is the most common.. Warthin's ttrnor (papillary' .!-stadenoma lymphomatosnn) is almost exclusively a

parotid neoplasm. It is a glandular and cystic tumor lined by a bllayered (inner columnarotlcoc)lic qnd outer ba.sal) epithelium with a lymphoid stroma.. Benign lymphoepithelial lesions include a wide range of cystic changes that share the

common denominator in atypical lymphoid hyperplasia. Hyperplasia refers to a prolifera-tion ofcells that is non-neoplastic. These changes are found often in patients infected withH I\i

L Clinical features of benign salivary gland tumors: mucosa is normal, painless,

-\.'ot r nodular, movable, fifm. and slow-growing.2. The most common site ofintraoral minor salivary gland neoplasms is the palat€.3. The most common site of intraoral major salivary gland neoplasms is theparotid.

\ecrotizing sialometaplasia is a recognized lesion of the minor saliYary glands char-

acterized by necrosis ofthe glandular parenchyma with associated squamous metaplasia

and hyperplasia ofthe ductal epithelium.

Important: The initiating event ofnecrotizing sialometaplasia is believed to be related to

ischemia, secondary to alteration of local blood supply. lnfarction of the salivary gland

tbllou s. presumably due to compromise ofthe vascular supply.

It is usually observed in adult rnales and presents as an asymptomatic, necrotic. ulcerated

area inr,olving the palatal mucosa. The hard palate appears to be the most common site.

Histologicall!', there is lobular necrosis of the glandular parenchyma, with squamous

metaplasia and hlperplasia of the ductal epithelium. Both clinically and histologicallSthe lesion may sin.rulate a malignancy and in the past, the condition has been misdiag-nosed as a squamous cell carcinoma or mucoepidermoid carcinoma.

Following biopsy and the establishment ofthe diagnosis, further treatment generally is notrecommended since healing usually occurs within 6-10 weeks.

Page 135: Oral Pathology

. Measles

. Mumps

. Rubella

. Chicken pox

1UCopyright O 20ll-2012 " Dent.l Decks

. Parotid gland tissue

. Submandibular gland tissue

. Sublingual gland tissue

. All of the above

l3scoplright @ 20r I -20 12 - Dental Decks

Page 136: Oral Pathology

Mumps is an acute viral illness. The mumps virus is a paramyxovirus in the same group

as the parainfluenza virus. The virus is acquired by respiratory droplets. It replicates in thenasopharynx and regional lymph nodes.

Important: Mr.rmps is the most frequent diagnosis of acute viral sialadenitis, mostly inthe parotid gland.

Clinical:. 907o ofthe cases occur before l4 years of age. A major sign is sudden salivary gland swelling without purulent discharge from the

duct*** Parotid

-90o% involvement and bilateral in two-thirds of the cases.

. \4iltl ferer. malaise and anorexia

. Most cases are self-limiting

Complications:. Orchitis (inflqmmation of the testis) and epididymitis can occur in post-pubertalmales. *** Important: May cause sterility. CNS systeln rneningitis and encephalitis. Dealness. myocarditis, pancreatitis, oophoritis and pyelonephritis

The serum amylase may be elevated during the acute phase. Prevention with a liveattenuated vaccine is 95% effective for at least five years. However, in non-inoculatedindir iduals, it is still a cause ofacute non-suppurative saliyary adenitis.

The stafne bone cavity or cyst is a developmental anomaly represented by a bone con-

car itl usually containing submandibular gland tissue. The posterior mandible region,pafiicularly at the angle and below the mandibular canal is the most common location. The

sraric bone cyst is usually discovered incidentally on dental radiographs. It is asympto-

matic and is not a true cyst. rather an anatomic depression in the lingual aspect of the

bodl of the mandible where normal salivary gland tissue rests. The radiographs show asmall, circular, corticated radiolucency below the level of the mandibular canal. His-tologicalll', nonr.ral submandibular salivary gland tissue is found and no treatment is re-quired except routine radiographic follow-up. Note: Many other terms have been used to

describe this entity, including submandibular salivary gland depression, lingual salivaryeland deoression. and Stafne bone cvst.

Stafne bone cyst in characteristic location inferior to the mandibular canal

Page 137: Oral Pathology

ORAL PATHOLOGY SG Thm

A 53-year-old woman comes into the dental clinic with bilaterallyenlarged parotid glands. It was discovered that she had recentlyb€en to the African contin€nt and had contracted tuberculosis.What is the name of the autoimmune disease associat€d with

enlarged salivary glands in association with a secondary disease?

. Sjogren's syndrome

. Mikulicz's disease

. Gorlin-Goltz syndrome

. Pierre-robin syndrome

. Apert syndrorne

136

Copyrighr aq 20ll l0ll - Dcnbl Decks

ORAL PATHOLOGY SGTirm

A 33-year-old patient comes into your office for a routine maintenance appt.While doing an intraoral exam, the hygienist discovers a bluish lesion ofthe

lower lip. The patient relates a history of biting this area last week wh€nhe had a sinus infection. What is the most likely diagnosis ofthis lesion?

. Ranula

. lnfectious sialadenitis

. Maxillary sinus retention cyst

. Mucocele

137

CoDrighr aq 20ll-2011- D.ntal Decks

Page 138: Oral Pathology

The term "benign lvmphoepithelial lesion" (also called Mikulicz's disease) is mani-fested essentially as a progressive, asymptomatic enlargement of the parotid and

submandibular glands. It is initially unilateral, but over time, it becomes bilateral. The

etiology is unknoun, however, there is increasing evidence that both Mikulicz's disease

and Sjogren's syndrome are both actually autoirnmune diseases in which the patient'so\\ n salivaD gland tissue becomes anligenic.

The benign lymphoepithelial lesion is rare. It occurs most frequently in middle-agedrvomen. Histologically', there is replacement of the gland parenchyma by lymphocyticinliltrate rvithin which there are scattered epimyoepithelial islands. This is tl.re histologiccomerstone lor the diagnosis of the benign lymphoepithelial lesion.

The differential diagnosis ofchronic bilateral enlargement ofthe salivary glands as a re-

sult of benign lymphoepithelial lesion must be separated for similar findings seen in sar-

coidosis. lymphoma, gout. leukemia, diabetes mellitus, chronic alcohol abuse, and, rarely

hlpenension.

Important: Most of these lesions remain benign. however. malignant transformationof the epimyoepithelial islands has been demonstrated.

\lucuse\tralasationphenomenon,commonlykno\\nbytheclinical term, "mucocele", is scparrtcd from

mucus retention c!-st bccause each has a disdncrivc pathogenesis and microscopy. Scveral clinical features

rl.,r .il.a:ale thcsc lcsions.. \lucus ertravasation phenomenon: is considered to be related to rnechanical lrauma to thc minor sali-\ 3a -!lalrd e\cretory duct. Although the lower lip is thc most frcqucnt sitc. the buccal mucosa, vcntral sur_

ficr- Lrf thc rongue (\'here the glanls of Blondin Nuhn arc locdterll. lloor ol thc rnouth, and reiromolar:r!iLrr rrc ofien atfccted. These lesions are usually painless and smoolh surl'accd, \\'ith an o\eral1 bluish hue

!rf rranslucenc) associated \lith a lnore superficial localion. The trcrtment ofchoice firr mucus extravasr-

:r!r: r!hcnonrenon is surgical cxcision rvith rcmoval ofthe associatcd gllnd.. \luaur retention cyst: is regarded as a cyst becausc it is lincd by an epithelium - unlike mucus ex-i::r\ 3sairon phcnomcnon. r.hich contains mucus pool surrounded by granulation tissue.This cyst resuhs

tirm obtnu.lion of sali\ary flo\l', The mucus letention typc mucocclc is less common than the extravasa-

:ri:] \rrc Thc rcrcntion t-vpe usually appcars aftcr 50 ycars ofagc, and it rarely prcscnts in the lorver lip.I:r.iead it is found in the palate, cheek, floor ofihe mouth. as wcll as in the nraxillary sinus. Clinical prcs-

.::]iron i\ characicrized by an asympbmatic s$elling, oflen without previous injury. The Icsions varyl:L.nr I to l0 mm and. on palpation. are rnobile, non-tender and lhe overlying mucosa is intact and ofror-mal color. Thc trca{ment ofthe mucus rctcntion cyst inchldcs surgical excisron *'ith rcmoval ofthe asso-

;ra:ed gland.

lmportant: "}Iucoccle" is uscd in the clinical setting as a generic term (hclbrc nicroscopic didguo.ri.r) to

refer to both the mucus retention cyst and the mucus extravasation phenomenon.

L The maxillary sinus ret€ntion cyst and pseudoclsts are commonly discovcrcd incidentally,

\ote3 ofren being ofgrcatcr curiosity than clinical significance on panoramic radiographs. May repre-

. sent b lockage o f sinu s sal ivary gland, or focal fluid accumulation of sinus mucosa. These lesions' arc asymptomatic and require no treatment.

:. Infectious sialadenitis: int'ections ofthe salivaF"' glands ftav be acute or chronic. viral or bac-

tc al.. \'iral:

- Nfumps is the most frequent diagnosis ofacute viral sialadenitis, mosdy in parotid.- Cytomegalovirus infections are chronic; usually seeD in immunosuppressed patjents

. Bacterial: the most commonly isolated organisms arc Staphylococus aurcus. Strcptococ-

cus. viridans. II. intluenzae. and E. coli.

Page 139: Oral Pathology

What is the most probable diagnosis for a lesion that presentsas a translucent, bluish, well-rounded, smooth-surfaced bulge

that protrudes from one side ofthe floor ofthe mouth?

. Adenoid carcinoma

. Squamous cell carcinoma

. A ranula

. A lymphangioma

138

Cop)aighr iar 201l-2012 - Dental Decks

ORAL PATHOLOGY SG Thm

A patient comes to your ollice complaining of pain when eating and evensometimes when thinking about food. Your intraoral exam reveals a small,hard swelling in the floor ofthe mouth. A mandibular occlusal radiographshows a per-sized radiopacity with 'onion-skin" thickening lingual to the

right mandibular border. Name the likely diagnosis:

. Sialometaplasia

. Sialadenitis

. Sialolith

. Sialosis

139

Coplriglit C 2011-2011 - Denral Dects

Page 140: Oral Pathology

Ranula, is a clinical term that is used to designate a mucocele that occurs specifically inthe floor ofthe mouth (see picture below). Pathogenetically and microscopically, it mayrepresent either mucus extravasation phenomenon or mucus retention cyst. The ranula is

associated with the duct system ofthe sublingual salivary glands, and, less commonly, thesubmandibular glands.

The ranula is usually caused by an obstruction produced by either a salivary stone or softorganic substance. It usually presents as a painless, fluctuant, unilateral, soft tissue mass

in the floor ofthe mouth. It t)?ically has a bluish appearance that has been compared toa frog's belly, hence the term "ranula."

Of diagnostic significance is a history of increased size just before or during a meal,and a decrease in size between meals. The treatment is surgical, either through completeexcision or by removing the roof ofthe cysl. If it persists, excision ofthe gland may beneeded.

A sialofith is a stone (salivary calculus) within a salivary gland or duct. The formation ofa sialolith is called sialolithiasis and occurs as a result of precipitation of calcium andphosphate salts around a nidus ofmucous or bacterial debris. Sialoliths occur as single ormultiple stones and can cause swelling and pain. The pain is experienced during salivarystimulation and is intensified at mealtimes.

The best radiographic projection for visualizing sialoliths in the submandibular duct andgland is the standard mandibular occlusal view. Occasionally, sialoliths are seen inci-dentally on periapical radiographs, in which case they may be misdiagnosed as os-teosclerosis.

The rate of occurrence in submandibular gland and duct is much higher than in theparotid or sublingual areas. This is thought to be due to the tenacity ofthe submandibu-lar saliva and the long and irregular shape of the duct. See picture #64 in booklet

The treatnent ofchoice is almost invariably surgical extirpation ofthe sialolith. Stones

located in the glandular parench;.rna usually require removal ofthe gland as well.

Page 141: Oral Pathology

Of the neoplasms rffecting the mNjor or minorglands; the is the most common.

. Basal cell adenoma

. Sebaceous adenoma

. Pleomorphic adenoma

. Ductal papilloma

140

Coplright O 201 I -20 l2 - D€nral Decks

. Sarcoidosis

. Mikulicz's disease

. Sj<igren's syndrome

. Hypothyroidism

. Diabetes mellitus

. Malnutritior/Starvation

. Dehydration

. Cystic fibrosis 141

CopJaight @ 201l-2012 - D€nral Decks

Page 142: Oral Pathology

Pleomorphic adenoma (benign mixed tumor, is the most common tumor of major and

minor salivary glands. It is a mixture of epithelial and mesench).,rnal elements (pleomor-

phic).Int\e parorid, 90% occur in the superficial lobe and most conmonly are seen in the

tail ofthe gland. Within the oral cavity, the majority are found in the palate. Pathologi-cally, it is characterized by slow growth and few symptoms. See pictures #65 and #66 introoklet

The gross pathologic appearance of a pleomorphic adenoma is a smooth or lobulated.well-encapsulated tumor that is clearly demarcated from the surrounding normal salivarygland. They are tlpically solid tumors and may have areas ofgelationous myxoid stroma.

Cystic degeneration or tumor infarction and nectosis are rarely seen except in large, long-standing lesions. Microscopically, these tumors are composed of varying propoftions ofgland-like epithelium and mesenchymal stromas. The treatment ofchoice is surgical ex-cision with a generous margin ofnormal tissue. Inadequate initial removal ofthe mixedtumor in major glands may result in recurrence. Approximately 25% ofbenign mixed tu-mors rvill undergo malignant transformation illesions are untreated for an extended length

of tirne.

Sebaceous adenomas are rare lesions composed predominantly of sebaceous gland-de-

rir ed cells; they are well differentiated in the benign forms. The parotid gland is the site

of chief involvement. Parotidectomy is the treatment ofchoice when lesions arise in this

sland.

Ductal papillomas are a rare group ofbenign papillary neoplasms ofthe large excretoryduct. There are three types major subtypes of ductal papillomas of salivary gland originthar are about equally rare: the intraductal papilloma, the inverted ductal papilloma, and

the sialadenoma papilliferum. The treatment for all three is conservative simple local ex-

cls1(]n.

There are many conditions associated with parotid gland enlargement, this may be unilateralor bilateral.

. Sjtigren's syndrome: is an autoimmune disorder affecting lacrimal and salivary glands\\ hich causes decreased moisture in glands. Dry mouth, tooth decay, mouth sores, enlargedsalir ary glands, sialoliths, and recunent salivary gland infections are possible symptoms.The syndrome also et'fects moisture in the eyes, which might cause chronic eye infections,comeal ulcers and vision loss.. Sarcoidosis: unknown cause; believed to be alteration in cellular immune function and in-\ ol\'ement of some allergen. Most often involves the lungs; can affect the parotid gland.Granulomatous fmacrophage infiltates) inflammation causes organ nodularity and loss ofparenchyma. Note: Sarcoidosis is also characterized by distinctive laboratory abnormalities,including hyperglobulinemia, an elevated serum angiotensin converting enzyme level andhypercalcemia. Glucocorticoids remain the mainstay oftherapy when treatment is required.although other anti-inflammatory agents are being used increasingly often.. \\'arthin's tumor: also called papillary cystadenoma lymphomatosum. Infections: for example, mumps, actinomycosis, tuberculosis. Benign lymphoepithelial lesion: also called Mikuticz's disease. l\4alnutdtion/Starvation. Dehydration. Cystic fibrosis and diabetes mellitus

\letabolic conditions associated with bilateral parotid gland enlargement include: chronic alco-holism, dictary deficiencies, diabetes mcllitus, hypertcnsion, hyperlipidemia and Sjdgren's syn-drome.

Page 143: Oral Pathology

The lcinic cell crrcinoma is derived from serous acinsr cellsand is fouRd almost exclusively in the:

. Submandibular gland

. Parotid gland

. Minor glands of the palate

. Sublingual gland

142Coplrighr O 201 1-201 2 , D€nial Decks

. Adenoid cystic carcinoma

. Mucoepidermoid carcinoma

. Acinic cell carcinoma

. Polymorphous low-grade adenocarcrnoma

1/f,3

Copt'righr O 201 I '2012 - Dental Decks

Page 144: Oral Pathology

\cinic cell carcinoma is the second most common parotid malignancy and the second most conlmc\n

^:i:rlnc salivary gland ma\gnancy (nucoepidermoid carcinoma is lhe nost conmonlbr both). CrL,"appearance demonstrates a mass ihat is well-circumscribed but lacks a true caps[le. Microscopic ap-

pearance has been categorizcd as solid, microcystic, papillary cystic and follicular. Tumor cells are darkstaining and have granular or honeycomb cytoplasm. The surrounding sffoma often demonstrates a lym-phoid infiltrate. Treatment of acinic cell carcinoma includcs surgical cxcision. This tumor is gcncrallyregarded as a low-grade malignancy.

Adenocarcinomas (NOS -

not olheheise specirted) ofthe salivary glands are rare but aggressive tu-mors. About halfofthese trmors present in the parotid glands. Minor saiivary glands on the palate, lipand tongue are the next most commonly affbctcd sitcs. Adenocarcinoma is different from other salivarygland neoplasms in that as many as 2570 ofpatients will complain ofpain or facial weakness at pres-

entalion. Gross pathology reveals a firm mass with irregular bordcrs and infiltration into surroundin-e tis-sue. It is generally a solid tumor wjthorit any cystic spaces- These malignancies can demonstrate a widerange ofgrowth pattems and, for lhis rcason, can bc somewhat difficult to classily. However' all adc-

nocarcinomas have in common thc formalion ofglandular structures and they are described as grades

I. II or lll based upon thc degrcc of ccllular differcntiation. Grade I lesions havc wcll-formcd ductalitruclures \\hile Grade III lesions have a more solid growth pattern with few glandular chamcteristics.

Bccausc thcsc are more aggressive tumors, treatment for adcnocarcinoma is more aggressive. Completelocal excision is the mainstay oftherapy.

\lalignant Mixed Tumor rcpresents thre€ separate entities that are histologically distinct; These in-clude the:

. Carcinoma ex-mixed tumor is the most oommon ofthe three salivary ncoplasms. It occurs whena carcinoma develops from the epithelial component ofa preexisting pleomorphic adenoma.. Carcinosaacoma or true malignant mixed tumor: thc mctastatic lesions contain both the stromalend epithelial elements. This is difl'erent from the carcinoma ex-mixed tumor in which only the ep-'rl_alial clcmcnti are prc5cnt in mclaslasii.. \Ietrstasizing mixed tumor refers to an otherwise benign acting pleomorphic adenoma that de-

r eloos metastatic deoosits oftumor

\tucoepidermoid carcinoma is the most common malignant salivary gland neoplasm in both minor and

rNaior glands. It devclops rnost commonly in thc major salivary glands, mosl oftcn the parotid (45-70'/o)

Tha sccond most common sitc ofoccurrence is the palate //8Zr). Thcse tumors grow slorvlY and pres-

lr:: as painless masses in most cases. Nlicroscopically, thcsc fumors are charactcrized by the presence

..: t\\ o populations ofcells the mucus cells and the epidermoid cells. the proponion of rvhich helps to

i3ine thc grade ofthc txmor. They arc uncncapsulated or poorly encapsulated and intlltrate srirround_

::r!lirsuefreelt.'High-grade"and"lowgradc"formsexisteachwithadifiercntprognosiskoodforloy'!,ttti. poor.fot high-g^rrle/. Surgical cxcision is the ffeatment. See picture #67 in booklet

.\denoid clstic carcinoma /'1CC/ accounts for approximately 23% ofall salivary gland carcinomas. Ap-p:o\iDrarcl) 50 to 709/0 occur in the minor salivary glands ofthe palate- In the maior salivary glands. the

prrotrd gland is nrost often affected. Microscopic appearance is described as cribriform, rubular or solidTh. cribrifonn pattem is the most common and most easily rccognizable. It is often referred lo as the_s\riss cheese" pattem. Tumor cells arc armnged in ncsts around cylindrical spaces that may contain a

nucrnous or hyalinized material. Cells lhat are arranged in layers and form ductal struclures character_

lzi rhc i.rbular pattem. Thc solid pattem contains sheets offumor cc]Js u'ith lo intent'ning spaces.

See picture #68 in booklet

.\CC is an unusual tumor It is slow-growing but relentless. It tcnds to be locally invasive and infiltraterhe 'iheaths" or coatings surrounding nerve fibers (per-lneurol spaces). ACC often rccuN years later at

ihe site \\here thc fumor firct arose or it mny metastasize. Unlike most carcinomas, it seldom metasta-

sizes to nearby lFnph nodcs but rather to distant sites. The lung is thc most common site ofmetastasis,

\\ rth rhc liver second. Bone metastases indicate a poor prognosis.

PofJmorphous low-grade adenocarcinom (PLGA) is the second most common malignancy in the

minor salivary glands and occurs most frequently in the palatc. lip and buccal mucosa. The microscopic

appearance ofthcsc tumors is what gjves them their name. Any ofa variety ofgrowth pattems (solid, tu'hula4 trubecular, glandular tribriforn, cysti., can be seen within the same lcsion or among different le-

sions. PLCA displays a tendency for perinerual and perivascular invasion, howevcr it typically followsan indolent course. Treatment consists ofconscrvativc yct complete local excision. Distant metastasis is

not common,

Page 145: Oral Pathology

. Lupus Erythematosus

' Sjogren's disease

. Sarcoidosis

. Crohn's disease

. Malignant

. Mixed ftenign and malignant)

. Carcinoma in situ

. Benign

141

Coplrighr e 201 l'2012 - Dental D€cks

'145

CopltiSht O 20ll-2012 - Dental D€cIs

Page 146: Oral Pathology

Sjiigren's syndrome is a chronic lymphocyte-mediated autoimmune disorder aIi'ecting the lacrimal and sali-vary glands as well as other organ systcms. Thc paticnts most commonly afTected arc post-mcnopausal womenwho prescnt with dry eycs. dry mouth and, in about 50% of the cases, enlargemen! ofthe parotid and sub-mandibular gl^nds bilarerally. Primary Sjdgren's syndrome consists ofxeroslomia and kcratoconjunctivilisstcc (dry eres), and possibly salivary gland enlargement. S€condary Sjcjgren's syndrome has the same xe-rostoma and keratoconjunctivilis sicca but is also associated with anothcr autoimmune disord€r such as R-{,SLE, etc. There is dense inflammatory infiltrate $ ith destruction of glandular tissue- Treatmenr is palliative.

Note: Biopsy ofthe labial or palatal salivary glands may bc helpful in establishing the diagnosis, along withsialograms, salivarj" flow rate tests , the Schimer test ldeterniles \lrcther the e),e procluces enough tears to*eep it moist)

^ndblood work. Important: Only halfofSjdgrcn paticnts have anti-SSA and/or anti-SSB /SJd-

grcn\ Svndronte A dnd B,) antibodies in their blood. Other tests which may be abnormal include the whiteblood cell count //ot . total gamna globulin level /rr'll. blood C3 and C4 complemen! levels /hx C4l. sed-imentation rate frigrl aud rheumatoid factor lporltlr?r.

Important: The decrease ir1 salivation may cause rampant caries reminiscent ofradiation cariqs.

Important: The histological f'eafures ofthe salivary gland lesions in both Sjitgren's syndrome and the "be-nign l) m ph o€pith€fial lesion" lalso calle.l Milrll.: 3 ./rs?.rr., are idcntical.

It is i ponant to remenlber that malignant lymphomrs and "pseudolymphomas" (also cdlled at picdl be-

nign lvehoid h.r!)erpld.rld) develop in some patients who have been diagnosed *ith Sjcigren's syndrome. Thismandarcl clo$ follo$-up oflhe parienl!.

Lupus er]thematosus is a chronic autoimmune disease that occurs in tivo forms:. Discoid lupus erythem^tos\s (DLE)| is I chronic skin condiiion of sorcs with inflamrnation and scar-ring fa\oring the facc, can, scalp, and/or oral mvcosl (i.e., buccdl nucosa. gingivd, rcr illion). Thcse lc-iions delelop as a red, inflamed patch with a scaling ilnd crusty appearance. Oral lesions mimic erosivefichen planus. Trcated with cortisone or other drldgs (i.e., Pldqucnil. Aralen, or Quinacrine).. St stemic lupus erythematosus (SIrr: is a chronic inflammatory connectivc tissuc disorder that involvcsmuhiplc organs including the kidneys. heaft,joints, skin, mucous membranes, and blood vessel walls. Abut-t.rll)-shapcd rash /ma1dr rdrrl on the t'ace that covers the cheeks and bridge ofthe nose is a c)assic sign-\ole: \umercus autoantibodics direcled against nuclcar and cytoplasmic antigcns are fbund in SLE palients.

Oral lesions are generally similar to thosc secn in DLE. Scrologic rcsis (ANA [aubantibodie.\J test and LEicll re-vr are Dositive in Datients with SLE.

.\pproximatel,v 8 out of l0 salivary tumors diagnosed are in a parotid gland. One in 10 diagnosed is in

l submandibular gland. Thc remaining l0% are diagnosed in othcr salivary glands. In g€neral, glands

morc likcl! to show tumor growth are also glands least likcly to show malignant nrmor growlh Thus.

elihr.ugh tumors ofthe sublingual glands arc rarc, almost all ofthem are malignant. In contrast, onlyrl.rlrt Iion ofparotid gland tumors are mahgnant.

Clinical leatures of malignant salivary gland tumors:

- Jlucosa is ulcerated - Firm - Painful - Nodulas - Fixed - Rapid growth

\lafignant Safivary Gl^ndTumors (b eflt)t. \denoca rcinoma, N OS lnot olherv,ise specified), arc rare but aggressive tumors About half ofrh.se tumors present in the parotid glands. Adenocarcinoma is different fiom othcr salivary gland neo-

fla5ms in that as many as 259lo ofpaticnts rvill complain ofpain or frcial rreakness ai presentatlon.

' {denoid c!,stic carcinoma: approximateiy 50 to 7oyo occur in the minor salivary glands of the

talate. In the major salivary glands, the parotid gland is most often affected. Microscoplc appearance

is described as cribriform, tubular or solid. The cribriform pattern is the most common and most

carily recogrizable. It is often refened to as the "swiss cheese" pattem.. \lucoepidermoid carcinoma: is the most common malignant salivary gland neoPlasm in both

minor and major glands. lt develops most commonly in the major salivary glands. nost otien thepatotid G5-70a.,'). Microscopically, these tumors arc characterized by the prescnce of two populations

ofcells thc mucns cells and the epidermoid cells, the proportion ofwhich helps to define lhe grade

of thc tumor. Acinic cell carcinoma: Acinic cell carcinoma is the second most common parotid malignancy and

thc second most common pediatric salivary gland malignancy ih&coep[dermoid carcinona is lhe osl

t'onnnon lor hoth).This tumor is gcncrally regarded as a lort-grade malignancy..llalignant mixed tumors: represents three scparate entities that are histologically distinct; These

includc thc carcinoma ex-mixcd tumor, carcinosarcoma or true malignant mixed tulnor, and thc metas-

lasizing mixed tumor.. Polymorphous low-grade adenocarciliotn (PLGA) is the second most common malignancy inthe minor salivary glands and occurs most frequently in the palate, lip and buccal mucosa. The mi-croscopic appeamnce ofthesc tumors is what gives them their name.

Page 147: Oral Pathology

ORAL PATHOLOGY

Oncocytomas are _ tumors that constitute aboutof benign epithelial salivary gland neoplasms.

. Common; 50o/o

. Common; 75%

. Rare; 2%

. Rare; l5olo

'146

Copyright C 20ll-l0l: - Denkl Dects

ORAL PATHOLOGY

A 65-year-old patient comes to your oflice complaining of a slowly-growingenlargement ofthe jaw. You palpate the angle of his right ramus and findan encapsulated mass that is non-tender and firm. Your oral pathologist

defines it as a glandular and cystic tumor lined by a bilxyered (innercolumnar oncoc!,tic and outer basal) epithelium with a lymphoid stroma.

Name this second most common benign neoplasm of the parotids.

. Pleomorphic adenoma

. Warthin's tumor

. Fibroadenoma

. Monomorphic adenoma

147

CoDrigllr t) 20ll-201? - Dental Decks

Page 148: Oral Pathology

The term "monomorphic adenoma" refcrs to a group ofrare salivary tumors that includes thc basal cell

adenomas, canalicular adenomas. scbaccous adenomas, myoepithcliomas and oncocytomas.

Oncocytomas arc rare tumors that constitute only 2% of benign epithelial salivary gland neoplasms

Thc majorjty ofthese tumors affcct lhc parottd gland 178%). The clinical prcscntation ofoncocyomasis esscntially identical to other bcnign salivary tumors a slowly growing, nontender mass. typicallyin the superficial lobe ofthe parotid. Microscopically, thcrc arc shccts, nests or cords ofuniform onco-

cytes.

Basal cell adenomas are also rare tumors that constitute only J'lo ofbcnign epithelial salivary gland !u-

mors and gpically occrrrs in the 6th decade oflife. It seems to occur morc frequently arnong Caucasians

than African Amcricans. The majority occur in the parotid gland where thcy present as a slowly cnlarg-ing firm mass. They are well-encapsulated, smooth tumors on lross inspection and arc divided into foursubtypes based on their microscopic appearance solid, trabecular, tubular and membranous

Canalicular adenomas are also rarc ftrmors (1.596) that most commonly involve thc minor salivaryglands of the npper lip (74yu) or brc.al mrcosnl ( 12a;).It peaks in thc 7th decade of life and, like tllebasal cell adcnoma, is morc common in whites than blacks Clinically it presents as a nonpainful sub-

mucosal nodule. On gross pathologic examination, canalicular adenomas may or may not possess a cap_

sulc and it is not unusual for therc to bc multifocal glowth. Microscopically there are cords ofsingle-layercolumnar or cuboidal cells forming duct-like structures in a background of fibrous stroma

Thc rarc m!oepithelioma accounts for less than one perccnt ofall salivary gland ncoplasms. Most arise

\! irhin the parotid gland and, less frequcntly, in the submandibular gland and intraoral minor salivaryglands. Three paftems ofmicroscopic appearance have been describcd. The spindle cell pattern is the

mosl common ovcrall and is typical for parotid myoepitheliomas. The plasmacytoid pattern is less

common but the most frequently encountered pattern in palate nlmors. Tle third pattern demonstrates

3 combination ofthc spindle and plasmacytoid cells and is uncommon.

\ot€: Sialoscintigraphy is a simplc and non-invasive procedurc that can usually scparate benign enti-

rres like \\'arthin's tumor and oncocytoma ofthe salivary glands from maligrant tumors, and signifi-aanrlv affcct the course oftrcatment.

Papillar)- cystad€noma lymphomatosvm (afq c! ameforWafthin's tumorl ls almost ex-cltLsir elv a parotid neoplasm. lt is a glandular and cystic tumor lined by a bilayered (inner

.olunindt on.oc)lic and outer basal) epithelium with a lymphoid stroma.

Clinical features: The vast majofity ofpatients are over 50 years ofage, with a 5:l male pre-

dc'minance. Approximately l0To to l5Yo of the tumors are bilateral. The tumor most oftenarises in the lorver pole ofthe parotid and presents clinically as a non-tender' slowly enlarg-

ing. flmr to fluctuant nodule oyer the angle or ramus ofthe mandible.

Histologic features: The tumor is encapsulated and co[tposed ofcystic spaces containing an

.r)sinophilic coagulum into which extend papillary projections of the lining epithelium. The

epithelium consists ofa double row ofcells with eosinophilic, granular cytoplasm, a luminal

lay er oftall columnar cells and a basal layer ofround, cuboidal or polygonal cells. lnterspersed

among the cystic spaces are aggregates of lymphoid tissue, some with germinal centers.

Treatment and prognosis: Surgery is the treatment ofchoice and recurrence is uncommon.

\lalignant vadants ofthe tumor have been reported but are mre.

\ote: Pleomorphic adenomas are the most common parotid tumor lt grows slowly and is

benign. A pleomorphic adenoma begins as a painless lump at the back ofthe jaw just belowrhe earlobe. These are more common in women.

Page 149: Oral Pathology

A 40-year-old female presented with a subcutaneous nodule onthe right latcral surface of her tong\e (afiort is poinrtng tu lesion).

The nodule was non-tender, soft, movable and had been slowlygrowing for about 2 years. The nodule was excised. Its eut surfacewas yellow and lobulated. What is the clinical signilicance ofthe

nodule being movable and slowly growing?

148

CopriSlrr O 20ll ?01? Denral Decks

ORAL PATHOLOGY

A developmental d€fect characterized by an overgrowth of tissu€sNORMAL to the organ in which it arises is a:

. Teratoma

. Choristoma

. Hamartoma

. None ofthe above

't 49

Copyrighl e 201l-2012 - Denral Decks

Page 150: Oral Pathology

Gcncral charactcristics of benign neoplasms:

. Well-diffcrcntiated

. Slow growth

. EncapsulatedAVell circumscribcd

. Localized

Gcncral charactcristics of malignant neoplasms:. Invasion. Immovable. Rapid growth. Metastasis. Not well-differentated (or qnaplqstic)

Important:. Paresthesia is suggestiv€ of metastatic disease. Metastasis is the most important characteristic that distinguishcs malignancy from ben-

ign

Radiographically, a benign neoplasm in bone may be diffcrcntiated ftom a malignant neo-

plasm in the following rvays:

. ln a benign lesion, the cortex tends to remain intact but may be thinncd and thc patinvolved expanded. ln a benign lesion, the margins are usually defined and demarcated lrom surrounding

bone

*** The nodule in the picture on thc front ofcard is a lipoma.

Te rms used in oncology:

. Oncology: the study ofneoplasms

. \eoplasm: an uncontrolled new growth oftissue

. Tumor: a Iocalized swelling, may or may not be a true neoplasm

. Hlperplasia: an increase in the size of a tissue or organ due to an increase in the

nurnber of component cells. H]'pertrophy: an increase in the size of a tissue or organ due to an increase in the

size of component cells. Cancer: a general term for all malignant neoplasms. Carcinoma: a malignant epithelial neoplasm. Sarcoma: a malignant mesenchymal (connective /i.tsae,,) neoplasm. Hamartoma: a developmental defect characterized by an overgrowth oftissues nor-mal to the organ in which it arises. Teratoma: a neoplasm composed of multiple tissues foreign to the organ in whichit arises; may be benign or malignant

Page 151: Oral Pathology

. Metaplastic

. Hwerplastic

. Araplastic

. Dysplastic

150

Cop)r'ghr O 201 l-2012 - D€ntal D€cks

. Cuboidal cells; columnar cells

. Columnar cells; stratified squamous epithelium

. Columlar cells; pseudostratified colurnnar cells

. Cuboidal cells; stratified squamous epithelium

151

Cop)rigltt O 20ll-2012 - Denral Decks

Metaplasia most commonly occurs by rephcementof

-bv

Page 152: Oral Pathology

Lack of differentiation, or anaplasia, is considered a hallmark of malignant transfor-mation. It is marked by a number ofmorphologic and functional changes. Both the cellsand the nuclei characteristically display pleomorphisrn variation in size and shape.

Characteristically the nuclei contain an abundance of DNA and are extremely dark stain-ing (hyperchromatlc). The nuclei are disproportionately large for the cell, and the nu-cl€ar-cytoplasmic ratio may approach I :l instead of the normal 1 :4 or 1 :6. The nuclearshape is usually extremely variable, and the chromatin is often coarsely clumped and dis-tributed along the nuclear membrane. Large nucleoli are usually present in these nuclei.

Histological grading of malignant neoplasms:. An attempt to estimate the aggressiveness of degree of malignancy of a rnalignantneoplasm based upon the degree of differentiation of the component cells and thenumber of mitoses.

- Grade l. Well-differentiated

- Grade 2. Moderately well-differentiated

- Grade 3. Poorly undifferentiatedGrade 4. Undifferentiated

.Applicable mainly to squamous cell carcinomas. Most pathologist use three grades andprefer to designate squamous cell carcinomas as well differentiated, moderately well-dif-ferenriared or Doorlv differenliarea.

\Ietaplasia is the process whereby one cell type changes to another cell ty?e in responseto stress and generally assists the host to adapt to the stress. The most common type ofepithelial rnetaplasia involves r€placement of columnar cells by stratified squamous ep-

ithelium.

Dlspfastic cells exhibit considerable pleomorphism (vqriation in size and shape) and oftenpossess deeply stained (hyperchromatic) nuclei, which are abnonnally large for the size ofthecell. It is associated with chronic irritation ofa tissue by a chernical agent, such as cigarehesmoke or by chronic inflammatory irritation, such as chronic cervicitis. The tissue appears

some\\ hat structureless and disorganized and may consist ofatypical cells without invasion.

Epithelium exhibits acanthosis 6rrr.ll r-t an abnorual thickening ofprickle cell layer).Important: When dysplastic changes are marked and involve the entire thickness ofthe ep-

irhelium. the lesion is considered a pre-invasive neoplasm and is refened to as carcinoma insttu.*** \{ild to moderate changes that do not involve the entire thickness ofepithelium may be

relersible, and with removal of the putative inciting causes, the epithelium may revert tonormal

Histologic features of malignancy:. Anaplasia . Hyperchromatism ' Pleomorphism ' Abnormal mitosis

The host response to a malignancy is best reflected by lymphocytic infiltration at the edge ofa tumor The most characteristic feature ofa malignancy as opposed to an inflammatory Ie-

sion is that a malignancy will grow after removal ofthe causative agent. The most impor-tant characteristic ofmalignant neoplasms, which distinguishes them from benign neoplaslns,

is their abilitv to metastasiz€.

Page 153: Oral Pathology

. Recurrent apthous minor

. Recurrent aphtous major

. Recurrent herpetiform aphthous

. Recurrent herpetic stomatitis

. Stomatitis

. Lesions ofthe eye

. Genital lesions

. Maculopapular rash

'152

CoplriShr O 20ll-2012 ' Dental Declc

153

Copyrighr O 20lt 2012, DentalDecks

Page 154: Oral Pathology

*** Recunent aphthous ulcers are commonly referred to as "canker sores" by 1ay persons- In the liter-afure othcr terms include aphthous stomatitis, rccurrent aphthous stomatitis, rccurrent ulcerative stom-atitis or ulcerative stomatitis.

Three classifications:1. Recurrent aphthous minor: 807o occur on non-keratinized movable mucosa. See picture #92 inbooklet2. Rccurrcnt aphthous major: heal often with scarring, more common in HIV patients. occur on the

soft palate, tonsillar fauces, labial and buccal mucosa, and tongue. See picture #93 in booklet3. Recurrent herpetiform: occur in up to 100 at a time, occur on any mucosal surface, heal withoutscarring. See picture #94 in booklet*** All three classifications present painful recurrent ulcers.

Importanti Vcsiclcs do not precede the formation ofthese painful recurrent ulcers. They appear on wettnot |ennilion) nonkeratinizcd oml mucosa (i.e., not hanl palate).

Rem€mber: Herpetic lesions are preceded by vesicles and are more likely to be found on keratinizedtissue. See picture #95 in booklet

\ote: Systemic diseases in which aphthous ulcers are seen include: Crohn's disease /rarnor aphthae),Be-hcet s syndrome ftnln or aphthae), Celiac sprue (minor aphthae). and AIDS (maior aphlhae).

These ulce6 appear to be associated with stress. The stress factors may include:. Bacterial infection. Trn[fia li.e., self-i ticted, oral surgen, procedures, routine dental procedures). Endocrine conditions (i.e., a lemales menstrual period). AIIergic factors fi.e., ceittain foods or drugs). Immunologic abnomalities. lr,Jn- Viramin B or folic acid deficiencres*** The calse is unknown; however, evidence supports they are related to the focal immune dys-function where T-lymphocytcs play a major role

Stevens-Johnson syndrome fS-/S/ is an immune-complex-mediated h)?ersensitivity com-plex that is a severe expression of er''thema multiforme. It is now known also as erythemamultiforme major ln SJS, the systemic symptoms are sever€ and the lesions are €xtensive,inr olling multiple body areas (especiqlb, the mtrcous membra es).

Tl?ical s\ mptoms are as follows:. Cough productive of a thick purulent sputum. Headache. \lalaise. Arthralgia

The tvpical " bull's-eye-shaped" target lesions are present. These lesions are consideredparhognomonic.The classical triad ofthis SJS consists ofeye lesions, genital lesions andstomatitis. lmportant: The lesions ol SJS are severe and often vesicular or bullous,r ith hemonhage after denudation. Note: Blindness can occur due to secondary infection.

Treatment of SJS is primarily supportive and symptomatic:. Vanage oral lesions with mouthwashes. Topical anesthetics are useful in reducing pain and allowing the patient to take in flu-rds. Areas ofdenuded skin must be covered with compresses ofsaline. Underlying diseases and secondary infections must be identified and treated. Offending drugs must be stopped

Drugs and malignanci€s are most often implicated as the etiology in adults and the eld-erly. Pediatric cases are related more often to infections than to malignancy or a reactionto a drus.

Page 155: Oral Pathology

ORAL PATHOLOGY Ulc Cond

The lesion below is a. smnll (2mm-Smm in diameter), whitish sore with a redborder. The patient strtes that it usually begins as a reddish area with a

trurning or tingling sensation. The most likely diagnosis is:

154

Copyrighr lil20ll 1012 ' Denlal Deck!

ORAL PATHOLOGY Ulc Cond

nrythema multiforme (EM) is an acute self-limited eruption characterizedby a distinctive clinical eruption, the hallmark of which is the:

. Chronic desquamative gingivitis

. Petechial hemorrhage

. Iris or target lesion

. Mucocutaneous rash

155

Coplrrght O 20ll l0l: Denral Decks

Page 156: Oral Pathology

There are three forms or classifrcations ofrecunent aDhthous stomatitis fuhich is alsocalled recurent aphthous ulcers, canker sores, etc.)

l.Recurrent aphthous minor: is the most common form of the disease and theone referred to by the lay public as the "canker sore." The lesions occur somewhatmore frequently in women than in rnen. Minor aphthae have a propensity for movablemucosa that is situated over minor salivary gland tissue. The lesions begin as a singleor multiple superficial erosion covered by a gray membrane. The lesion is typicallyvery painful. The lesions vary in size from 2-3 mm to over l0 mm in diameter Theygenerally persist for 7- l0 days and heal gradually with little or no evidence of scar-

ring.2. Recurrent aphthous major: is characterized by the occurrence of large(5-20 nm) painful ulcers, usually one to ten in number. These ulcers occurat frequent intervals and rrany patients with this disease are seldom free from the pres-

ence ofat least one ulcer Unlike the minor aphthous ulcer, these lesions persist for upto six weeks and leave a scar upon healing.3. Recurrent herpetiform: the prominent feature of the disease is the numerous,pinhead-sized, gray-white erosions that enlarge, coalesce, and become irregular ulcers.Ulcers occur in clusters of 10 to 100.

Remember: Vesicular lesions do not precede the formation of ulcers in all of theabor e. This is a distinctive diagnostic feature.

\ote: In healing ofan ulcer, the epithelium that eventually will cover the defect is derivedliom intact epithelium al the ulcer margin.

Erlthema multiforme frMl is a hypersensitivity reaction that occurs in mild and scvere forrns ltproduccs tissue reactions ccntered on the superficial vcsscls of lhe skin and mucous menbranes.

Pre.ipilating iactors includc infcctions such as herpes simplex, mycoplasma pneumonia and histo-plasmosis. dmgs, radiation therapy. etc. lt occurs primarily in children and young adults. The di-

a!:no5is oferythema multilorme is primarily based on the classic skin lesion appearance.

Clinical features; 3 forms:. E\1 minorl prirnarily involves thc skin with only 2570 in oral mucosa. A low-gradc fcvcr, gcn-

eral malaise and headache usually precede thc appearance ofthe lesions by 4 to 7 days. Assocl-

ared * ith secondary herpes simplex hypersensitivity. Oral lesions appear as .cd macules, papules

or \ esicles that may become erodcd and painful. Note: These lesions are covercd by a yellow-ish-\t hitc membnne aftcr rupturing. Focal or diffuse arcas oferytherna follow and the skin "tar-gct" or "bulls eye" lcsions appear See picture #69 in booklet

. Chronic ENI Minor: mildcst foml and lesions are small in sizc and shortcr in duration. Oral

lesions rary from focal erosions similar to aphthous ulcers to more dilluse areas of erythema orerosions that are painful.

. Elu Nfajor lterers-Johnson Syndrome): acute form that involvcs the skin and mucous Ineln-

brane. Large bullae form on the mucous membranes and skin. A positive Nikolsky's sign is com-

mon. The bullae collapsc which produces a whitish pscudomembrane on the tnucosa and dark

red crusted lesions on the dry skin surfaces (lips and eyes very common).

Note: Behcet's syndrome is a rare disorder that causcs chronic inflammation in blood vessels

rhroughout the body. Manifestations include oral and genital aphthous-type ulcers. conjunctivitis,uveitis, arthritis, headache, and other CNS symptoms. Treatment with corticosteroids and other im-munosupprcssive drugs prcvents serious complications, such as blindness.

Page 157: Oral Pathology

. Coccidioidomycosis

. Histoplasmosis

. Tuberculosis

. Actinomycosis

. Scarlet fever

. Syphilis

. Gonorrhea

. Chlamydia

. Tuberculosis

156

Copfight O 2011,2012, D€ntal Decks

't57Cop)ri8hr O 201 I '2012 - Dertal D€cks

Page 158: Oral Pathology

Actinomycosis is a subacute{o-chronic bacterialinfection caused byActinom)ces israellii, an anaerobic, graln-pos-

ilive filamentous bacterium. The infection is not a contagious discasc and can not be spread fron person lo pcrson.

Inlcctions appear after trauma! surgery or previous infection.

Clinical fcatures:

. \4ost infections occur in thc lhorax. abdomen and head and neck - called cervicofacial actinomycosis /i,fection n'picall.v occurs i puti?ntsIi!hpoorlental hryiene or lilloti gorc| surgen). oftcn prescnts as a swelling ofthe nandible and usually is painful. Once in the tissue, Actinomyces forms an abscess, prcducing a hard. red-to-reddish-pu4)le Iump, olien on thejaw

- "lump) jalr" *** See picture #70 in booklet. Exudatc from thc draining sinus tracts often contain small, clinically visiblc ycllow-green calcified structures(sulfur grunules)

-\\hich arc adually colonics ofinficting organisms

*+* Treatnrent for aclinomycosis is long-term penicillin.

Histoplasmosis is a chronic lung infection caused by inhalalion ofspores of Histoplasma capsulatum. It is en-

dcmic ro rhcAmerican Mid\r'esr. The €lassic oral manifestation ol histoplasmosis infection is a chronic non-healing ulcer. Note: The lesions are usually covercd bv a non-specifrc gray membranc and are induratcd.

Clinical features:. Fe\er. malaisc, cough, and dyspnea. Ccrvicll lymphsdenopathy is common. In chronic fonns, dissemination to the skin and oral mucous membrancs may be the first sign ofinfec-

"** Treatmenr for histoplasmosis is an)photericin B, itraconazolc or ketoconazole for 6-12 months.

Coccidioidomlcosis (also ktlo\n as Mller lever) results fron inhaling the spores olCoccidioides species

'C,iLtJi)id!\ i,tmilis or Coccidioicles posada';ir. Most infections in the United States arc acquired within the

l|: ,,: reuions ofcndcmiciq ofsouthern Arizona, centfitl or othcr arcas ofClalifornia. southcm Nc$'Mcxico,::rc \.i: Tc\as. Symptoms are similar to histoplasmosis and it is usually treated $ iih amphotericin B.

\otc: Scarlet feier is a systemic infection causcd by Strcptococcus pyogenes. It i! characterized by pharyn-

irirs. l!!cr. malaise, strawberry tongue (hos d :||hile coating with red, inflaned fungiforn pupillae) and ^

ikr:r :ash Important: The p!'rogenic (et_rlhrogeni() e\?toni,'? causes the rash ofscarlet fevcr and systemic

:.,\ ta ihock slndrome.

Srphilis is an inf'cctious. contagious venereal discase that is caused by the spirochetc treponemapallidum. It is one of the less common scxually transrnitted infections /S11/. The incubation pe-nod \aries some$.hat bctween 10 and 90 days. Thc sylnptonls ofsyphilis may pass unrecognizcd,!rr Dra\ be nlisintcrpleted and at times there arc no initial symptoms at all. Whcn prcscnt. thc clas-irJai s\mptoms ofsyphilis lnanifest themsclves in threc stages as lbllows:

L Primarr': the first symptom is a non-painlil chancre that generally appears 2-6 weeks aliere\l-r.)sure. lt usually is fbund on thc part ofthe body exposed to thc partner's ulcer. such as thctL'ni\. thc \ul\a, or the vagina. lt can also develop on thc cervix, tonguc. lips. or othcr parts oflhe brrd) The chancrc disappears within a l'erv wccks whethcr or not a penon is trcated. Ifnotlrcatcd during thc primary stagc. about one third ofpeoplc will progress to chlonic stages.

i. Secondarl: is a highly infectious stage; it occurs 6 weeks aftcr non-treatment of primary:r philis. \\'idely dissetrinated spirochetes cause mucous lnerlbranes 10 cxhibit a reddish brownnraculopapular cutaneous rash and ulcers that are covered with a nucoid cxudate ku/led mu-.ot,r pdtches). Condylomata lat^ (\'hich are elevated bntad-based pluquesl are also seen onikln and mucosal sr-rrl'aces. lf left untrcatcd, thesc symptoms will rcsolve on their oq,n but thciniictious microbc remains behind. It is at this point that syphilis passes into its latent phase.This \ilent period ntay last fbr many ycars and permits the infcction to evolve wi{hout any ob-\ rous cxtemal symptoms. At this point. the only nrcthod ofdctecting the prcscnce ofsyphilis is\ i r., blood lc.l spccilic lor syphrlis.

.l. Tertiary: occurs in inf'ected persons many years alier non-trcatment ofsecondary syphitis. Thcgumma (*,lrich is a .fotal nodular nlasr) typifies this stage. lt most cornmonly occurs on lhepalate and tonguc. The bacteria damage the hcart, eyes, brain, nervous svstem, bones.joints, oralmosl any othcr part ofthe body. Note: Headache, stit'fneck, and f'ever are symptorrs of neur-os)philis.

Remember: Conorrhca is a sexually transmitted disease caused by Neisseria gononhoeae. An oralmanifestation of gonorrhea is oral pharyngitis.

Page 159: Oral Pathology

. Coccidioiodomycosis

. Mucormycosis

. Aspirgillosis

. Zygomycosis

158

CoplriShr @ 201|,2012 - D€ntal Deck

'| 59

Cop)'righr O 201 I -2012 - Dental D€cks

Page 160: Oral Pathology

Congenital syphilis is caused by infcction with thc spirochctc Treponema pallidum during thefetal period. Expectant mothers who have syphilis can transmit the disease through the placenta tothe unbom infant. Nearly halfofall infants infected with syphilis during gestation die shortly be-forc or after birth.

The severity ofcongenital syphilis depends upon l) The time in which the organisn]s pass the pla-cental barier (protected up to I6th week), 2) The mother's stage of syphilis and, 3) The immuno-logic rcsponsc ofthe fctus. Iftrcated by the 4th or 5th month, 957n show no manilestationsi ifnotfeated, fetal sepsis may result in stillbirth or visceral and mucocutaneous manifestations.

l. Symptoms in the newborn.lrritability. Failure to thrive. Bloody discharge from the nose. Saddle nose, lrontal bossing and short maxilla. Later rash: coppcr-colorcd, vcsicles on the palms and soles. Early rash: small blisters or a flat bumpy rash on the lbce. palms and sole

2. Symptoms in older infant and young child. Bone pain. Joint swelling. Abnormal teeth fHrbhinson's incisors). Saber shins r'borre abnormalitT- qfthe lower leg). Gray, mucous-like patches on the anus andy\tlya (cond),lona lata). Visual loss. CN VIII nerve deafness and intentitital keratitis. Scaning of the skin around earlier lesions of the mouth, genitalia, and ants (called rha-gctdesl

Remember: Hutchinson's triad -the

combination of Hutchinson's teeth, interslitial keratosis,and deafness due to lesions ofthe eighth cranial nerve. Note: "Screwdriver" incisors and "Mul-berry molars," dcntal defects seen in congenital syphilis are caused by direct invasion of toothgerms by Treponema organisms.

The gencra most commonly rcsponsible for mucormycosis usually are Mucor or Rhizopus. Orbito-rhrno-cerebral mucormycosis, the most common typc, genemlly occurs in conjunction rvith sinus

rrr nasal involvement. Mucormycosis also may aflect other parts ofthc body, including the lungs,

GI tract. or skin.

Diabetic patients arc predisposed to mucormycosis because of thc decreased ability of thcir neu-

rrophrls 10 phagocytize and adhcre to endothelial walls. Furthermore, the acidosis and hyper-glycemia provide an excellent environrnent for the fungus to grow.

Othcr patrent. at risk include thc follou ing:. Patients on chronic antibiotics, steroids, or c),totoxic therapy. Palienls with chronic renal failure or liver problems. Parients with transplants. Palients with cancet HtV, malnutrition or acidosis

Important: In the head and neck, most lesions appear as destructive ulcerations in the paranasal

sinuses or nasal cavity.

The use of systemic amphotericin B is important in treating mucomrycosis; its use, along with in-crcased awareness ofthe disease, has decrcascd thc mortality

Rhizopus is the principal cause ofzygomycosis, which occurs primarily in patients suffcring fromdiabetic ketoacidosis (thinocerebral diseasel, malnutrition, scvcrc bums, or who are itnmuno-compromrsed.

Aspergillosis is a group ofillnesses caused by mold. In some people, the spores trigger an allergicreaction. Other people develop mild to serious lung infections. The most serious form of as-

pergillosis invasive aspergillosis occu6 when the infection sprcads to blood vessels and be-yond, into the lungs to othcr organs. Note: Aspergillosis is the second most frequently seen f'ungalinfection ofthe f'ace and mouth in patients receiving chemotherapy. It is second only to candida inits fiequcncy. Oral lesions appear as necrotic ulcerations with gray pscudornembranes ofthe gin-siva and hard palate.

Page 161: Oral Pathology

A 4-year-old pti€nt comes with h€r mother for a routine appointment. Themother states that her d.ughter just started not f€eling well and had a mildfever earlier in the day. Th€ daughter has be€n having troubl€ swallowing.

An intraoral exam reveals multiple 1-2 mm vesiculopapular lesions ofthenasopharynx and soft palate. Your working diagnosis is:

. Herpangina

. Hand-foot-and-mouth disease

. Herpes simplex infection

. Pemphigus lrlgaris

'! 60

Coplrigbr iO 20ll-2012 , Denul Decks

ORAL PATHOLOGY

Which type ofherpes virus is associated with the lesion on the lower lip?

. HSV- l

. HSV-2

. HSV-3

. HSV-4

161

CopFighr C 201 l-201 2 - Denral Decks

Page 162: Oral Pathology

Herpangina refers to a stomatitis (inflammotion ofthe mouth) caused by a strain oltheCoxsackie- virus. It is differentiated in clinical practice from Type I Herpes infection (the

cold sore virus) by the fact that the ulcerations ofherpangina are localized to the poste-rior soft palate and nasopharynx. Remember: Herpes Type I lesions are found typi-cally more forward in the mouth on the tongue, gingiva, buccal mucosa and appear as

vesicles (smal/, cleqr blisters that ulcerate and crust) around the mouth and on the lips.

Clinical features:. Most commonly affects infants and young children. Typically occurs during the summer. Typically spreads via the fecal-oral route or via the respiratory droplets. Symptoms are mild and short in duration (no more than I v'eek). Sore throat and difficulty swallowing. Mild fever. Small vesicular or punctuate lesions with white base on posterior soft palate near

uvula and anterior fauces ofthe tonsils *** See picture #71 in booklet

Important: The disease usually runs its course in less than a week. The treatment is pal-liative.

\ote: Hand-foot-and-mouth disease, is a highly contagious systemic infection caused

coxsackievirus A of limited duration in which vesicular eruptions occur on the palms ofhands. soles offeet and mucosa ofthe anterior part ofthe mouth. It is uncommon in the

oropharyngeal are a (unlike herpangina).

*** Remember: The majority oforal herpes cases are caused by HSV-l and the majority ofgenital herpes cases are caused by HSV-2.

Oral and para-oral presentations of Herpes Simplex Type I include;

. Herpes fabialis (also called fever blisters or cold sores) is an extremely common disease

caused by the herpes simplex virus Type I . charactedzed by an eruption of small and usu-

aLll painful blisters on the skin ofthe lips, mouth, gingiva, or the skin around the mouth.

\ote: The reason most patients suffering from recunent herpes labialis rarely give a history

of having had actne herpetic gingivostomatitis is that the pdmary infection rvas subclinical.

. -{cute herpetic gingiyostomatitis (also known as primary herpeti. gingivostomqtltis)generall-v'. affects children under the age ofthree and young adults. There are prodromalslmptoms (,fever, malaise, irritabiliry, headache, clysphagia, vomiting, b,mphadenopdth)')l -2 days prior to local lesions. Then small, yellowish vesicles form, which rupture quickly.

resulting in shalloq round, discrete ulce$ with an eDthematous halo. It affects both the free

and attached mucosa. A generalized marginal gingivitis may prccede the ulcers. Treatment

includes fluid intake, good oral hygiene and gentle debridement ofthe mouth. ln healthy in-

dividuals the lesions heal spontaneously in 7- l4 days without a scar.

. Recurrent /secord4ry, herpetic stomatitis generally occurs in adult patients and is trig-gered by trauma, fatigue, immunosuppression, stress, allergy or sunlight, which causes the

release (or rcdctivalioz) ofthe latent HSV-l virus in the trigeminal ganglion. This reacti-vation causes a recurrent infection f.e., told soles) on the lips (that is bound to periost-euml, hard palate, attached gingiva and alveolar ridge. Site-specificity is a characteristicmanifestation. Note: Lesion on the finger is called herp€s whitlow.

Page 163: Oral Pathology

. Cytomegalovirus

. Epstein-Barr virus

. Herpes simplex virus I

. Herpes simplex virus 2

. Lipschutz bodies

. Civatte bodies

. Lisch nodules

. Reed-Stemberg cells

162Cop)righr O 201l-2012 - Dental Decks

163

Coplright O20ll-2012, Denhl Decks

Page 164: Oral Pathology

Primary herpes is most common in children and young adults. Patients develop fever. ir-ritability, regional lymphadenopathy and headache. Within days. the gingiva becomes in-tensely inflamed. Any pan of the oral mucosa and lips may become involved. Vesicles

then form and rupture a short time later to leave shallow ulcers covered with a gray mem-

brane and surrounded by a red halo. These ulcers are very painful. These ulcers will heal

on their own within 7 to 14 days.

After recovery from primary HSV infection, the virus is not cleared from the body, but,

rather. it lies dormant in a non-replicating state, in the sensory nervous system (specili-call,r, the trigeminal ganglion/. Periodically, Iatency reactivates. allowing the virus toretum to the skin or mucous membranes, where it causes a recurrent inlection. Cold sores

are a manifestation ofrecurrent herpes simplex virus infection around the mouth. The

lr.rost common site is on the lips. Some factors that are often associated with a recurrent

outbreak are: sunbum, fatigue, emotional upset, trauma, upper respiratory tract infectionor menstruation. Often a day belore the formation of vesicles there will be a tingling oritching ofthe skin or mucosa. Vesicles ulcerate and resolve the same as in the primary in-i-ection.

Histofogically, the cytopathic effect (CPE) take the form ofballooning degeneration ofrhe epithelial cells with loss of cohesion to adjacent cells. The nuclei are olten multiple*ith margination ofthe chromatin around the intra-nuclear inclusions called Lipschultzbodies. These changes can be seen in scrapings taken from an unroofed vesicle (tlrcse

.rcrapings are callerl a Tzanck smear).

The Tzanck smear is done by smearing cells taken from a fresh blister or ulcer onto a

mrcroscope slide. The cells are stained with a special stain, such as Wright's stain, and

then examined under a microscope for characteristic changes caused by a herpesvims.

Herpes causes giant cells with multiple nuclei. The shape ofeach nucleus appears moldedro t'it together u ith those adjacent. The background ofthe cell looks like ground glass and

contains snall dark spots called inclusion bodies (Lipshutz bodies).

Orher tests that can be used to diagnose herpetic lesions include:. Fluorescent staining: cells show positive fluorescence when stained with fluor-escent labeled HSV immune serum and globulin. This procedure is used to distinguishbenr een herpes zoster and heryes simplex..Isolation in tissue culture. Antibody titers (anti HSV ontibod!- titers)i is a test for complement fixing or neu-

tralizing antibody in acute and convalescent sera as well as on tissue sections (this be-gins irt one v:eek arul peaks in three veeks).

Biopsied material will show an intraepithelial cleft covered by an exudates offibrin andpolyrnorphonuclear leukocytes. The epithe|um willexhibit degenerative cells, which in-clude bizarre giant cells and cells with displaced chromatin with perinuclear halos and

inclusions.

The treatment for herpes is primarily supportive:. Analgesics. Topical anesthelics prior to eating. Maintain electrolyte balance. Antiviral agents

Page 165: Oral Pathology

. Herpes zoster

. Herpangina

. Recurrent herpes

. Chicken pox

tuCopynghl O 201l-2012 D€ntal Decks

. Manifested as ANUG

. Associated with HIV

. Subclinical

. Characterized by severe lymphadenopathy and acute dermatitis

165

CoprrightO 20l l-2012 - De al Deks

Page 166: Oral Pathology

The varicella zoster virtrs (VZV) is a member ofthe herpes virus group. It causes the dis-ease chickenpox (varicella) and shingles (herpes zoster). The virus is very contagious and

may be spread by direct contact or droplets.

Chickenpox is primarily a disease of childhood, which peaks at school-age in the winterand spring. lt is characterized by the appearance on skin and mucous membranes ofsuc-cessive crops of typical pruritic vesicular lesions that are easily broken and become

scabbed. lt is generally accompanied by mild constitutional symptorns (/everr malaise).ltis most contagious one day before the onset of the rash and until all the vesicles have

crusted. It is relatively benign in children, but adult infection may be complicated by pneu-

monia and encephalitis. Note: ZIG (Zoster Immune Globulin) reaches morbidity in high-risk children.

Shingles (herpes zosle, is the result ofreactivation ofa latent varicella-zoster virus that

is believed to reside in the sacral ganglia from a childhood case ofchickenpox. The virusreaches the sensory ganglia ofthe spinal and cranial newes, producing an inflammatoryresponse. It is characterized by painful vesicles that occur on the skin or mucosal surfaces

along the distribution ofa sensory nerve in a distinctive unilateral pattern.

\ote: The histology for both chickenpox and shingles shows the same cytopathic effectas seen in herpes simDlex.

Herpes simplex is one ofthe most common viral diseases affecting man. The primary in-fection. rvhich is known as primary herpetic gingivostomatitis, is most common inr oung children /rin der live yeors old). It usually occurs in a child who has had no contactu ith the Type I herpes simplex virus, and who therefore has no neutralizing antibodies.

h mal also affect young adults //J-25). Nearly all primary rnfections (90o/o) are ofthe sub-clinicaf type (they moy only have flulike symptoms) and one or two mild sores in the

mouth lvhich go unnoticed by the parents.

In other children, the primary infection may be manifested by acute s)'rnptoms, which is

knorvn as acute herpetic gingivostomatitis. These symptoms include feveq initability,cen ical lymphadenopathy, fiery red gingival tissues and small yellowish vesicles that

mprure and result in painful ulcers on the free and attached mucosa. The most serious po-tential problem in a child with this infection is dehydration due to the child not wantingio eat or ddnk because ofthe pain. See picture #72 in booklet

The treatment is supportive and aimed toward the reliel of the acute symptoms so that

fluid and nutritional intake can be maintained. Primary herpetic gingivostomatitis usu-

ally runs a course of l2-20 days, and the ulcers heal without scaning.

\ote: Corticosteriods are contraindicated in patients with herpes simplex infections.

Page 167: Oral Pathology

Disv-B

After the initial primary attack during the early childhood period,the herpes simplex virus remains inactive most comrnonly in the:

. Geniculate ganglion

. Ciliary ganglion

. Trigeminal ganglion

. Pterygopalatine ganglion

166

Coplright O 20ll 1012 Denral Decks

ORAL PATHOLOGY V-B Dis

A 49-year-old patient ofAshkenazi heritage presents to your oflice complalnlngof "blisters in her mouth.' Your intraoral exam shows ulcers present on multipleareas of mucosa. She also related to you that while getting out ofthe car earlier,her skin ofher arm rubbed against the car door and tore. Known as Nikosky's

sign, this phenomenon is associated with which disease?

. Herpes zoster

. Lupus erythematosus

. Lichen Planus

. Pemphigus

t67Copyright,e 201 l-:012 Dental Decks

Page 168: Oral Pathology

The inactive herpes simplex virus resides in sensory nerve ganglia (mo.st commotll!', the

trigeminal gangliol, but will often reappear later as the familiar "cold sore," usually orr

the outside of the lips. This disease is referred to as "recurrent herpes labialis." Emo-tional stress, trauma and excessive exposure to sunlight have been implicated as factorsfor the appearance of the recurrent herpetic lesions on the lip. Acyclovir 57o ointment(Zovirax) as well as valacyclovir, and famciclovir have been successful in reducing the du-ration and severity ofthese sores.

Remcmber:I . Herpes Simpfex Type I (prinary; herpelic gingirostomLttiti.t, recuftent herpes labi-dlrt is transmitted by direct contact. It affects the lips. face, skin and oral mucosa.

2. Herpes Simplex Type ll (herpe.s geuitalis/ is spread by sexual contact. It affectsthe mucosa ofthe genital and anal regions.

\ote: Cenital herpes may have serious consequences in pregnant women because ther irus can be tnnsmitted to the infant during vaginal delivery. The virus can cause dam-age to the infant's central nerr,'ous system and/or eyes.

Remember: The primary infection of herpes simplex can range from subclinicalIQs\tnptonqtic, v'hich is most common) to severe systemic infections.

Penrphigus (an autoimmune disorrler) is a term used to describe blistering ofthe skin caused

b1 binding ofantibodies to the surface ofthe cells ofthe outer layer ofthe skin, the epider-

mis. In pemphigus yulgaris, the most common form ofpemphigus. antibodies are directed

against rhe desmosomal adhesion molecule Dsg3, resulting in severe mucosal erosions and

eprdermal blistering in areas such as the mouth. As a result, patients develop severe oral ul-cerations. and may also have inflammation or erosions of the lining of the eye and eyelidsI conitol(tiya), the nasal mucosa, or the genital mucosa. Half of the patients also develop b)is-

rers or erosions ofthe skin, often in the head and neck area. It seldom occurs before the age

oi j0 rususlly benteeu 30 and 50) and occurs more frequently in Jewish people.

There are four types ofPemphigus:L Pemphigus vulga s: most commonL Pemphigus vegetans

3. Pemphigus foliaceus4. Pemphigus erythematosus

Oral lesions are often the first manifestation ofthe disease. Intact bullae are rarely seen in the

oral cavit,v. instead, large areas ofulceration and erosions are often seen that are covered by au hite or blood+inged exudates. Sometimes, areas of epithelium will slide ofl simply by rub-

bing of an apparently unaffected area (this is 1e7-lt?ed Nikolsky's sign/. This disease is often

fatal without therapy, which includes high-dose systemic steroids or chemotherapy /br e-r-

drp1e, methotr€xate/.

Important histological features: The vesicles and bullae are fbrmed entirely intraepithelially,just above the basal layer of cells (calletl suprabasilar veslcleJi. There is intercellular edema

and loss ofintercellular bridges with loss ofcohesiveness. This is called acantholysis. Clumpsof cells are often found floating free in the vesicle space (tieve cells are called Tzanck cells).

Page 169: Oral Pathology

. Candidiasis

. Hairy leukoplakia

. Desquamative gingivitis

. Hemorrhagic mass

168

Copt{ight O 20ll-2012 - Denhl Decks

. Human papillomavrrus (HPV)

. Adenovirus

. Epstein-Barr virus

. Parvovirus

169

Cop'-i8hI O 201 I '2012 - Dental Decks

Page 170: Oral Pathology

Benign mucous membrane pemphigoid (BMMP) is an autoimmune or "self-allergy" dis-ease in which a patient's own circulating antibodies become altered so that they attack the

fibrous attachment of the skin and membrane epithelium to the underlying connective tis-sues. Women are more commonly affected than men and the disease is usually diagnosedbetween the ages of 40-60 years. The typical lesion is a small or large, clear-fluid blisterwhich breaks fairly rapidly in the mouth to leave a flat white, somewhat tender ulcer witha thin red line around it. The gingivae are especially likely to be involved, resulting insloughing during eating or tooth brushing ("desquamative gingivitis"). Systemic steroidtherapy has provided adequate management of BMMP Note: Conjunctival involvementmay lead to blindness.

Rememb€r: Nikolsky's sign, which is an indication ofpemphigus vulgaris, nay also be

lound in BMMP The sign occurs when apparently normal epithelium may be separated

at the basal layer and rubbed offwhen pressed with a sliding motion.

While similar in its clinical presentation to pemphigus vulgaris, it is much less severe

and the involved antibodies attack the attachment fibrils (Type VII collagen) ofthe base-

ment membrane antigens fi.e., Laminin 5 and BP 180), rather than the desmosomal at-tachments (desmoglein 3 [Dsg3] ) between epithelial cells, as occurs in pemphigus.

Important: Histologically, the major difference between BMMP and pemphigus r,ulgaris

is that the vesicles in BMMP are subepidermal and there is no evidence of acantholy-sis - in pemphigus vulgaris there is acantholysis and a suprabasilar vesicle,

This is the same virus that causes genital wans which lcads to dysplasia and cervical cancer. Some estimates ofthe prevalence of IIPV infection in the population range as high as 79%. Warts may arise on

anv skin surface, bnt occur most commonly at acrd (peripheral) sites. Although treatment may removerhe \\'art. the virus remains latent within the skin cells. However, treatrnent may diminish spread ofHPVin the skin ofthe infected patient and possibly to uninfccted contacts.

. Papillomas:. Benign epithelial prolifefttions (pedu culated or sessile) of \ftle significance.. \trruca vulgaris (also called squamous papillona) has an incubation period from about six\\'eeks to a ycar. Although it is primarily a lcsion ofthe skin, it may occur in the oral cavity, partic-

ularly on the lips and palate. Clinically, it is a sessile, soft, cauliflower-like lesion. Ifexcised, theyusually do not recur, but autoinoculation is possible. Note: Intraorally, that is how most cases dc-vclop. Much more prevalent in HIV positive patients

. Condyfoma acuminatum (ge ilalwa s): Caused by HPV subtypes 6 and l1; oral lesions acquired

b) oral-genital contact; broad-based verruciform lesion.

. Focal epithelial hyperpl^si^ (Heck\ dise4se,): Most common in ethnic groups fi.e., Nat[re Artel-icans ond Centtul Anlencdrt. Multiple small, dome-shaped warts on oral mucosa. Caused by HPVsubttpes 13 and 32.

Important: Sexually transmitted, high-risk HPVS include types 16, 18, 31, and 33. Tlese are the HPVt] pes associated with cancer.

Verrucr vulgaris on the tongue

Page 171: Oral Pathology

V-PLes

A 45-year-old female walks into your olfice complaining ofa "wart" on hergums that has been th€re for y€ars. Your exan reveals an asymptomatic,well-circumscribed, slightly raised, papillomatous lesion on the buccal

gingiva of tooth #5. A likely diagnosis of this is:

. Fibrosarcoma

. Neurosatcoma

. Lipoma

. Verucifonn xanthoma

170

CopyriSlrr O 20ll ?01: - Denral Decks

ORAL PATHOLOGY V-PLes

A 63-year-old completely edentulous patient comes into your office becauseher d€ntures hrve "finally gott€n too bothersome to wear.' Eer health history

consists of COPD and cigarette 40-year pack history. Your intraoral examreveals a noxious odor and an ill-litting upper denture. When you remove theupper denture you note multiple red, papillary projections ofthe hard palate.

Your pati€nt states she does not remoye her dentures at night or tretween meals.After reviewing denture hygiene instructions, you give her the diagnosis of:

. Epulis fissuratum

. Papillary hyperplasia

. Nicotinic stomatitis

. Kaposi's sarcoma

171

CopriSht O 201l'?0ll ' Denlal Decks

Page 172: Oral Pathology

This unique Iesion occurs in middle-aged individuals /alera.ge a.ge i.s 45 t t'ars). Thc usual intraoral lo-cations are the gingiva and alveolar mucosa, but any oral mucosal site may bc involvcd. Thc lesion ap-pears as a rvcll-circurnscribed, slightly elevated mass with a papillomatous or vcrrucous surfacc.Thclevel of keratinization of the surface will influence its color. which ranses fiom \1hit(r or red.

Well-circunNcribed, lobulated, soilyellowish mass; vessels visible oversuiface; buccal mucosa, floor ofmouth. long!€ common srtes

Surgery or radiationiprognosis is lair to good.

Rarely encountercd in soft tissues ofthe head and neck. Yellowish submu-cosal mass. Slow-growing malignancy.

Adnlts (average age is ,l5r; gingiva &alveolar mucosa are most commonsites; well-circumscribed, slightlyftised, papillomatous or verrucoussurface

Wide surgical excision;reculTence not common

When in bone,lesionma), arise fiom perio-stcum. endosteum orPDL. When in sofltrssue. fibroblasts.

Rare sofl tissue and bony malignancyofthe head and neck. Young adults are

most commonly affected. Infi ltrativeneoplasm that is locally deslructive.

Eithcr from a

pre-existing lesion ofneurofibronlatosis ordl: ,,o'r,. Thought tobe from Schwann cell.

Rare malignancy; Pain or paresthesiamay accompany lesion in bone. ln sofltissues, it appgars as an expansile massthat is usually asymptomatic

Widc surgical excision;Recurrence is conmon,metastases are fiequentlysccn. Prognosis is fair iogood

f,nrity Etiolo$ LocationClinical

CharacteristicsTreatment

lrd Prognosis

De.nire-induced fi broushlpcrplasia, I t'll a|""at ory b,peryla si a,d?nhrc h\perplasia, attd'Epulis lissvtatum")

lll-fitting prosihesis Common lesionthat occuis in the

flange conects

I'ainless folds offi'

overextended dentureflange

Surgical excision

prosthesis withpossible remaking

relining d€ntur€s

Papillar,v hyperplasiar _Palatrl papillomatosis!')

Poor oral hyglene &ill fitlinf prosthesis

Hard palatepapillary projections.

Surgical cxcision

l. There is no malignant potential to any ofthe librous hyperplasias.\otes 2. The peripheral fibroma is a reactive hyperplastic mass that occun in the gin-

giva and may be derived from connective tissue ofthe submucosa or the PDL. Itprcsents as well-demarcated focal mass with either a sessile or pedunculated base.It is similar in color to the surrounding connective !issue. [t may be ulcemted. Thetreatment for a peripheral fibroma is local excision. Recurrence is rare.Note: Other variant forms ofthe peripheral fibroma include the peripheral ossi-iying fibroma and the peripheral odontogenic fibroma.3. Focal fibrous hvperplasia is hyperplasia oforal mucosa. Tt is also called tmu-matic fibroma, irritation fibroma, and hyperplastic scar. lt is a reactive lesioncaused usually by chronic tmrrma to oral mucous membrane. See picture #23 inbooklet. The giant cell fibroma is a focal fibrous hyperplasia in which connectivetissue cells, many ofwhich are multinucleated, assume a stellate shape.

Page 173: Oral Pathology

The picture below shows a benign epithelial neoplasrn which appears as ap€dunculated, whitish cauliflower-like mass on the lower lip.

The most likely diagnosis is:

172

coprighl a(, 201 1-2011 - Dental Decks

ORAL PATHOLOGY

A 54-year-old African American female presents to your clinic for an initialexam. She has a history of hypertension controlled with beta-blockers but noother contributory health lindings, Your intral oral exam reveals a bilateral

lilmy opalescence ofthe buccsl mucosa. When stretching out h€r cheeks,this white hue disappears. Your most likely diagnosis is:

. Squamous cell carcinoma

. Fordyce granulation

. Leukoedema

. Leukoplakia

173Cop)right C 20ll-2012 - Dental Decks

Page 174: Oral Pathology

The paprlloma is a benign exophytic papillary growth of stratified squamous epithelium.Note: The common wart, or verruca vulgaris, is a lrequent tumor of skin analogous tothe oral papilloma.

Clinical features:. Adults. Anywhere on the oral mucosa. Sessile or pedunculated exophytic growth. Papillary ftauliflowerJike) appearance. Long duration. May show considerable keratin and in some instances appear white clinically. If ithas little keratin on the surface it will appear pink.

Treatmenl and Prognosis: Conservative excision: recurrence is rare.

Rememtrer: A fibroma is a benign neoplasm of connective tissue origin.

The appearance varies from a filmy opalescence of the mucosa in the early stages to amore definite grayish-white cast with a coarsely wrinkled surface in the later stages. Thelesions usually occur bilaterally and are most noticeable along the occlusal line in the bi-cuspid and molar region. Diagnostically, one can stretch the tissue and the white essen-tiallv disappears. Important: Leukoplakia would not disappear when stretched.

Important point: Leukoedema appears to be simply a variant ofnormal mucosa and notreatment is necessary; merely diagnosis. See picture #73 in booklet

The differential diagnosis should include:. Leukoplakia. Snuffpouch (SP). Frictional keratosis. \Vhite sponge nevus. Hereditary benign intraepithelial dyskeratosis

Histologically, in leukoedema, the epithelium is parakeratotic and acanthotic, with markedintracellular edema of spinous cells. Note: The white appearance ofleukoedema is causedbv s ater within the spinous cells causing the light to reflect back as whitish.

Remember: Snuff pouch (SP) is a form of hyperkeratosis with various degrees ofclin-ical manifestation (1.e., white mucosal change). SP develops on those mucosal sites wherethe tobacco is held. The causal agents ofSP are considered to be the nitrosamines and hy-drocarbons contained in tobacco. Prolonged use ofthis habit may conduce to the devel-opment of a squamous cell carcinoma due to the carcinogenic potential of thosecomponents. See picture #77 in booklet

Page 175: Oral Pathology

ORAL PATHOLOGY WLes

A 67-year-old Caucasian male comes into your oflice for a routine check-up, Herelates to you that he just got back from Florida where he goes for the fall and

winter months. He enjoys taking his boat out with his wife. Your extraoralexam shows chapped lips but his lower lip also presents with grayish-white

plaques. There is marked loss ofelasticity of the vermillion border.Which of the following would you make your diagnosis:

. Actinic keratosis

. Actinic cheilitis

. Actinic dermatitis

. Solar lentigo

174Copyright O 20ll-2012 ' Denlal Decks

ORAL PATHOLOGY WLes

An incisional biopsy is indicated for which of the following lesions?

. A.5 cm papillary fibroma ofthe gingiva

. A 2 cm exostosis ofthe hard palate

. A 2 cm area ofFordyce's disease ofthe cheek

. A 3 cm hemangioma ofthe tongue

. A 3 cm area of leukoplakia ofthe soft palate

175Copyrighl (l20ll ?012 DentalDecks

Page 176: Oral Pathology

Actintc (solar) cheilitis, a variant oforal leukoplakia, is considered to be the labial coun-

terpart of solar (actinic) kerctosts (a precursor of SCC of the skin). The lips appear drymottled, and opalescent with slightly elevated white or gray plaques and that cannot be

stripped off Isolated areas ofhyperkeratotic callus may also be evident as well as loss ofelasticity and definition ofthe vermilion border. See picture #74 in booklet

The short-term effects of exposure to UV light (especioll.v UVB, 2900 to 3200 wn) are

transient, but the cumulative long-term effects produce irreversible damage (actinic cheili-/lr, usually to the lower lip ofexposed individuals.

Other clinical signs include:. Eq.thematous or hemorrhagic areas. Parallel marked folds. An unobtrusive "chapped lip" appearance. The junction of vermilion and skin becomes indistinct.

Malignant change is manifested clinically by areas of more diffuse cheilitis and ulcera-

tions ofrelatively long duration. Although degenerative changes have been observed pre-

dominantly in men after the age 40, the condition now is increasingly recognized in

) ounger men.

Important: This condition is considered premalignant and may lead to squanous cellcarcinoma. It should be treated accordinslv.

Leukoplakia is a premalignant lesion. This means that ifleft untreated, some ofthe lesions progress

r(r carcinoma. [t is bccause ofthis chance ofmalignant translbrmation that all leukoplakias should

be biopsied, Note: Leukoplakia is a clinical diagnosis.

. Idiopathic leukoplakia: refers to white/opaque oral mucosa lesions that do not rub off and

are not clinically diagnostic for any other whitc lesions. The causc is unknown, however, tobacco

iitation (especialll pipel and alcohol may be contributing facto.s. [t is more common in older

men. lt is a slowly developing change in a mucous membrane characterized by thickened, white,

1irmly attached patches that arc slightly raised and sharply circumscribed. Lesions ofthe floorof the mouth and base ofthe tongue are most aggressive. Most display no dysplasia but can

de\ elop into rnalignancy (5'% to I5o/A.ln all cases, leukoplakia must be completely excised

since diagnosis cannot be made clinically. See picture #75 in booklet

. Proliferative verrucous leukoplakia: it is a high-risk form of leukoplakia The cause is un-

knorvn, although somc are associated with human papillomavirus 16 and 18. Lesions arc re-

cuffent or persistent and usually multiple. Lesions may start llat but progress to broad-based'

v artllke (vermcforn) lesions. There is a high risk ofmalignant transformrtion to verrucous

carcinoma or squamous cell carcinoma.

Explanation oflesions on front ofcard:. Papillary fibroma: is a benign neoplasm ofconnective tissue ongin. Exostosis of the hard palate: lori (p!:tlatal or nandibulat): most cotr..rnon exophytic lesions,

slou-grou rng benign Lnots ofbone. Fordyce's disease (or granules): ectopic sebaceous glands in the buccal mucosa and/or lipThey are present in over 757o ofadults. They usually appear as yellow, sometimes yellow white

submucosal clusters that are essentially normal. See picture #76 in booklet

Note: When mechanical irritation produces a white lesion it is called frictional keratosis.

Page 177: Oral Pathology

ORAL PATHOLOGY

You are conducting a routine exam on a 54-year-old patient withdiabetes mellitus t'?e 2 and a 20-year pack history of smoking.

You see a white patch on left alveolar edentulous ridge. The lesioncannot b€ wiped off and the patient denies a history of traumaor allergies. Which ofthe following would be your diagnosis?

. Squamous cell carcinoma

. Lichen planus

. Erythroplakia

. Leukoplakia

175

Copyright (i 201l':01: - Dental Decks

ORAL PATHOLOGY WLes

A 75-year-old patient comes to your olfice wanting a new set of dentures,She hasn't be€n wearing her old d€ntures for rbout 2 years. She has a

collapsed \aDO and her physician is conc€rn€d rbout her iron deficiency.The corners ofher mouth are fissured, dry, and erythernatous.

Which ofthe following conditions is the likely diagnosis?

. Squamous cell carcinoma

. Angular cheilitis

. Vemrca vulgaris

. Stomatitis nicotina

177

Copyrighr e 20ll-:0ll - Dcntal Decks

Page 178: Oral Pathology

The etiology ofleukoplakia is thought to be a varied one. Possible etiologic tactors includetobacco, aicohol, and oral sepsis. It is most often due to tobacco use. Some investigatorsbelieve that pipe smoking is most harmful.

Leukoplakia is a clinical white patch or plaque on the oral mucosa which will not rub offand which cannot be characterized as any specific disease. Most reports indicate thatleukoplakia is more common in elderly men. Although less common than leukoplakias.erythroplakias, have a much greater potential for becoming malignant.

Important: Any white or red lesion that does not resolve itself in two weeks should be

reevaluated and considered for biopsy to obtain a definitive diagnosis.

The term carcinoma in situ is applied to mucosal lesions which resemble leukoplakia inall respects €xcept that dysplasia is very pronounced and involves almost all epithelial lay-ers. It shows no tendency to invade or metastasize to other tissues.

The clinical differential diagnosis ofa white patch should include:. Leukoplakia. Lupus erlthematosus. \\'hite sponge nevus. -A. chen.rical / thermal bum

. Candidiasis

. Lichen planus

. Migratory glossitis/stomatitis

. Frictional hyperkeratosis

It is also associated with the loss of vertical dimension.This situation is generally ob-sen ed in elderly patients.The corners ofthe mouth become painful, iffitated, red, cracked,and scaly. The fungus Candida albicans (thrush) may grow in the corners ofthe mouth,keeping them sore. Note: It can also result from a bacterial (i.e., Staph)'lococcal) infec-tlon.

It occurs in individuals that habitually lick their lips and deposit small amounts ofsalivain the commissural angles. It is also associated with nutritional deficiencies (i.e.. vitaminB-) [riho.favinJ, vitamin B-3 [niacinJ, vitamin 8-6 [pvridoxineJ. or vitamin B-12

Io anoc obalaminJ, or a deficiency in iron).

\ote: Nvstatin will invariably eliminate the fungal infection and an antibacterial is usedlo treat the bacterial inlection ifpresent.

See picture #78 in booklet

Page 179: Oral Pathology

. Hyperkeratosis

. Leukoplakia

. Epidermolysis bullosa

. White sponge nerus

178

Coplrighr C 201 l'2012 - Dental Decks

Hairy tongue is a condition chrracterlzed by hypertrophy of the:

. Filiform papillae

. Fungiform papillae

. Circumvallate papillae

. Foliate papillae

179

Coptrighr O 201l-2012 - Dental Decls

Page 180: Oral Pathology

White sponge nevus is an autosomal dominant trait due to mutations ofkeratin 4 and/or 13. tthas no sex preference. lt presents as an asymptomatic. deeply folded, white or gray lesionthat may affect several mucosal sites. Lesions tend to be somewhat thickened and have a

spongy consistency. The presentation intraorally is almost a)ways bilateral and symmetric and

usually appears early in life, typically before puberty. The buccal mucosa is the usual loca-

tion. See picture #79 in booklet

Important: There is no treatment for $,hite sponge nevus! howevel since the condition isperfectiy benign, the prognosis is excellent. There are no serious clinical aomplications.Note: lt is often mistaken for leukoplakia.

Nlicroscopically, a featufe that unique to prickle cells lbund in rvhite sponge nevus is per-

inuclear eosinophilic condensation of cytoplasm.

Note: Hyperkeratosis is an abnonnal increase in the thickness of the keratin layer (stratum

corneun) of Ihe epithelium. It is one ofthe most common white cheek lesions ofthe oral mu-

cous rnembranes (often in an area of cht onic cheek biting) and presents as being thick and

scailr.

Epidermolysis bullosa is a general term that encompasses one acquired and several genetic

t ariet\es (dystrophic, .jtnctional, sizpler/ of disease that are basically chamcterized by thelbnnation ofblisters at sites of minor trauma (especiall ot'er tlrc elbov and kleesl. Severalgenetic types range from autosomal dominant to autosomal recessjve. The feature common toa1l subtypes of epidermolysis bullosa is bulla formation from minor provocation, usuallyor er areas ofstress, such as the elbows and knees. Oral lesions are panicularly common and

:er ere in the recessive forms and uncommon in the acquired form. These lesions include bul-lae. scaning. and hypoplastic teeth. These lesions are most pronounced in the type known as

recessi!e dystrophic epidermolysis bullosa.

Hain tongue is a benign condition ofthe tongue. The dorsum ofthe tongue appears furrydue to the elongated papillae. The color varies lrom yellowish-white to brown or black.\ote: It is associated with poor oral hygiene, extended use ofantibiotics, cofticosteroids.hr droven peroxide and smoking. See picture #80 in booklet

The four trpes of papillae present on the tongue:1. Filiform: most numerous, small cones aranged in "V"- shaped rows parallelingthe sulcus terminalis. They are characterized by the absenc€ oftaste buds and increa-sed keratinization.l. Fungiform: scattered among the filiform papillae, they are flattened, mushroomshaped and found mainly at the tip and lateral margins.-1. Circumyallate: iargest, have circular shape. Arranged in an inverted ,'V',- shapedrou tolard the back ofthe tongue. Associated with the ducts ofvon Ebner's glands.Thev are the least numerous of all papillae.l. Foliat€: found on lateral margins as 3-4 vertical folds.

Page 181: Oral Pathology

. Lupus erylhematosus

. Erythema multiforme

. Pemphigus vulgaris

. Lichen planus

. Leukoplakia

. White sponge nelus

. Candidiasis

. Lichen planus

180

Cop)Tiglt O 2011-2012 Dental Decks

t81CoplriSht O 201 I,20 12 - Dmral D€cls

Page 182: Oral Pathology

These lacelike white striae, the so-called Wickham's striae, are a classic presentation oflichen planus. They are often bilateral and symmetrical in distribution.

Lichen planus is a fairly common inflanmatory disease that usually affects the skin,the mouth, or sometimes both. It aflects women slightly more than men, and occurs mostoften in middle-aged adults. TJymphocytes destroy basal keratinocytes, however. the rea-

son for this immunologically mediated phenomenon is unknown. Lichen planus of themouth most commonly affects the buccal mucous membrane. It may also been seen onthe tongue, lips, hard palate and gingiva. The lace-like striae are usually asymptomatic,but sometimes the patient may complain ofa burning sensation. The intraoral lesions re-spond to topical steroid therapy. In addition to the usual form of lichen planus, there are

lrvo other foms, bullous and erosive. [n the bullous forrn, fluid-filled vesicles projectfrom the surface. [n the erosive form, the lesions are intensely red or raw-appearing.When drese lesions of erosive lichen planus involve the gingiva, they resemble desqua-

mative gingivitis. See picture #81 in booklet

The microscopic appearance of lichen planus is characteristic and pathognomonic:. Hyperparakeratosis with thickening ofthe granular cell layer. Deyelopment ofa "salv tooth" appearance ofthe rete pegs. Degeneration ofthe basal Iayer ofcells. lnilltration of inflammatory cells into the subepithelial layer ofconnectrve tissue

Oral candidiasis also kaown as thrush, causes white, curd-like patches in the mouth orlhroat. These patches typically appear on the tongue, inside ofthe cheeks, or on the palate.Oral candidiasis typically occurs in people rvith abnormal immune systems. These caninclude people undergoing chetrotherapy for canceq people taking immunosuppressivedmgs to protect transplanted organs, or people with HIV infection.

Candidiasis is caused by a yeasrlike fungus, C. albicans, rvhich causes an inflammatory,pruritic infection characterized by a thick, white discharge. Acute lesions appear as dif-tirse. curly or velvety white mucosal plaques on the cheeks. palate and tongue rhat car belriped off, leaving a red, raw or bleeding surface. Note: Chronic lesions are erythematous.

This veast-like fungi is a normal inhabitant of the oral cavity and vaginal tract, howeverit is normally held in check by the indigenous bacteria of these areas. The treatment fororal candidiasis is topical through the use of lozenges (also called trouc}esl and mouthrinses. the most widely used is nystatin, Note: Systemic treatment includes the use oflluconazole or ketoconazole.

l Acute pseudomembranous candidiasis is the most common fom of oral\otes' candidiasis and is usually found on the buccal mucosa, tongue and soft palate.

....r. Oral cytology smears are usel'ul for a diagnosis (it will reveal budding otgqn-isnts *ith branching pseudohyphae).2. Angular cheilitis (Perleche) has also been linked to C. albicans.3.Factors that may stimulate Candida growth include: the extended use ofan-tibiotics, steroids, diabetes, pregnancy, or a deficiency in iron, folate. MtaminBrr or zinc.

Page 183: Oral Pathology

ORAL PATHOLOGY WLes

The "white patch" seen below has been present for nine months on a patientwho is a heavy pipe smoker. What is the treatment of choice?

142CopyriShr C 2011 20ll Denral Decks

ORAL PATHOLOGY WLes

A 3s-year-old healthy female pr€sents to your office for routine dental workWhile completing the restorativ€ treatment on the LL quadrant you notice thather tongu€ has multiple irregularly shaped red lesions that have a white trorder.

You make a note in your chart. When she returns two weeks later for therestorative work on the LR quadrant, you note that there are still lesions,

but in different locations on the tongue with different shapes.What is your diagnosis?

. Fissured tongue

. Macroglossia

. Geographic tongue

. Hairy tongue

183

Copvriglrt a):0ll-l0ll - Dental Decks

Page 184: Oral Pathology

Clinically. this is leukoplakia and should ahvays be biopsied due to the possibility ofit being apremalignant lesion. Remember: Pipe smoking is thought to bc one of the nrost impona nt pre-disposing ctiologic factors for thc dcvclopmcnt ofa lcukoplakia.

Important: The floor ofthe moutir, tongue and lower lip ale the regions at greatcst risk for car-cinoma occurring in leukoplakia.

Stomatitis nicotina is related to pipe smoking (as uell as cigor snrokfugl and occurs exclusivelyon thc palate. It aff'ccts malcs prcdominantly. The palate first appea$ rcd and inflamqd. Soon itdevelops a diff-usc, grayish-white, thickened, multinodular popular appearance rvith a small red

"spot" in thc ccntcr ofeach tiny nodule. This "spot" corrcsponds to the orifices ofpalatal sali-vary gland ducts. Thc treatmcnt for this condition is the cessation ofslnoking. It is usually notconsidcrcd to bc a prcmalignant lesion.See picture #84 in booklet

\ote: In India, thc usc ofsmokclcss tobacco in various forms is vcry popular. This habit, whichusually involves the chewing ofa betel q\id (combined areca nut, betel le.i, tohacco dnd slacklindl. has lcd to thc dcvelopment, in a large proportion ofuscrs. ofa unique generalized fibrosisof thc oral soft tissues, called oral submucous fibrosis. It typically affects the buccal mucosa.Lips. rctromolar arcas and thc soft palatc. Early lcsions prcscnt as a blanching of the mucosa,rmpaning a mottlcd, marble-like appcarance. Latcr lesions dcmonstrate palpablc fibrous bandsrunning vertically in the buccal mucosa and in a circular fashion around the mouth opening or lips.As the disease progresses the mucosa becomes stiff, causing difficulty in cating and considcrablyrcstricting thc paticnt's ability to open the mouth (risnl ir,r/. Ifthc tongue is involvcd. it becomes

sritTand has a diminishcd sizc. Thqre is a fibroclastic transformation of thc.iuxta-epithclial con-necti\ e tissues and an increased risk of oral carcinoma liom the tobacco ofthe quid.

ImDortant: Betel quid and smokcless tobacco also increascs the risk ofvcrrucous carcinoma.

*** lr rs also called eD4hema migrans. benign migratory glossitis, rvandcring rash ofthe tongue, erythema

lreara migrans. and stomatitis arcata migrans.

Grographic tongue is a hannless and very common condition in u'hich there is dcsquamation ofthe fil-iform papillae. h is charactcrizcd by having one or more iregularly shapcd patches on thc longuc. Thc

a-r.r arca is redder lhan the rcst of the tongue and the edges of the patch are whitish in color Thcsc

r:rchcs eppcar and remain for a short time. heal. then reappear aI anothcr site. The patchcs usually do

rrrr r.sprrnd to reatment but disappcnr spontaneously. Note: The patient may complain ofa slight burn-

r:1g ofrhe tonguc. S€e picture #82 in bookl€t

Fis!ured tongue /a/.f.r tttlleclsctotultongue) is charactcrized by a deep median fissure wilh laterally ra-

.iratrns srrrovcs. Thc lateral groovcs vary in number but are usually symmctrical in arrangcmcnt. It is rarc

r:r .illdrcn and increascs in incidence with age. The fissuring occurs across the dorsum ofthe tonere:r.J rs u'ua11r' asyrnptomatic. but may bccomc painful if infccted with Candida Albicans.

See Dictur€ #83 in booklet

\ote: Fissurcd tongue is lbund in NlelkerssorFRosenthal Syndrome (dlong h ilh gtdnulonatous cheili-

t:t and liaial ner\e parallsis).

Remember: Gcographic tongue often occurs in association \\'ith fissured tonguc

\lacroglossia is tonguc enlargement that leads to ftrnctional and cosmctic problenls. Although this is a

relarivcl-v uncommon disorder. it may causc significant morbidity. Thcrc is no clear definition ofmacroglossia and it may bc dcfined in relative, funclional, or structural terms. Causes includci

. An inflammatory cause is chronic glossitis

. Traumatic causes includc postoperative edema

. \Ietabolic causes are myxedcma, amyloidosis, lipoid proteinosis. chronic steroid therapy, and

acromegal). The congenital causes are p n1ary idiopathic rnacroglossia. cretinism, hemangioma, lymphan-gioma, Beckwith-Weidmann Syndrome, Dorvn Syndrome, generalized gangliosidosis syndrome,

and anv of the MuooDolvsaccharidoses.